X



宇宙の質問が書き込まれたら誰かが即答するスレ 44 [無断転載禁止]©2ch.net
レス数が1000を超えています。これ以上書き込みはできません。
0001名無しさん@お腹いっぱい。
垢版 |
2017/02/26(日) 14:51:09.12ID:jQSfeaRa
オカルトや宗教的な書き込みはスレチなのでお控えください
キチガイ発生時には、荒れやすいので各自NG登録を行いスルーしましょう

前スレ
宇宙の質問が書き込まれたら誰かが即答するスレ 43 [無断転載禁止]c2ch.net
http://rio2016.2ch.net/test/read.cgi/galileo/1485726950/l50
0004名無しさん@お腹いっぱい。
垢版 |
2017/02/26(日) 18:34:41.16ID:ibJvLu54
ブラックホールのその後はどうなるの?
0005名無しさん@お腹いっぱい。
垢版 |
2017/02/26(日) 18:45:08.77ID:OJkgQEGy
0007名無しさん@お腹いっぱい。
垢版 |
2017/02/26(日) 18:53:19.87ID:ibJvLu54
銀河と銀河の間には実は岩石も彗星もガスもチリもないんじゃないか?
あくまでも銀河の内部にそれらがあるだけで
0008好奇心旺盛
垢版 |
2017/02/26(日) 22:09:29.94ID:lAeVOrMQ
いつもお世話になってます。
数年前に冥王星を観測していた人工衛星があったと思いますが、
今は何処を観測しているのですか?

また、探査衛星の寿命は何年ぐらいを想定しているのでしょうか?

宜しくお願いいたします。
0009名無しさん@お腹いっぱい。
垢版 |
2017/02/26(日) 23:23:48.03ID:xqo84tMe
ニュー・ホライズンズの事か?というよりこれしかないな
今は再来年接近する予定の外縁天体観測するまで基本待機モードで慣性飛行してるだけ
寿命は電源の原子力電池の出力が落ちて送信出力が無くなるまで数十年かな 
その前に他の機器が壊れるか もう有用なデータは得られないと運用停止になるかどっちかだな
0010名無しさん@お腹いっぱい。
垢版 |
2017/02/26(日) 23:34:47.97ID:xqo84tMe
しかし「冥王星 人工衛星」という不適切な検索ワード(ニューホライズンズはフライバイで衛星ではない)
でも ググれば結果の先頭にニューホライズンズのwikiが出てくるのにな 何でわざわざここに書き込むのか

 
0012名無しさん@お腹いっぱい。
垢版 |
2017/02/26(日) 23:41:12.36ID:ibJvLu54
ググればわかるってそんなこのスレの意味なくなるじゃんw
0013名無しさん@お腹いっぱい。
垢版 |
2017/02/27(月) 01:26:10.87ID:k+2dz/hN
思考実験だけど
内側が鏡ばりの球体の中に光を閉じ込めて密封出来たとしたらどうなるんだろ
鏡の反射率が100%じゃないなら容器の熱量に変換されるだろうけど思考実験だから100%として考えて
0018名無しさん@お腹いっぱい。
垢版 |
2017/02/27(月) 02:36:32.08ID:x262f4k0
光→熱
闇→寒
部屋を暗くして、擦ると熱を持ち赤くなる
熱を持つと燃える
0019名無しさん@お腹いっぱい。
垢版 |
2017/02/27(月) 03:07:51.34ID:Dyk85+Z0
なぜ地球と近距離のGPS衛星で実際にそのような時差が生じるんだろう。

太陽系は銀河を公転してるし、銀河も時速何十万キロで公転してる
そんな中の小さな太陽系の中の地球と衛星の中で時差なんて
って思うが実際時差は生まれてるわけなんだよな
何を基点に時差が生まれるのかがわからん

時間は速度ではなく重力に影響されることがわかってきたって言ってる割に
重力を根源から否定するような説も生まれてたりするよね今
0021名無しさん@お腹いっぱい。
垢版 |
2017/02/27(月) 04:06:52.17ID:RFwaSz4V
>>19
> 時間は速度ではなく重力に影響されることがわかってきたって言ってる割に
GPS衛星については地上との速度差による時間の遅れももちろんあるのだけど
重力の影響に比べて一桁ぐらい少ないと言うだけ。
0022名無しさん@お腹いっぱい。
垢版 |
2017/02/27(月) 04:13:23.86ID:RFwaSz4V
>>19
> 何を基点に時差が生まれるのかがわからん
地上(の時計)とGPS衛星(の時計)の重力ポテンシャルの差と相対速度。
比較対象とする2点以外を持ち出しても意味がない。
0023名無しさん@お腹いっぱい。
垢版 |
2017/02/27(月) 04:24:19.39ID:v6t2Aj0o
EMドライブの原理の解明はまだか?

推進剤を使うことなく真空で宇宙船に推力を与えられる電磁(ElectroMagnetic)推進システム「EMドライブ」

燃料不要の夢の宇宙エンジンは可能、NASAが発表
http://natgeo.nikkeibp.co.jp/atcl/news/16/112400450/
物理法則に反したエンジン
ところが、EMドライブは何も噴射しない。これがニュートンの第3の法則と、
もうひとつの古典力学の教義である運動量保存の法則と矛盾するのだ。
EMドライブが何も後方に噴射させずに前進するとすれば、その推進力を説明する反作用の力が何も存在しないことになる。
0025名無しさん@お腹いっぱい。
垢版 |
2017/02/27(月) 09:54:18.52ID:LFw+OLE5
まだ光は減衰しないって話にこだわってる奴がいるのか
完全な真空、100%の反射材、どちらも現実の宇宙には存在しない
自分の脳内宇宙の質問されても困る
0029名無しさん@お腹いっぱい。
垢版 |
2017/02/27(月) 12:40:47.26ID:qAus1p96
>>23
マイクロ波の閉じ込めにより磁場が発生
その磁場が極僅かに空間の歪みを発生させる
その歪みに流される形で動力が発生する。
これは俺の根拠のない妄想だが、発想だけを提供してみたw
0031名無しさん@お腹いっぱい。
垢版 |
2017/02/27(月) 12:44:32.53ID:qAus1p96
そもそも磁場の力が強すぎて、たとえ副作用で重力が僅かに発生していても検出できないんじゃないのかな?
0032名無しさん@お腹いっぱい。
垢版 |
2017/02/27(月) 12:47:58.94ID:qAus1p96
>>19
時間の基点に絶対的なものはない。全て相対的なもの。
適当な基点を決めて、そこのとの比較でしか時間差を計れない。
宇宙において絶対的な基点となり得るのは光速度不変のみである。

空間も時間もふにゃふにゃな水みたいなものだ
0033名無しさん@お腹いっぱい。
垢版 |
2017/02/27(月) 12:58:18.42ID:qAus1p96
電磁波って波長が長いほど反射させることが容易になるよね?
電子レンジの窓なんてパンチング穴のある金属で中が見えるけどマイクロ波の波が大きいから穴を通過できず閉じ込められる。

逆に波長が短すぎると粒子の性質が強くなるから
これまた閉じ込める易くなるのではないの?
0034名無しさん@お腹いっぱい。
垢版 |
2017/02/27(月) 13:23:30.21ID:CsHv89jC
>>33
波長短いほど屈折率大きくなるから、全反射させやすいとか
超電導みたいに完全に透明な物体ってない?
0035名無しさん@お腹いっぱい。
垢版 |
2017/02/27(月) 13:35:12.58ID:qAus1p96
> 当てた光と同じ波長を持つ光が強力に放出される現象を、誘導放出と呼びます。
http://www.ielement.org/er.html

光の損失分を上回る光を増幅させる物質があるから
これで囲えば永久に光が中で発生され続けるのかな
0036名無しさん@お腹いっぱい。
垢版 |
2017/02/27(月) 13:49:36.06ID:WmgURfKq
レーザー光を当ててエネルギー状態を上げ続ける必要があるから、外部エネルギーは必要でしょう。
0038名無しさん@お腹いっぱい。
垢版 |
2017/02/27(月) 19:21:20.44ID:K4WaHm8m
>>37
合ってると思う。
ビッグバンの光は宇宙中を飛び回っているが、何らかの物質にぶつかってエネルギー変換するのは、ほんの一部なんだろうね。
0039名無しさん@お腹いっぱい。
垢版 |
2017/02/28(火) 01:50:09.39ID:Cahrw9pl
恒星出来る→年月経つ→超新星→年月経つ→ブラックホール→???
なんだけど
恒星出来る→年月経つ→超新星→年月経つ→ブラックホール→年月経つ→規模の小さなビッグバン→恒星出来るじゃね?
0042名無しさん@お腹いっぱい。
垢版 |
2017/02/28(火) 03:41:27.67ID:bM4WGbzY
>>39
ブラックホールに重力が存在してる(他の3つの力と分かれてる)時点で
ビッグバンと同じ状況にはなり得ないんじゃないかな
0044名無しさん@お腹いっぱい。
垢版 |
2017/02/28(火) 04:03:13.04ID:oPrjNKJ3
電磁気力を伝える粒子は光子ですが、
暗闇でも磁石が反発しますし光子がほんとに媒介してるの?
0045名無しさん@お腹いっぱい。
垢版 |
2017/02/28(火) 07:07:12.24ID:bM4WGbzY
>>44
磁石が反発してる時、正確には磁石が動いてる時になるけども、
目には見えないけど光子が放出されてる…はず
理屈で言えば乾電池を動かすだけでも放出されるんでないの?
0046名無しさん@お腹いっぱい。
垢版 |
2017/02/28(火) 09:17:24.16ID:3tZDGyj9
>>44
じゃあ暗闇で電球やLEDが光るのはなぜ?(物質から光が放出されるのはなぜ?)
日常的に身の回りにあるものなので意識することは無いと思うけど、なぜかと聞かれると答えられないですよね
それはなぜかと反復質問されると根本的に答えがでる現象は一つもない

ちなみに、電磁相互作用は周囲に満ちてる光が力を媒介するのではなく、
物質の一部のエネルギーが光として放出されるイメージが近いのかな?

互いにエネルギーの一部を(光で)交換するけど、
ベクトルが異なるエネルギーを吸収することで粒子の軌道が変化する
力というと押したり引っ張ったりするイメージがあるけど、
電磁相互作用は、ベクトル合成に近いイメージですね
0047名無しさん@お腹いっぱい。
垢版 |
2017/02/28(火) 09:23:56.98ID:3tZDGyj9
要するに、右に進む光子を沢山吸収すれば粒子もだんだん右に進む
それと同時にいらないエネルギーを捨てる(交換)
(押したり引っ張ったりはしない、ただのベクトル合成で説明ができるということ)
0049名無しさん@お腹いっぱい。
垢版 |
2017/02/28(火) 12:48:23.90ID:K4yuaXpt
仮想なの?
ネオジム磁石なんて強力で、ムキムキの人間が引っ張ってもびくともしなほどの結合力
この強力な力を光子で伝達してるというのが不思議
0051名無しさん@お腹いっぱい。
垢版 |
2017/02/28(火) 13:51:16.63ID:K4yuaXpt
>>50
サンガツ
なんとなくわかった。
磁石の場合、磁場だけで電場が発生してないから、電磁波という光は全く発生しない。
目に見える光は電場と磁場の振動波なだけで
光子そのものは電場だけでも磁場だけでも活躍しているということかな。
この場合、目に見える光(電磁波)は関係ない
0052名無しさん@お腹いっぱい。
垢版 |
2017/02/28(火) 13:55:23.12ID:K4yuaXpt
あと磁力の伝わる速さが光速度と同じというのも力の伝達に光子が介在しているというのも説得力がある
0053名無しさん@お腹いっぱい。
垢版 |
2017/02/28(火) 13:58:12.63ID:K4yuaXpt
ようは電場と磁場が暴走したり、撹乱されて飛び散ったものが、電磁波という光になるという感じなのかな。
0057名無しさん@お腹いっぱい。
垢版 |
2017/02/28(火) 14:06:52.82ID:MnVh/t4U
エネルギー基底状態の場である時点で、無という意味の真空ではないな
真空=無という考え方は理論物理学者はもう誰もしていない
だからこそ真空期待値という真空なのに有限値の値が出てくるわけで
0060名無しさん@お腹いっぱい。
垢版 |
2017/02/28(火) 15:37:15.93ID:bM4WGbzY
>>59
実在してないと説明できない現象があるんだよ(詳しい事は「光電効果」で)
アインシュタインは光電効果を「光量子仮説」で説明して、
ミリカンっていう物理学者がこの仮説を実験で証明した事でノーベル賞を貰った
(ミリカンも2年後にノーベル賞を貰ってる)

相対論ではノーベル賞貰ってないんだよね
0061名無しさん@お腹いっぱい。
垢版 |
2017/02/28(火) 16:23:48.52ID:5xsEDVt2
ホログラフィック宇宙で考えると
この光子のふざけた万能的性質で世の中を描けるな。

光子がペンキで、強い力弱い力が密着性を高める下塗り剤
重力が絵を固定する額縁だ。
0062名無しさん@お腹いっぱい。
垢版 |
2017/02/28(火) 18:01:03.89ID:RNbLqvNA
光子も電子も、重力波も観測されているのは分かりますし、実在は納得しています。
ところで、磁力の素となる素粒子とか磁力波とか、磁力の基となるものは何なのか分かつているのですか?
そして、その磁力の基となるものの影響力はどこまで及ぶのでしょうか?
重力や光のようにどこまでも及ぶのでしょうか?
0065名無しさん@お腹いっぱい。
垢版 |
2017/02/28(火) 21:23:38.80ID:XdMfeqVF
磁力の元は電子
0071名無しさん@お腹いっぱい。
垢版 |
2017/03/01(水) 00:21:48.60ID:Xlv2H5c3
>>69の表現が的を射てると思う
4番目の次元に時間を含めたため距離(位置)と時間が独立ではなくなり幾何学的に速度に上限ができそれが比例定数のcになるんじゃないかと
だから質量のない粒子は全て光速なのだろう
0072名無しさん@お腹いっぱい。
垢版 |
2017/03/01(水) 00:50:50.66ID:gEci3ZoQ
質量ゼロ、大きさゼロ、でも実在する粒子であって波でもある。

ふざけすぎだろw 
チートだろこれ。設計したの誰だよ
0073名無しさん@お腹いっぱい。
垢版 |
2017/03/01(水) 00:52:22.07ID:F4bs725j
地球以外にまだ生命は発見されてませんが、地球外生命を見つける目的はなんでしょうか?
1.地球以外にも生命がいたって事は我々は孤独ではないのだぁと喜び人類同士の戦いは馬鹿げていると思わせるため
2.その生命が生きれる場所って事は我々もそこに行けば生きられると考え、移住改革を立てるため
3.地球にいる生命とDNAが似ているのかを調べ、似てたら我々を含む地球生物は宇宙から運ばれたとなり、似ていなければ地球生物は地球で生まれたとわかるため
4.その他
0075名無しさん@お腹いっぱい。
垢版 |
2017/03/01(水) 01:02:12.09ID:gEci3ZoQ
>>73
1、進化を早める為には競争相手が必要なので人間は同種間の争いよりも新たな敵を欲しがる(本能的に)
2、距離的な制約から現実的でない。隣の惑星に移住することのほうが優先順位が高いであろう。
3、大した目的にはならん。
4、地球外生命を見つける目的のプロジェクトは散発的で資金がほとんど出ない。
 あくまで観測による発見を増やし物理学や宇宙の謎解明や発展、新技術の発展に繋げるのが目的。
 地球外生命をネタにする報道には、人々の関心を集め、投資や資金調達を増やすのが目的。
 もちろん子供達に夢を与え興味を抱かせ、立派な物理学者を志す人を増やすことにも繋がる。
0076名無しさん@お腹いっぱい。
垢版 |
2017/03/01(水) 01:08:18.74ID:gEci3ZoQ
近隣の系外惑星の大気に酸素があることが観測で確認されたとしたら、生物がいる可能性が大きくなるだろう。
そして人類は次に何をするであろうか?

1、その星に向かって信号を送る
2、信号を送ることを禁止し、ひたすら覗こうとする
3、世界が恐怖に包まれパニックになる
0077名無しさん@お腹いっぱい。
垢版 |
2017/03/01(水) 01:11:44.65ID:SVd55EfI
光子だけじゃない、
厳密にいえば粒子も原子も何も実在してはいないし、実在してるとも言える
その概念が本質を正しく表現できていないという意味で

例えば、リンゴは実在する
じゃあ、リンゴの欠片は実在しないのか
どこからどこまでがリンゴで、原子で、粒子なのか
現実の物質や現象にはそんな区切りは無いのに、人は現実の一部を切り抜いて特徴ごとに名前を付ける
0078名無しさん@お腹いっぱい。
垢版 |
2017/03/01(水) 01:15:20.98ID:SVd55EfI
粒子や原子の視点で見れば、リンゴとその周囲の空気にさほど違いがないというか、リンゴという区切りは無い
実在するとはそういう事
0079名無しさん@お腹いっぱい。
垢版 |
2017/03/01(水) 01:21:29.26ID:gEci3ZoQ
エントロピーの視点で見ると、太陽も火星も月も、エントロピーが増大しきった状態が現状の姿ですよね。

地球は人間がどんどん複雑な構造物を作り、物質を精錬し加工し、エントロピーを縮小させている。

生物の役目というか機能はエントロピーの縮小作業ではなかろうか。
0080名無しさん@お腹いっぱい。
垢版 |
2017/03/01(水) 01:59:26.29ID:SVd55EfI
>>72
違いを考えることは矛盾のない新しい概念を生み出すのに役立つけど、
違う事それ自体を主張するのはただの哲学不足です
本質を正しくとらえてる概念など一つもないので、どこかで矛盾が生じてくるのは当たり前なのです
0082名無しさん@お腹いっぱい。
垢版 |
2017/03/01(水) 07:32:08.87ID:L0IT3/87
>>79
生物は局地的にエントロピーを縮小させるけど、生物を含めた系全体ではエントロピーは
やっぱり増大するよ。

>エントロピーの視点で見ると、太陽も火星も月も、エントロピーが増大しきった状態が現状の姿ですよね。

君はエントロピーが何かについて理解できてない。
0083BIG1等当せん
垢版 |
2017/03/01(水) 09:26:06.43ID:mbJ0Q2pt
ごく稀に数秒にわたって発光する流星もあり、これを運良く見ることができれば願い事を3回唱えることも不可能ではない。
0085名無しさん@お腹いっぱい。
垢版 |
2017/03/01(水) 09:35:35.52ID:mbJ0Q2pt
ごく稀に数秒にわたって発光する流星もあり、これを運良く見ることができれば願い事を3回唱えることも不可能ではない。
0087BIG1等当せん
垢版 |
2017/03/01(水) 09:45:04.85ID:mbJ0Q2pt
ごく稀に数秒にわたって発光する流星もあり、これを運良く見ることができれば願い事を3回唱えることも不可能ではない。
0089名無しさん@お腹いっぱい。
垢版 |
2017/03/01(水) 11:44:27.36ID:XDJaPqhz
重力波にもドップラー効果はありますの?
0091名無しさん@お腹いっぱい。
垢版 |
2017/03/01(水) 14:07:41.84ID:XDJaPqhz
>>90
なるほろ
どうもありがとう
0092名無しさん@お腹いっぱい。
垢版 |
2017/03/01(水) 15:47:58.41ID:ZEZe2Fy1
いやだからさ、しつこいようだけど、磁石のS極とN極の間に磁場が発生するのは、どういう粒子が関係しているの?
電場や光は発生していないから、光子は関係ないよね。
磁場が発生していて、磁力線が鉄粉で目に見える実験はよく知っているが、その磁力線はどういう粒子が関係しているのか教えて下さい。
0094BIG1等当せん
垢版 |
2017/03/01(水) 16:20:21.83ID:hk36WRQD
ごく稀
0095BIG1等当せん
垢版 |
2017/03/01(水) 16:21:58.27ID:hk36WRQD
ごく稀×運良く見る=BIG1等当せん
0096名無しさん@お腹いっぱい。
垢版 |
2017/03/01(水) 17:30:51.76ID:SVd55EfI
電子のスピンが綺麗に整列してるのが磁石で、
それが同じ向きか違う向きか、それしかないです
S極やN極といった区別は無いです
0100名無しさん@お腹いっぱい。
垢版 |
2017/03/01(水) 21:26:33.43ID:1fNmgwHN
>>98
みつこー!
0102名無しさん@お腹いっぱい。
垢版 |
2017/03/01(水) 22:01:49.57ID:F4bs725j
>>93
つぶこー!
0108名無しさん@お腹いっぱい。
垢版 |
2017/03/02(木) 00:57:59.09ID:stZR607R
>>97
ケチつける気はまったくないんだけど、なんの役にたつのかまったくわからない
すぐ壊れるからガンの治療にも使えなさそうだし
税金を使って研究者が名前を残すごっこしている

ケチをつけていることになるのか
0110名無しさん@お腹いっぱい。
垢版 |
2017/03/02(木) 01:26:47.13ID:33MAcLQC
>>109
なんの根拠も説得力もない意義が最後にチラっと書いてあるな
つうか理化学研究所のホムペかよw
国に研究資金出して貰ってんだから、くだない研究ですとは書けんわなw
0111名無しさん@お腹いっぱい。
垢版 |
2017/03/02(木) 02:20:13.88ID:C/qHx8cc
物理学の中での意義としては量子力学の理論を確認・証明するため
って事になるみたいだけど…少なくとも現代の実生活の範疇では生かし所が全く無いかな
癌治療って事なら陽子線がある程度有効性を示してるしね

とは言え将来も必要無いかどうかはわからんからなぁ…
こういった事に使ってる技術を転用して何か生まれるかもしれないし
やれるんならやるべきでないの?
0115名無しさん@お腹いっぱい。
垢版 |
2017/03/02(木) 06:26:07.25ID:VK27F6sU
ドレイクの方程式で出された答えは正しいのでは?
宇宙誕生から1万の文明があった、ある

宇宙科学で面白いのは昔は間違いでも今は正しくなったり、昔は正しいのに今は間違いとなったりするところ
0116sfc ◆.6saxwJgXRBD
垢版 |
2017/03/02(木) 06:30:46.28ID:tb4fM5QU
ごく稀に数秒にわたって発光する流星もあり、これを運良く見ることができれば願い事を3回唱えることも不可能ではない。
0117名無しさん@お腹いっぱい。
垢版 |
2017/03/02(木) 06:35:07.31ID:VK27F6sU
地球のように多種多様な生物がいるのは逆に珍しいのでは?
地球はそれ程生命体に適しているから、これだけ多種多様な生物が存在してるだけに過ぎず、大抵の惑星は生命の種類が少なく、その分、争いも少ないと考える

その為、その惑星でトップになる種類は地球にいるより簡単かも知れない
0118名無しさん@お腹いっぱい。
垢版 |
2017/03/02(木) 06:40:51.41ID:VK27F6sU
地球生命は身を守る為や食料を取るために進化した
それ程、敵となる種類が多い
人類はどうか?人類は他の種類に比べて形を極端に変えるような進化はしていない
つまり、敵がいなければ進化しなくて済む
もっと言えば進化するから頭が良くなるではない、進化する必要がない者の方が頭が良くなる、そんな気がする
0121名無しさん@お腹いっぱい。
垢版 |
2017/03/02(木) 10:11:40.78ID:rM+q6gy6
ブラックホールも何らかの物質でできてるけどなにからできてるの?
ほこりもちりもつもれば目に見えるくらいの固まりなるけどブラヅクホールも吸い寄せられた物がひっつきまくったら大きくなるの?
0124名無しさん@お腹いっぱい。
垢版 |
2017/03/02(木) 13:24:14.07ID:f15UVrgW
>>115
入力するデータがなけりゃ意味ない
この前の放送大学では地球が出来たときの海は猛毒だから大陸がなければ生命は無理だってさ
海の深さが±1km違ってたら生命なしの地球になるくらい条件が厳しいから地球生命は孤児だって
0126名無しさん@お腹いっぱい。
垢版 |
2017/03/02(木) 13:49:23.97ID:33MAcLQC
>>117
生命体の絶滅を防ぐのと、環境維持においては多様性があればあるほど良い。
水槽という隔離された環境内で魚を飼育する時の初期立ち上げも、
種水を入れて多種多様なバクテリアを最初から用意したり、複数種の魚や貝類、水草、エビ類など、種を越えたたくさんの生物を入れたほうが圧倒的に水槽環境が安定する。
病気にもなりにくくなる。放置でも水槽内が全滅ということが難しくなる。
0128名無しさん@お腹いっぱい。
垢版 |
2017/03/02(木) 13:54:50.09ID:33MAcLQC
>>121
大きくなったホコリもチリも、量子の世界で見れば、狭間だらけのスッカスカ。
物質は力によって大きさを作られているのであって、実際には大きさなど無い
0129名無しさん@お腹いっぱい。
垢版 |
2017/03/02(木) 14:02:49.88ID:33MAcLQC
物質が可視化されていのは「強い力と弱い力」によって大きさのある物質として作られているから。
ようは遠くある風船が見えるのは、風船内の力によって膨らまされているから。
風船が潰れていたら、遠くから見えない。

ブラックホールは物質が潰れているなら可視化されていない。
中に入っても見えないだろう。
そして物質の質量や種類や性質といった全情報は、潰れた物質から分離していてブラックの周囲に保持される。
0134名無しさん@お腹いっぱい。
垢版 |
2017/03/02(木) 16:38:39.47ID:C/qHx8cc
>>132
初期の地球はそんな感じだったらしいね

最初の海は硫酸とか塩酸で酸性、岩石との反応で中和が進むとCO2が溶けるようになる
で、CO2が増えてくるとそれを利用する生物が出てくる(光合成する藍藻類)
それによって酸素が反応しきれないほど大量に作られるようになって、
>>133にもあるけど旧来の生物にとっては毒にしかならないんで粗方駆逐されちゃった

同時期に酸素呼吸の生物も生まれてるけど、その中の1種は旧来の生物の1種と
共生の道をとった。それが俺らの中にも入ってるミトコンドリア…という事らしい
0135名無しさん@お腹いっぱい。
垢版 |
2017/03/02(木) 17:37:39.04ID:ozHRXqK2
tes
0136名無しさん@お腹いっぱい。
垢版 |
2017/03/02(木) 18:39:01.21ID:TWgzk13v
>>134
それは興味深いね

ミトコンドリア

ほとんどの生物の細胞の中に含まれ、有機物質からエネルギーを取り出す働きをし、独自のDNAをもつ、小器官。
0137名無しさん@お腹いっぱい。
垢版 |
2017/03/02(木) 19:27:13.45ID:vdm9NAOK
ミトコンドリアの形って学校で習ったのは細胞内で
丸長くてぐるぐる往復に線が入ってる
模様の様に成ってるとずーっと思ってたが
あれは細胞をカットした図でミトコンドリアもカット
されていてぐるぐるの線が内臓で本当は長丸い筒の様な
形をしてると最近知ったのを思い出した。
0138名無しさん@お腹いっぱい。
垢版 |
2017/03/02(木) 22:09:43.82ID:j9uo0oqm
ドラゴンボールでセルが太陽系が吹き飛ぶくらいのパワーって言ってるけど分かりやすく言うとどんくらいのパワーが必要なの?
0140名無しさん@お腹いっぱい。
垢版 |
2017/03/02(木) 22:26:29.02ID:C/qHx8cc
>>138
太陽の位置から爆発のエネルギー…簡単に熱という事にしようか
それで海王星までの8つの惑星を全部消し飛ばそうって事なら
多分超新星でも全然足りない
0141名無しさん@お腹いっぱい。
垢版 |
2017/03/02(木) 22:56:33.58ID:VK27F6sU
地球生物は目が2つしか無いと思うんですが、地球では目が2つあれば問題ない環境だからだと思います
しかしながら我々に目が3つ、4つあれば空間が違ったように見えるんじゃないでしょうか?
宇宙生物の中には頭の後ろにも目がある生物がいるそうで、360度見れてないとヤバイ環境だとそうなるそうです
0143名無しさん@お腹いっぱい。
垢版 |
2017/03/02(木) 23:03:30.58ID:TWgzk13v
>>141
トンボは空中での運動能力が圧倒的で、時速100kmの高速飛行や反転(トンボ返り)
ホバリングで空中停止もできる。
トンボは大スズメ蜂でさえ捕食する。

目は、複眼と単眼の2種類が備わっており、単眼は3つあり、
複眼は、数万個の眼の集合体だ。
人間では見えない紫外線波長域で世の中を見れる。
ターミネーターのように生物(有機物)が発光して見えるので獲物を簡単に見つけられる
0144名無しさん@お腹いっぱい。
垢版 |
2017/03/02(木) 23:05:46.38ID:VK27F6sU
地球には居ない生物としてスライムのような生物と植物なのに歩き回る生物などがありますが、そんな生物も宇宙にはいるそうです
0145名無しさん@お腹いっぱい。
垢版 |
2017/03/02(木) 23:10:39.53ID:TWgzk13v
>>141
イルカやコウモリといった哺乳類は、超音波の振動波で世の中を見れる。
電磁波(光)から脳内映像化するのと、音波で脳内映像化するのとでは大して変わらん。

また、ペンギンやアシカは屈折率の高い水中での視力に最適化しているから
地上ではド近眼で眼が悪い。
0147名無しさん@お腹いっぱい。
垢版 |
2017/03/02(木) 23:30:07.53ID:bHIMrCry
>>144
だから、ソース
視覚器官を持つような高等生物が地球外生命で存在するってすごいことなんだが
それとも、オマエの妄想?
0150名無しさん@お腹いっぱい。
垢版 |
2017/03/03(金) 08:40:12.04ID:cYSLgsS6
聖書を読め!
クルアーンを読め!
0153名無しさん@お腹いっぱい。
垢版 |
2017/03/03(金) 09:47:09.43ID:laSWqVnA
このスレヤクいな
0154名無しさん@お腹いっぱい。
垢版 |
2017/03/03(金) 12:05:46.90ID:T8Tc19I9
五億年前のカンブリア紀の生物って宇宙生物っぽいよなー (目が沢山有ったり形状がマシンみたい)
0160名無しさん@お腹いっぱい。
垢版 |
2017/03/03(金) 13:47:55.20ID:qYnyuR6B
トンボはゴキブリより古参なんだよな
原始的昆虫の一つの完成形だ。

鳥類もそうだけど、空を支配する生物ってちょっと古い型が多いな。
空飛ぶ哺乳類なんてコウモリぐらいしかおらんし。

空への適応における進化に時間かかるのかな
0161名無しさん@お腹いっぱい。
垢版 |
2017/03/03(金) 13:52:49.14ID:qYnyuR6B
ちなみに害虫を食べるトンボは易虫とされ、日本では古代から神聖な生き物として扱われてきた。
日本書記や古事記では、日本の本州を指す言葉にトンボが含まれている。
装飾品や和服にトンボが描かれていることもある。

しかし欧州では、ドラゴンフライと呼ばれ、古代から嫌われる虫であった。
0162名無しさん@お腹いっぱい。
垢版 |
2017/03/03(金) 14:03:13.77ID:EbQ6by2H
子供の頃からトンボを見ているから、ホバリングなど当たり前と思ったけど、実はすごい技術で昆虫でも正確にホバリング出来る種は少ないかも。
ハチドリはホバリングに関しては鳥類での最高峰だけどね。
0163名無しさん@お腹いっぱい。
垢版 |
2017/03/03(金) 14:13:29.62ID:bIGaS8Sg
>>160
> 空を支配する生物ってちょっと古い型が多いな。
> ・・・
> 空への適応における進化に時間かかるのかな


空を支配する生物は古い型がほとんどな原因としては、
空への適応に要する進化時間が長いというのもあるだろうが
空への適応に成功するケースが非常に少ないというのも
あるんじゃないかと個人的には想像している

つまりなかなか空への適応に成功する生物が現れないから
ひとたび空に適応できさえすれば古い生物でも空での生存競争で淘汰されず
ずっと生き残っているという考え方だ

実際、風力で空を舞う微生物を除いてマクロな生物にとって地球重力に逆らい空に適応するのことが非常に困難な課題なのは確かだ

特に、蝙蝠やモモンガ・ムササビ(や古の翼竜)のように自力での飛翔を諦め滑空に頼るのならばまだしも
一部の大型のを除くほとんどの鳥や昆虫のように自分の力で揚力を発生させて飛ぶには
身体の軽量化と羽ばたくための筋肉の増強の両立を達成させなければならないからね
その両立は想像するだけでもとても難しいのは以下に述べるように明らかだ

なぜならば、その両立とは次のようにある意味では矛盾した事柄を同時に満たすということだから、つまり
軽量化には身体を支えているフレーム部分(脊椎動物のような内骨格構造ならば骨格であり昆虫のような外骨格ならば殻構造)を
薄く弱くするのが一番簡単だが、そのフレーム部分の軽量化を不注意に行うと飛翔に必要な強い筋力を受け止められなくなる

つまり自力で長時間飛翔できるということはこの両立を非常に高いレベルで達成できた場合だけなのであって
両立できてない限りは(つまり自力飛翔可能な生物への進化の途中では)、
自力で飛べず飛翔生物としての生存性の高さ(天敵から飛んで逃げられること)は
枝から枝へジャンプしかできない普通の動物とあまり違わず、
その一方でフレーム部分の軽量化による傷に対する脆弱化は先に現実の問題になるので
自力飛翔生物への進化途中の段階では、進化しない場合と比べて生存性の面でトータルでマイナスになってしまう可能性が高い
0164名無しさん@お腹いっぱい。
垢版 |
2017/03/03(金) 14:13:47.19ID:qYnyuR6B
>>162
ハチドリは餌となる特定植物との共進化で一気に進化した。
植物は蜜を取られないように奥へ奥へと蜜を隠していき
ハチドリは、ホバリング性能を向上させ、クチバシをどんどん伸ばしていった。

共進化の良い例。このような競争関係がある種は進化スピードが早くなる。
ティラノザウルスの巨大化も共進化とされ、巨大化したのは非常に短期間である。

人間は数百万年の間、他の類人猿と競争していたが
3万年前に一人勝ちして共進化する相手がいなくなった。
そして同種間を競争相手に選んだ。戦争である
0166名無しさん@お腹いっぱい。
垢版 |
2017/03/03(金) 14:26:13.39ID:qYnyuR6B
まぁ宇宙に絡めると、系外惑星に地点生命体がいたとしたら
そこでは1人勝ちだろうから、新たな競争相手(敵)を欲しがるのは当然だろうな。

地球の近隣に地点生命体がいないのはラッキーと思うね
0169名無しさん@お腹いっぱい。
垢版 |
2017/03/03(金) 18:11:18.05ID:J0LWnB49
今月のニュートンの特集によると、昔のテレビの深夜の雑音ノイズには、宇宙背景放射が含まれていたそうだ。
なるほどね。
0170名無しさん@お腹いっぱい。
垢版 |
2017/03/03(金) 18:13:41.97ID:pLhjsUyt
そんな微細なものまでちゃんと拾ってたんか
流星観測にラジオ使うぐらいだから、まあ拾ってても確かにおかしくはないわけだけど
0171名無しさん@お腹いっぱい。
垢版 |
2017/03/03(金) 18:51:45.76ID:9y7P4q/i
>>169
ビッグバンを見る方法はテレビのノイズだって言ってた
何%かはビッグバンの名残だって言ってた
0172名無しさん@お腹いっぱい。
垢版 |
2017/03/03(金) 18:54:32.49ID:9y7P4q/i
都市伝説か事実か知らないけど昔テレビのノイズで朝方4時くらいに何チャンネルか忘れたけど、見てたら女の裸が映るってのがあったな
0173名無しさん@お腹いっぱい。
垢版 |
2017/03/03(金) 18:55:14.93ID:1YWW524Z
>>169
大地震の前に電離層が変化するらしいから、
テレビのノイズも変わるのかな、地震警報器になればいい

アナログテレビでないとだめか
0174名無しさん@お腹いっぱい。
垢版 |
2017/03/03(金) 18:58:18.41ID:baZsznBu
>>170
> 流星観測にラジオ使うぐらいだから、まあ拾ってても確かにおかしくはないわけだけど
FMラジオを使った流星の電波観測で受信するのは、流星によって反射した
地上のラジオ局の電波だから背景放射と比べるのは無理があると思う。
0176名無しさん@お腹いっぱい。
垢版 |
2017/03/03(金) 22:18:34.22ID:herhNtgx
もうそろそろドイツがテレビ初放送した軍事色満載の
プロパガンダ放送を受信した異星文明の奴らが
危機感で攻めて来るかもなー
0181名無しさん@お腹いっぱい。
垢版 |
2017/03/05(日) 19:04:16.83ID:dLsM8ldm
ゲームの世界でもバグで無限ループになったら続行不可能だしな
リアル世界にバグは無いのだろうか
0182名無しさん@お腹いっぱい。
垢版 |
2017/03/05(日) 19:17:46.68ID:kEN+mjbe
>>181
リアル世界(俺らがいるこの宇宙という意味で)にバグはない
あったら130億年以上も保たない

でもマルチバース論によれば、バグがある宇宙も生まれては消え生まれては消えを繰り返しているそうだ
0183名無しさん@お腹いっぱい。
垢版 |
2017/03/05(日) 20:38:56.57ID:Gr6TLIeK
>>181
「リアル世界」の捉えようにもよるけども
例えば腫瘍は遺伝子のバグで分裂が暴走した細胞…だったはず
生物の進化もある意味バグの出かたがうまい事はまっただけとも言える

そういうバグなら細かく見ればそこら中にありそうだけど
おそらく全部物理学の法則に反しない範囲でのバグなはず
法則まで無視したバグはちょっと想像できないな…
0186名無しさん@お腹いっぱい。
垢版 |
2017/03/05(日) 21:51:22.49ID:kQlgMsSq
もし、我々の太陽がケプラー62くらいのサイズのK型恒星と入れ替わったら、地球にからはどのようにみえるのでしょうか?
現在の太陽よりも小さく、光も弱々しくみえるんでしょうか?
0187名無しさん@お腹いっぱい。
垢版 |
2017/03/05(日) 21:54:54.82ID:HgbKwgeV
>>185
暖かい服を着てれば少なくとも生きられるらしいよ
微妙にすごいな
金星の高度50キロ地点とかもなかなかいけそう
火星は磁場がないからテラフォーミングはむりだとおもう

勝手に文章書いてすまん
0188名無しさん@お腹いっぱい。
垢版 |
2017/03/05(日) 22:48:42.28ID:dLsM8ldm
>>187
暖かい服でなくとも、冷たい風による熱伝導がないから断熱層は薄くてもOK。
でも体からの熱放射(赤外線)を遮断しなければならないので服にアルミ層が必要。
で、結局、普通の宇宙服とほとんど変わらん
0192名無しさん@お腹いっぱい。
垢版 |
2017/03/05(日) 22:58:54.36ID:HgbKwgeV
>>188
つまり寒い南極みたいな感じですかね
スキューバー用の空気ボンベがあれば歩き回れるってのは
すごいと思ったんですが
0197名無しさん@お腹いっぱい。
垢版 |
2017/03/06(月) 04:46:35.82ID:CFlUyhJh
>>186
ちょっと考えてみた
K型の星の明るさは太陽と比べてもそんなに暗くないんだけども
ケプラー62の絶対等級は+5.9、太陽が+4.8なんで1.1の差で太陽の36%の明るさ

大きさは7割弱だから見え方としての円の面積で言うと…多分45%くらい
単位面積当たりの光の強さが8割くらいになるから眩しさはあまり変わらないかも

周りの景色の具合については、欠け方の大きい日食を見た事があるなら
(5年前の金環食とか)それを想像してもらえればいいかと

ちなみにM型(赤色矮星)で質量の下限ギリギリだと満月くらいの明るさになる
0198名無しさん@お腹いっぱい。
垢版 |
2017/03/06(月) 06:40:16.92ID:Z93oXDoY
太陽のように自ら光を発する星が、他の恒星の惑星になってるところってあるんでしょうか?
例えばアンタレスのような太陽の何百倍の大きさもある恒星なら
太陽サイズの惑星が存在してもおかしくないですよね?
ネットで調べてみたのですがよくわかりませんでした。
よろしくお願いしますm(_ _)m
0199名無しさん@お腹いっぱい。
垢版 |
2017/03/06(月) 07:04:28.08ID:UVn2KCVa
2重星や3重星は良く有るけど、それじゃあ駄目なのかな?
0200名無しさん@お腹いっぱい。
垢版 |
2017/03/06(月) 08:01:54.54ID:B0MJjAQV
>>198
二つの恒星が互いの周りを回る連星なら分かるけど・・・。
自ら光を発する星は惑星とは言いません。
アンタレスは赤色超巨星で、年老いて膨張した恒星ゆえ、
寿命は数百万年以下。仮に惑星が公転していたとしても
早々に超新星爆発に巻き込まれてオワル。
太陽ほどもある惑星は普通存在しない。
そんなに大きいなら、核融合が始まって恒星になるはず。
0201名無しさん@お腹いっぱい。
垢版 |
2017/03/06(月) 08:16:32.39ID:UbMDjnyf
明るさに差のある二重星といえば死兆星で有名なミザールなんか暗いほうの恒星が惑星みたいな感じで回ってるんじゃないか?
0206名無しさん@お腹いっぱい。
垢版 |
2017/03/06(月) 10:53:51.08ID:7Ws15CFp
太陽と木星(他の惑星も同じですが)は、それぞれの重力によって共通重心を持ち、その周りを回っていることになるそうです。
もちろん、太陽がはるかに巨大なので、共通重心はほぼ太陽のすぐそばにあるようです。
0207名無しさん@お腹いっぱい。
垢版 |
2017/03/06(月) 12:28:45.24ID:hkFJc2Ow
>>205
>>206
なるほど、わかりました!
恒星の周りを他の恒星が惑星のように回っていても
それは惑星ではなく連星になるんですね。
ご回答ありがとうございました。
0208名無しさん@お腹いっぱい。
垢版 |
2017/03/06(月) 12:41:31.04ID:vREMZHP2
ドムだよ!!!
0211名無しさん@お腹いっぱい。
垢版 |
2017/03/06(月) 13:50:17.84ID:dE7S6vgY
>>209
これは2体間の運動だから細かい事を考えなければ楕円軌道で周回するか双曲線軌道で飛んでいくかは最初から決まっている
周回していたのが突然離れていくことはない
0214名無しさん@お腹いっぱい。
垢版 |
2017/03/06(月) 18:13:53.27ID:JV5NCfF8
>>212
この動画は縮尺とか取っ払ったイメージとしても正しくないけどね。
太陽の進行方向は黄道の北極方向じゃないんだよなあ。
0215名無しさん@お腹いっぱい。
垢版 |
2017/03/06(月) 19:16:15.69ID:kKzl89RF
俺もそこが気になった
北か南か知らないが極が進行方向の先頭という必然性は無いと思った
0216名無しさん@お腹いっぱい。
垢版 |
2017/03/06(月) 20:04:50.19ID:0sr/WoNT
太陽の進行方向の定義は?
銀河に対しての進行方向だろうけど、この動画では銀河を基点にした進行方向とは確認できないw
0218名無しさん@お腹いっぱい。
垢版 |
2017/03/06(月) 22:49:50.06ID:Z6y49hwH
>>197
詳細な解説ありがとうございます!
0219名無しさん@お腹いっぱい。
垢版 |
2017/03/07(火) 04:28:40.63ID:9TxGha3i
空間には光が飛び交っておりエネルギー0の場では完全に相殺されているため認識できない
その光のエネルギーは素数のみで構成されたゼータ関数の集合として表される
完全に相殺されている空間を飛び交っている光のエネルギーは非自明なゼロ点の間隔の大きさに等しいため無数に存在する

E=lim[n→∞]Σ1/P(k)^s=1/2^s+1/3^s+1/5^s+・・・+1/P(n)^s ←素数のみで構成されたゼータ関数もゼロ点s=1/2+i*yのとき0になる
E=lim[n→∞]Σcos(y*logP(k))/P(k)^x+i*Σsin(y*logP(k))/P(k)^x=mc^2+i*hν

E=lim[n→∞]Σcos((y+hν)*logP(k))/P(k)^(1/2+mc^2)-i*Σsin((y+hν)*logP(k))/P(k)^(1/2+mc^2)=mc^2+i*hν


m=0 hν≠0
0=Σcos((y+hν)*logP(k))/P(k)^(1/2) ←y+hνは当然非自明なゼロ点である必要があるためhνは非自明なゼロ点の間隔にならなければならない
hν=-Σsin((y+hν)*logP(k))/P(k)^(1/2)   ←y+hνが非自明なゼロ点である時hνが0になってしまうためhν≠0のときm=0とはならないので光には質量がある

冦c^2≒0が光の質量エネルギーとすると
冦c^2=0=Σcos((y+hν)*logP(k))/P(k)^(1/2+冦c^2) ←1/2から僅かにプラスx方向にずれるため実部が出現する
hν=-Σsin((y+hν)*logP(k))/P(k)^(1/2+冦c^2)     ←1/2からずれるためhνが出現する
0220名無しさん@お腹いっぱい。
垢版 |
2017/03/07(火) 04:40:15.65ID:9TxGha3i
E=lim[n→∞]Σcos((y+hν)*logP(k))/P(k)^(1/2+mc^2)-i*Σsin((y+hν)*logP(k))/P(k)^(1/2+mc^2)=mc^2+i*hν 

エネルギー0の空間に存在するエネルギーつまり空間のエネルギー
E=lim[n→∞]Σcos((y+0)*logP(k))/P(k)^(1/2+0)-i*Σsin((y+0)*logP(k))/P(k)^(1/2+0)=0+i*0 ← 質量は0だが光は飛んでいる
yが非自明なゼロ点であれば良いためhν=ゼロ点の間隔の光が飛び交っていてもエネルギーは0になる
E=lim[n→∞]Σcos((y+hν)*logP(k))/P(k)^(1/2+0)-i*Σsin((y+hν)*logP(k))/P(k)^(1/2+0)

(y+hν) , yが非自明なゼロ点であれば上記を満たすため
ゼロ点の間隔=(y+hν)-y =hν

(d/dmc^2)*mc^2=(d/dmc^2)*Σcos((y+hν)*logP(k))/P(k)^(1/2+mc^2)=logP(k)*Σcos((y+hν)*logP(k))/P(k)^(1/2+mc^2)=-(d/dhν)*hν
(d/dhν)*hν=(d/dhν)*-Σsin((y+hν)*logP(k))/P(k)^(1/2+mc^2)=logP(k)*-Σcos((y+hν)*logP(k))/P(k)^(1/2+mc^2)=-(d/dmc^2)*mc^2
E=mc^2+i*hν
E=(d/dmc^2)*mc^2+(d/dhν)*hν=0     ←特定の座標に存在するエネルギーを光と質量に割り振る時左のようになる
空間のエネルギーが光の形態をとろうとすると質量が減少し質量の形態をとろうとすると光が減少する
0221名無しさん@お腹いっぱい。
垢版 |
2017/03/07(火) 04:41:29.53ID:9TxGha3i
>>58
全ての座標にはmc^2+i*hν=1/2+i*yの質量と光がエネルギー0の状態で存在する
m=1/2*1/c^2=εμ/2で光を伝えるのはこの質量の揺れ
yはゼロ点のy軸でhνはゼロ点の間隔の大きさのエネルギーで空間を飛んでいる
質量が何もない空間に移動すると空間が歪んでこの微小な質量が質量が存在する座標により集まり質量を構成する
質量が消えるとより集まったm=εμ/2の質量が周囲に一瞬で散らされ揺れが全方位に伝搬する
0222名無しさん@お腹いっぱい。
垢版 |
2017/03/07(火) 04:51:46.31ID:TieUmT+w
全然成り立ってないじゃん。
なんで素数を光に結びつけるんだよ。
0223名無しさん@お腹いっぱい。
垢版 |
2017/03/07(火) 07:54:54.88ID:r0yw46+q
E=mc^2じゃなくなったらいろいろ破綻するくない?
0224名無しさん@お腹いっぱい。
垢版 |
2017/03/07(火) 08:51:32.99ID:GE9t0X7W
E=mc^2は物体の静止時の話なんで、そもそも質量ゼロの光には当てはまらんのよね
E^2=p^2c^2 + m^2c^4 (pは運動量)がちゃんとした式で、静止状態の場合はp=0になりE^2=m^2c^4なのでE=mc^2になってるわけで
光の場合はm=0だがpは0ではないのでE=pcのエネルギーを持っている
だから光は質量0でもエネルギーを持てるんだな
0225名無しさん@お腹いっぱい。
垢版 |
2017/03/07(火) 08:53:02.17ID:n8zFmod8
S(n)はn番目の素数
S(0)=1
0≦a<b    0≦c   a,b,cは全て整数
[Σ1/S(c)^(x+i*y)]^2-[Σ1/[S(a)*S(b)]^(x+i*y)]=Σ1/n^(x+i*y)=0 ←ゼータ関数が0になるとき

Σ1/n^(mc^2+i*hν)=0
mc^2=1/2 -∞<hν<∞
非自明なゼロ点が原子核のエネルギー間隔に沿うとき
原子核にはとびとびの光しか取り込まれない

Y=| Π*S(c)^((1/2)+i*y)*[Σ1/S(c)^((1/2)+i*y)] |=|Π*S(c)^((1/2)+i*y)√[Σ1/[S(a)*S(b)]^((1/2)+i*y)]|
Y=| Π*S(c)^(mc^2+i*hν)*[Σ1/S(c)^(mc^2+i*hν)] |=|Π*S(c)^(mc^2+i*hν)√[Σ1/[S(a)*S(b)]^(mc^2+i*hν)]|
0226名無しさん@お腹いっぱい。
垢版 |
2017/03/07(火) 09:01:37.03ID:n8zFmod8
エネルギーと質量は相互に変換できるということであって
エネルギーが必ずしも質量を伴うとは限らない。

最終的に宇宙全ての質量が消えてエネルギーだけ残ったとしたら
空間という座標が必要無くなる
0228名無しさん@お腹いっぱい。
垢版 |
2017/03/07(火) 09:13:50.79ID:n8zFmod8
全座標で完全に光が打ち消されている
A*sin(ωt)+(-Asin(ωt))=0 ←Aは定数
光源で位相がずらされると打ち消されなくなり光が現れる
A*sin(ωt+kx)+(-Asin(ωt))=A*(cos(kx)-1)*sin(ωt)+Asin(kx)*cos(ωt)=A*√(2-2*cos(kx))*sin(ωt+X)
A*2*sin(kx/2)*sin(ωt+X)=hν
A*2*sin(kx/2)*sin(2πνt+X)=hν ←位相がxずらされた時左の光が現れる
(A*2*sin(kx/2)*2π)*ν≒hν ←近似値
k=2πν/c
A*(4π*sin(πνx/c))≒h ←プランク定数がxの位相の変化に対して一定の時
x=B*1/ν ←Bは定数
A*(4π*sin(πB/c))≒(2π)^2/c*(AB)≒h
AB=hc/(2π)^2  Bが仮に1のとき
E=hc/(2π)^2*sin(2πνt)のエネルギーが空間を満たしておりかつ全座標で相殺されている
光源を通過した光がx=1/νだけ位相をずらされ相殺されなくなるためhνの光が光源から飛ぶようにみえる
0229名無しさん@お腹いっぱい。
垢版 |
2017/03/07(火) 09:39:01.50ID:I6K5gV79
フィードバック宇宙論 人間原理
https://youtu.be/EapRBY5RNLc
0230名無しさん@お腹いっぱい。
垢版 |
2017/03/07(火) 09:39:30.73ID:HV6L2Yxg
顔文字にしか見えない
0232名無しさん@お腹いっぱい。
垢版 |
2017/03/07(火) 09:59:01.08ID:KtxyyNAG
そもそもn番目の素数の数を求める計算式は存在しないのに、計算式が成立しないだろが。

S(n)はn番目の素数
0233名無しさん@お腹いっぱい。
垢版 |
2017/03/07(火) 10:05:24.46ID:r0yw46+q
>>224
それは静止中の質量mだからじゃん
運動中の質量は静止してる質量と違うから
運動中の質量をmにすれば
E=mccで運動中の式も成り立つ
0236名無しさん@お腹いっぱい。
垢版 |
2017/03/07(火) 10:17:40.03ID:r0yw46+q
>>234
あんたちょっと相対論勉強し直してこい
0237名無しさん@お腹いっぱい。
垢版 |
2017/03/07(火) 10:19:55.47ID:GE9t0X7W
>>236
じゃソース出してよ
まさか見かけの質量増大とか言ってるわけじゃないだろ?
あれは運動量エネルギーテンソルであるpcに起因してるエネルギーなんだから
0238名無しさん@お腹いっぱい。
垢版 |
2017/03/07(火) 10:37:45.45ID:r0yw46+q
>>237
エネルギー運動量テンソルは質量と同価だってアインシュタインさんが言ってた
0241名無しさん@お腹いっぱい。
垢版 |
2017/03/07(火) 13:35:10.89ID:Ie/fkwwy
増えるとしても相対論的質量であって静止質量じゃないわな。
今時相対論的質量なんて表現は使わないそうだけど。
0242名無しさん@お腹いっぱい。
垢版 |
2017/03/07(火) 14:24:16.72ID:wRNYRdb4
相対論的質量なんか光速近くでもニュートンの運動方程式を
成立させるために質量が変わる事にしとこうという浅知恵
0243名無しさん@お腹いっぱい。
垢版 |
2017/03/07(火) 16:50:17.43ID:YOVWzBla
>>228
それ誰かの本に書かれてた?
0244名無しさん@お腹いっぱい。
垢版 |
2017/03/07(火) 17:08:18.03ID:YOVWzBla
>>228
A*sin(ωt+kx)+(-Asin(ωt))=hν
となる理由が分からない。教えて欲しい
0245名無しさん@お腹いっぱい。
垢版 |
2017/03/07(火) 17:12:27.24ID:bnF+bZfT
地熱発電ってあるじゃないですか?
あれって、地球の星としての寿命を縮めるんじゃないかと
心配してるんですがそんなことないですか?
0246名無しさん@お腹いっぱい。
垢版 |
2017/03/07(火) 17:18:20.59ID:R2IYOcaF
使わなければ減らないって訳じゃ無い
火山の噴火や大陸や地殻プレートを動かす力で消耗してる
それに比べれば地熱発電の消費量なんてたかが知れてる
0247名無しさん@お腹いっぱい。
垢版 |
2017/03/07(火) 17:44:02.41ID:nkIWWbf/
>>246
ナルホド。安心しました。
ありがとうございました。
0248名無しさん@お腹いっぱい。
垢版 |
2017/03/07(火) 17:52:09.62ID:HV6L2Yxg
(*'ω'*)+(*'ω'*)+_(:3 」∠)__(┐「ε:)_:(;゙゚'ω゚'):=( ´_ゝ`)
0249名無しさん@お腹いっぱい。
垢版 |
2017/03/07(火) 17:53:37.59ID:HV6L2Yxg
( ´_ゝ`)×( ´_ゝ`)=(´-ω-`)
0250名無しさん@お腹いっぱい。
垢版 |
2017/03/07(火) 18:13:43.46ID:HV6L2Yxg
わかった事があるんですが、人工知能にの進化により日本人や欧米人など裕福な国に住む人々は人工知能に仕事を盗られて、お金で生きてる我々は餓死すると思われる
ですが、アフリカなどの民族はどうでしょう?彼等は未だに狩をして生きています。なので人工知能の進化により死んで行くのは我々、お金社会にいる人間であり、民族達は生き残ると思います
0251名無しさん@お腹いっぱい。
垢版 |
2017/03/07(火) 18:51:04.58ID:n8zFmod8
xに1以上の整数値を入れyに任意の値を入れる
(d/dy)*|Π[k=1→n]S(k)^(x+i*y)*Σ[k=1→n]1/S(k)^(x+i*y)|=0
|Π[k=1→n]S(k)^(x+i*y)*Σ[k=1→n]1/S(k)^(x+i*y)|=∫0dy=S(k) < S(n+1)^2

| Π[k=1→n]S(k)^x*[cos(y*logΠ[k=1→n]S(k))+i*sin(y*logΠ[k=1→n]S(k))]*([Σ[k=1→n]cos(y*logS(k))/S(k)^x]-i*[Σ[k=1→n]sin(y*logS(k))/S(k)^x]) |< S(n+1)^2

cos(y*logΠ[k=1→n]S(k))*[Σ[k=1→n]cos(y*logS(k))/S(k)^x]+sin(y*logΠ[k=1→n]S(k))]*[Σ[k=1→n]sin(y*logS(k))/S(k)^x] }
i*{-cos(y*logΠ[k=1→n]S(k))*[Σ[k=1→n]sin(y*logS(k))/S(k)^x]+sin(y*logΠ[k=1→n]S(k))*[Σ[k=1→n]cos(y*logS(k))/S(k)^x]}

Y = Π[k=1→n]S(k)^x*√{ [Σ[k=1→n]cos(y*logS(k))/S(k)^x]^2+[Σ[k=1→n]sin(y*logS(k))/S(k)^x]^2 } < S(n+1)^2
xに1以上の整数値yに任意の数値をいれYがS(n+1)^2より小さな整数値をとるとき必ず素数になる
0252名無しさん@お腹いっぱい。
垢版 |
2017/03/07(火) 18:54:18.81ID:n8zFmod8
cの速度でi*c^2のエネルギーが空間を伝搬している
空間の特定の座標でこのエネルギーの速度が
a=c*{√(M^2+4)-M}/2まで落ちるとその座標に質量が出現する
M=0つまり質量がない時a=c M→極大の時はa→0に近づいていく
質量内部をもし光が貫通するならこのaの速度で光が質量内部を抜けていく
この質量が運動すると質量内部を抜けていくaの速度も遅れるため
M'c^2=Mc^2/√(1-(v/c)^2) + a*v^2/(c*√(1-(v/c)^2))になる
この余剰項a*v^2/(c*√(1-(v/c)^2))={√(M^2+4)-M}/2*v^2/√(1-(v/c)^2)
Mが大きい時
{√(M^2+4)-M}/2*v^2/√(1-(v/c)^2)≒(1/M)*v^2/√(1-(v/c)^2)のエネルギーが質量Mが運動したときに周囲にばらまくエネルギー

内部時間=(1-(cosθ*v/c)^2)*[√(1-(cosθ*v/c)^2)-i*(cosθ*v/c)]*(1-[(2*GM/(Rc^2))-((v/c)*sinθ)^2])*[√(1-[(2*GM/(Rc^2))-((v/c)*sinθ)^2])-i*√[(2*GM/(Rc^2))-((v/c)*sinθ)^2]]

m'c^2=mc^2/√(1+(v/c)^2-(2*GM/(Rc^2))) + {√(m^2+4)-m}/2*((2*GM/(Rc^2))-(v/c)^2)/√(1+(v/c)^2-(2*GM/(Rc^2)))
質量mが質量MからR離れた座標でvの速度でRの距離を維持しながら鉛直線に対して垂直に運動する時
{√(m^2+4)-m}/2*((2*GM/(Rc^2))-(v/c)^2)/√(1+(v/c)^2-(2*GM/(Rc^2)))のエネルギーを周囲にばらまく
地球上で質量mが速度vで地表に対して平行に運動する時
mの値が小さいほど周囲にばらまくエネルギー量が多い
電子が運動すると電磁波を放射するのはばらまくエネルギー量が多いため認識可能になる
重い質量も微量ながら電磁波をばらまいている
0253名無しさん@お腹いっぱい。
垢版 |
2017/03/07(火) 19:04:30.49ID:yYFiL6zy
>>250
人工知能などの情報革命やブレイクスルーが起きると、途上国の人々の生活を一辺させる。影響は先進国よりも途上国により大きい。

例えば日本では60年以上前から各家庭に電話が普及し、
日本の高度経済成長を実現させたが、ほとんどの途上国では90年代後半まで人々が電話を使わない生活をしていた。
アフリカだけでなく東南アジアや南米や中国でもだ。
未だにそれらの国では家庭電話機がほとんど普及していない。

しかし携帯電話によってアフリカの土人でさえネットが使える時代になった
(貧困地域でも中華製携帯新品500円とか普通にある。FacebookやViberなどSNSは個人情報(プロフ登録)と引き換えに該当アプリのネット無料提供がデフォ)
0254名無しさん@お腹いっぱい。
垢版 |
2017/03/07(火) 19:13:57.18ID:yYFiL6zy
コンピューターによる組合せ最適化問題、巡回セールスマン問題が、人工知能や量子コンピューターで解決されるようになると
世界のあらゆる物事の効率化が劇的にアップする。

まずタクシー料金が激安になる。
待機タクシーが減る。
ロクに稼働しないマイカーを持つよりもタクシーを利用したほうが経済的になってタクシー客が増える。
物流トラックは効率化により半減する。渋滞も減る。
物流費用が下がる。

いつかはマイカー購入という夢を見ていた途上国の人々が、夢を実現することなく社会は変わってしまう
0256名無しさん@お腹いっぱい。
垢版 |
2017/03/07(火) 19:42:11.76ID:Epgm5PQ3
以下の中で、最強なのはどれですか?

全、無、なんでもあり、自由自在、不定、観測者不在、痛む、痛むということに価値を置く、痛みに耐える

痛みに耐えるということに価値を置く、どうでもいい、どうでもいいということに価値を置く、考えない、考えないということに価値を置く

決めない、決めないということに価値を置く、どうなってもいい、どうなってもいいということに価値を置く
0257名無しさん@お腹いっぱい。
垢版 |
2017/03/07(火) 21:49:28.72ID:HV6L2Yxg
>>256
無に勝てる最強はいない
0259名無しさん@お腹いっぱい。
垢版 |
2017/03/07(火) 22:22:32.75ID:HV6L2Yxg
>>258
無様は無である為
貯金が0という概念も無ければ、貯金が1千万あると比べる概念も無いから
0260名無しさん@お腹いっぱい。
垢版 |
2017/03/07(火) 22:30:24.34ID:HV6L2Yxg
無は馬鹿と瓜二つである
正真正銘の馬鹿は悩みを持たない
俺の親族に身体障害者に近い思考能力がほとんどない女の子がいる
彼女は幼稚園から高校まで行き、当然虐められた事もあっただろうが1回も遅刻をしたり、休んだ事は無かったらしい
馬鹿の1つ覚えとはよく行ったもんで、1度インプットされると悩みを持たずに毎朝同じ時間に学校に行くというのが必然になるらしい
0261名無しさん@お腹いっぱい。
垢版 |
2017/03/07(火) 23:54:08.59ID:Epgm5PQ3
>>258-259
いや、>>256に書いてある「無」は、完全なる無のことであって、
そういう、何かと比較した場合の無ではない。
つまり、ありとあらゆる物質、時空間、次元など、とにかくありとあらゆるものがない、完全なる無のことを言ってるのです。
0262名無しさん@お腹いっぱい。
垢版 |
2017/03/08(水) 00:08:42.25ID:i+nJhtf0
>>261
前にも何度か同じことを言う人が現れていて答えは出てる。

物理学で無は存在しない。
無を知りたかったら数学板に行け。
つまり板違い。
0264名無しさん@お腹いっぱい。
垢版 |
2017/03/08(水) 02:57:00.00ID:0UAtWdPh
無は無いのではなく無になるには途方もない訓練が必要
無心になると言いますが、無心になろうと考えていたり頭の中を真っ白にしようと考えている
無は至る所に存在するが無になれない
完全に無になれば右の手刀で狙ってる天津飯の事もわかる
0265名無しさん@お腹いっぱい。
垢版 |
2017/03/08(水) 09:59:41.58ID:c7SZJth9
無がどうのこうのって奴はずいぶん前からいる荒らしだよ。
質問すること自体が目的だから回答するのは無駄だし、奴に必要なのは
哲学じゃなくて、掲示板を荒らしたりしない心の平穏だ。
かまう奴がいるからつけあがる。
スルーすべし。
0267名無しさん@お腹いっぱい。
垢版 |
2017/03/08(水) 11:23:29.80ID:tLUbdCSq
>>264
無は人の頭の中にしか存在しないですよ、まずは概念が何なのかを学びましょう

現実には有る無しで区切れる現象なんて一つもないです
現実や宇宙に区切りはないけど、様々な現象を特徴ごとに名前を付けて概念化し、
人の脳が区切りをつけてるだけです
0271名無しさん@お腹いっぱい。
垢版 |
2017/03/08(水) 11:38:25.98ID:tLUbdCSq
無いが有るということ?概念が何なのかも理解してない人の戯言にしか聞こえませんが

仮にそれが存在するとして、あなたは何が目的でその発言をしてるのかな?
0272名無しさん@お腹いっぱい。
垢版 |
2017/03/08(水) 11:45:42.01ID:3LGGe+PI
聞きたいのは、>>256なんです。
>>256の中で、最強はどれなのでしょうか?
0273名無しさん@お腹いっぱい。
垢版 |
2017/03/08(水) 11:53:07.86ID:tLUbdCSq
なるほど、たしかに自分も子供のころならそういう議論に楽しさを感じてた時期があったかも
横やりで邪魔して失礼しました
0280名無しさん@お腹いっぱい。
垢版 |
2017/03/08(水) 16:56:42.73ID:QxIHglxn
ガリレオ
宇宙は数学という言語で書かれている。
そしてその文字は三角形であり、円であり、その他の幾何学図形である。
これがなかったら、宇宙の言葉は人間にはひとことも理解できない。
これがなかったら、人は暗い迷路をたださまようばかりである。
0281名無しさん@お腹いっぱい。
垢版 |
2017/03/08(水) 16:57:44.15ID:QA1dsjYA
アマゾン奥地のピダハンと言う原住民は言葉が
特殊で未来とか過去とか大きい数の表現が無いそうだ
始めに言葉有りきで未来を特に考えないので悲観する観念も全然無い。
(まぁ彼らは大家族で住んでて男は少しジャングルに入れば豊富に獲物が取れるし
女子も近くで木ノ実や簡単な栽培植物や川で魚もすぐ取れるそうだが)
0282名無しさん@お腹いっぱい。
垢版 |
2017/03/08(水) 17:34:12.06ID:pU+YU8Wg
ブラックホールの特異点とホワイトホールが同じようなもんだと思ってた
ホワイトホールがビッグバンだと
ただブラックホールが吸い込む量が宇宙のビッグバンに匹敵するくらいの
エネルギーをもっているのかと言われるとなんともいえないけど
ホワイトホール=ビッグバンっていうのは個人的に理にかなってると思う
0283名無しさん@お腹いっぱい。
垢版 |
2017/03/08(水) 18:22:31.56ID:3vP4Ju+2
>>278
そのどちらでもないし、哲学とか宗教ってのは中身があるものだ。
無が云々の奴はそもそも中身がない。
いってる本人も、なにが質問の趣旨だかわかってないよ。
メンタルヘルス板が一番適切な案件。
0284名無しさん@お腹いっぱい。
垢版 |
2017/03/08(水) 18:24:56.35ID:3vP4Ju+2
>>282
こういう自分勝手な妄想を書く奴って必ずブラックホールが「吸い込む」って
表現するよな。
ブラックホールに落ち込むのと、手から離したボールが地面に落ちるのは
本質的に何も変わらん。
「吸い込む」と認識しちゃうと、いろいろ妄想が生まれるのかな。
0286名無しさん@お腹いっぱい。
垢版 |
2017/03/08(水) 18:59:18.11ID:i+nJhtf0
>>244
>>255
>>222
空間にE=1/2のエネルギーを均等に分布させる
特定の空間座標に空間に歪みが生じると分布した1/2のエネルギーが歪みに収束する
特定の空間座標のエネルギー量は歪んでいない座標より当然増える
1/2のエネルギーが分布した空間から特定の空間座標をみる時質量または光が生じているとみなせる
A座標からB座標まで質量が移動する時AからBまで歪みが移動する
A座標で質量を構成していた空間に分布するエネルギーは周囲にばらまかれ
B座標に移動してきた歪みにより周囲から収束させられたエネルギーがB座標で質量を構成する
この際A座標から周囲の空間に歪みが光速で伝搬するためこれが重力波になる
A座標からばらまかれた重力波は周囲の空間に分布したエネルギーを当然A座標に引き寄せるが
隣接したA座標とB座標はほぼ同一とみなせるため重力波により引き寄せられたエネルギーはA座標にはいかずB座標に飲み込まれB座標に移動した質量を構成する

Aをダークエネルギーとする時
(d/dE)*E=(d/dmc^2)*mc^2+(d/dhν)*hν+(d/dA)*A=0 のようになる
E=1/2
E=[(d/dmc^2)*mc^2+(d/dhν)*hν+(d/dA)*A]*dE=1/2
(d/dmc^2)*mc^2+(d/dhν)*hν=0
E=[(d/dA)*A]*dE=1/2
A=Eのとき
E=A=1/2になる
0287名無しさん@お腹いっぱい。
垢版 |
2017/03/08(水) 19:01:44.44ID:JmDmXkow
ホワイトホールはイベントホライゾン近傍のホーキング輻射のことだよ。
ホーキング輻射は落ち込めば落ち込むほど強くなる。
イベントホライゾンに近づくにつれて蒸発が激しくなるから外にいる物質がイベントホライゾンを超えることはない。
0288名無しさん@お腹いっぱい。
垢版 |
2017/03/08(水) 19:01:58.77ID:VVl586II
エネルギーは全て波の掛け合わせで表される
空間のエネルギーはすべてゼータ関数で表される
E=Σ1/n^((1/2+mc^2)+i*(hf+hν))=Σcos[(hf+hν)*logk]/k^(1/2+mc^2) - i*Σsin[(hf+hν)*logk]/k^(1/2+mc^2)=mc^2 + i*hν

質量も光もない空間には
E=Σ1/n^((1/2+0)+i*(hf+0))=0をみたすhfの光が流れている
hfはゼロ点のy座標になる
hf(n)=n番目のゼロ点のy座標とする

hν=hf(n+1)-hf(n)
0=-Σsin[(hf+hν)*logk]/k^(1/2+mc^2)  ←hνのエネルギーの光を照射したにも関わらず光が相殺されて光が認識できなくなる
原子核エネルギー間隔がゼロ点の間隔に等しいのはこのため
ただしエネルギー0の空間に飛ぶhf(n)の光は質量に近いほど小さいため (hf(n)の光が質量に置き換えられるため
原子核エネルギーなど微小な領域からずれるとhf(n)つまりnの値が大きくなりゼロ点の間隔を求めるのが難しくなる  
0289名無しさん@お腹いっぱい。
垢版 |
2017/03/08(水) 19:02:14.97ID:TqZJllnt
>>266
そもそも光源から光子が発射される訳で、位相のズレなんて関係ないように思える。
0290名無しさん@お腹いっぱい。
垢版 |
2017/03/08(水) 19:07:21.85ID:TqZJllnt
光の波を数学的に位相のズレとして表すことは出来る?のか
0292名無しさん@お腹いっぱい。
垢版 |
2017/03/08(水) 19:19:14.23ID:JmDmXkow
有限の時間内にはブラックホールは形成されない。
ブラックホールとは表面の脱出速度が光速の天体のことだが、ブラックホール内と外の二つの物体の相対速度が光速に達するには外の物体から見て無限大の時間がかかる。
0293名無しさん@お腹いっぱい。
垢版 |
2017/03/08(水) 20:16:00.56ID:Uvp+IBd7
>>291
否定するのって命題によってはすごい難しいよ?
少なくとも科学の分野においては肯定でも否定でも証明しないといけないから
「肯定も否定も出来ない」なんて事になる場合も多いし
0294名無しさん@お腹いっぱい。
垢版 |
2017/03/08(水) 20:16:35.63ID:3vP4Ju+2
>>291
>だったら自分の考えを言えば?

馬鹿だから

>ブラックホールに落ち込むのと、手から離したボールが地面に落ちるのは
>本質的に何も変わらん。

って書いてあるのが読めないんだろうな、頭が悪いと大変ね。
とにかくこのスレは妄想書くスレじゃないんだから「ぼくのかんがえたうちゅうろん」を
書くこと自体が間違ってるよ。
0295名無しさん@お腹いっぱい。
垢版 |
2017/03/08(水) 21:22:57.75ID:ANS2l8ao
そもそもビッグバン理論ってどうなの?
インフレが起きて時間と空間が生まれたことになってるけど
インフレ中とかインフレ前は時間はなかったの?
それって俺らが近くできるの?
0296名無しさん@お腹いっぱい。
垢版 |
2017/03/08(水) 21:24:19.42ID:TqZJllnt
>>254
少なくとも先進国がそうなる時より後のことであるのは間違いないな
0297名無しさん@お腹いっぱい。
垢版 |
2017/03/08(水) 21:31:53.56ID:TqZJllnt
ホーキング輻射が発表されてから、ブラックホールとホワイトホールは別のもだとするような考えてるえ方は無くなってるよ。
0298名無しさん@お腹いっぱい。
垢版 |
2017/03/08(水) 21:34:40.20ID:JmDmXkow
>>295
そもそも空間の正確の定義がない。
空間は世界と言い換えることもできる曖昧な表現。
定義もできない言葉を使ってる時点で察するべき。
論考の対象となるのは定義できるもので表現されたもの、つまり物理法則だけだ。
ビックバン理論は単なる想像の産物、妄想と同類。
0300名無しさん@お腹いっぱい。
垢版 |
2017/03/08(水) 22:43:50.82ID:UZvN7XUH
宇宙の生成時を計算すると、数学では破綻するらしい。
そこで、哲学的思考によって、ビッグバンという「特異点」を作り出し、計算不能なところから宇宙が誕生した、と説明したのがビッグバン理論。
現在、その最初の地点を含めて、更にその以前にまで遡った宇宙論を数学的に解明しようという動きが出てきている。

サイクリック宇宙論とか、膜宇宙論とか。
いずれも、数式的根拠は示してはいる。
しかし、正しいかどうかはいまだに論争が続いている。
0301名無しさん@お腹いっぱい。
垢版 |
2017/03/08(水) 22:48:37.04ID:QY5r9YLi
宇宙は加速度的に広がり続けてるんじゃなくて無に向かって加速度的に落ち続けていて
その結果宇宙空間が広がり続けてる、っていう可能性はないのかな
こう考えたらダークエネルギーなんて必要なくならない?
0302名無しさん@お腹いっぱい。
垢版 |
2017/03/08(水) 23:15:28.37ID:6dGEW1Gu
>>292
ブラックホールに他の物体が取り込まれる様はポテンシャルの傾きに注目してイメージするとわかりやすいと思う
BHに物体が近づくと二つの間のポテンシャルの傾きは緩くなる つまりBHを囲んでいた地平面が凹む事になる それとは逆に物体のBHと反対側の傾きは急になるので新しく地平面に囲まれた領域が出現する
さらに物体がBHに近づくと離れていた二つの地平面に囲まれた領域は接続してしまう
つまり物体は有限の時間でBHに取り込まれることが可能である
0303名無しさん@お腹いっぱい。
垢版 |
2017/03/08(水) 23:54:12.20ID:b8mx8aXv
ブラックホールに落ちたらとか、吸い込まれたらとかそういう表現がブラックホールは穴だと思わせる表現になる
かくゆう私もブラックホールは穴だと思っていたw
実際は球体であり、360度何処からでも引っ張る力がある
つまり、吸い込まれたら、落ちたら、地面のようなものにぶつかる
という見解であってますか?
0307名無しさん@お腹いっぱい。
垢版 |
2017/03/09(木) 04:12:36.99ID:NzrWak4y
>>303
外部から見ると、ブラックホールの周囲の物質は、BHの強力な重力に引き付けられ、潮汐力によってバラバラに破壊され、BHの回りをグルグル回りながらやがてBHに吸い込まれていくように見える。
またその一部は、BHの極部からジェット化して噴出する。

一方、BHに落ち込む側の物体から見ると、重力によって時空間が歪められ、時間が引き延ばされて、永遠にBHに向かって落ち込んでいくように見える、と言われている。
0309名無しさん@お腹いっぱい。
垢版 |
2017/03/09(木) 07:18:20.58ID:QInJmvUh
>>298
ググったけど面白いな
放射なんたらの発見でビッグバン説はほぼありなんでしょ?
偽の真空とかひらいてる宇宙とかまだ納得できないけど
0310名無しさん@お腹いっぱい。
垢版 |
2017/03/09(木) 07:44:49.62ID:xTwshWVe
>>309
物体の運動を記述できるのは物体が運動するための空間があるから。
その空間が生じるとか言ってる時点で物理じゃない。
宇宙の膨張?というのもデタラメ。
空間が膨張しているからとされてるが、これは簡単に否定できる。
観測できるのは単純に天体が遠ざかってるだけのことで、空間がどうこうは関係なく斥力としか言えない。
たぶん学者同士の間では斥力について言及しない暗黙の了解でもあるんだろ。
斥力があるということになったら物理法則は美しくもなんともなく物理定数は天体が存在するためにとことん都合のいい値になってるってバレるからだろ。
たぶん宇宙は解明されない。そしてもう一つ解明されないことがあってそれが人の意識にかかわるクオリア。
オッカムのカミソリというのがあって物理定数がデタラメなほど宇宙が存在するために都合のいい値になってると考えるよりも、はっきり言って宇宙は意識が作り出してると考える方が合理的だな。
0312名無しさん@お腹いっぱい。
垢版 |
2017/03/09(木) 12:29:02.87ID:Ebo3cWNU
>>303
ブラックホールに落ちても何にもぶつからないよ
落ちてく方向に無限に引き延ばされて引きちぎられるだけ
1ミリ先に追いつく事すらない
0314名無しさん@お腹いっぱい。
垢版 |
2017/03/09(木) 15:35:07.20ID:fufBCO/U
>>312
中心に到達した時はどう表現したらいいだろうな?
ホーキングパラドックスへの回答として中心は数学的な点ではなく
素粒子レベルの領域に「11次元の膜」が折りたたまれた状態で存在して
吸い込まれた物質はその表面を動いてるという事になってるらしいけど…
それだと「ぶつかる」というより「取り込まれる」の方が合ってるような気がしなくもない
0315名無しさん@お腹いっぱい。
垢版 |
2017/03/09(木) 16:13:47.43ID:3KRlqhtQ
ブラックホール内部は物質を作る核力(強い力弱い力)が重力によって打ち消されているから既に物質という形態では無いのでは?
つまり物質が存在せず、情報だけが素粒子によって維持されている。
物質に質量を与えるヒッグス粒子も、消失した物質から乖離して地平面に膜を作ってるのではないでしょうか
0316名無しさん@お腹いっぱい。
垢版 |
2017/03/09(木) 16:28:52.57ID:xTwshWVe
イベントホライゾンを超えられると思ってる人がいるのか。
ならホーキング輻射はどうなる?
輻射が離れた場所とイベントホライゾン近傍で同じだとでも言うつもり?
0317名無しさん@お腹いっぱい。
垢版 |
2017/03/09(木) 16:32:19.23ID:xTwshWVe
ちょっと整理しようか。
イベントホライゾン近傍での時間経過はそこから離れた場所とどれくらい違う?
イベントホライゾンに到達する瞬間の時間経過はそこから離れた場所とどう違う?
到達する前に輻射で質量失うだろ?
0318名無しさん@お腹いっぱい。
垢版 |
2017/03/09(木) 17:31:42.83ID:fufBCO/U
ブラックホールに向かって自由落下してる観測点から見て
事象の地平面はどのように扱うべきなのか、という事にもなるのかな
潮汐力云々については考えないという事で
0319名無しさん@お腹いっぱい。
垢版 |
2017/03/09(木) 18:24:22.53ID:Vcv+8i6o
専門的なことは知らんけど啓蒙書だと事象の地平面は落ち込みつつある
立場から見れば普通に通過できると書いてるね。

事象の地平面付近の潮汐力も超大質量のブラックホールなら小さいので
物質が成り立つどころか人間が生きてる事も可能だし。
0320名無しさん@お腹いっぱい。
垢版 |
2017/03/09(木) 18:31:34.27ID:xTwshWVe
自由落下視点とロケットで脱出可能な周回軌道の視点の二つで考えてみたら。
まず明らかなのは周回軌道の視点で見たものと自由落下の視点で見たものが食い違うことはないということ。
見る対象はイベントホライゾンからの輻射。
周回軌道の視点では重力による時間の遅れの影響は相対的に少ないからホーキング輻射も相対的に少ない。
落下軌道の視点では重力による時間の遅れの影響は相対的に多いからホーキング輻射は相対的に多い。
両視点にとっての時間経過は違うのだから、これにはなんの矛盾もない、時間経過が違うなら単位時間当たりの輻射エネルギーも違って当然。
重力による時間の遅れの影響のない場所があることを仮定して、そこの視点でブラックホールが蒸発するまでの時間を10の100乗年とする。
周回軌道ではブラックホールが蒸発するまでにかかる時間はそれよりも幾分か短い。
落下軌道にある視点ではブラックホールが蒸発するまでにかかる時間は、実質的にイベントホライゾンに到達するまでの時間よりも早くなる。
0321名無しさん@お腹いっぱい。
垢版 |
2017/03/09(木) 18:35:12.19ID:xTwshWVe
>>319
まず、落下軌道にある視点と落下起動にない視点で見るものが食い違うことはないことを考えよう。
ホーキング輻射がないのであれば落下視点では普通にイベントホライゾンを通過する。
だがそれは同時に落下起動にない視点で無限大の時間が経過することを意味する。
無限大の時間ブラックホールが存在する保証はないどころか素粒子の寿命すら超えてるんじゃないか、たぶん分からないだろ。
0322名無しさん@お腹いっぱい。
垢版 |
2017/03/09(木) 19:43:56.25ID:0dMAMoPY
ブラックホールは天体ですが、その大きさはビー玉くらいである
よって、そこに落ちるとかぶつかるでは無く、ブラックホールに吸い込まれていくと粒子よりも小さな物質となってしまう
という見解であってますか?
0324名無しさん@お腹いっぱい。
垢版 |
2017/03/09(木) 20:23:05.85ID:xTwshWVe
もうね、幼稚園児みたいな好奇心はいいから、有限の時間内にイベントホライゾンが形成されるのかを考えよう。
それは、ない。
それでもイベントホライゾンがあるなら、ビックバン理論が言うような空間と時間とかが始まるりよりも前にイベントホライゾンが出来たということだよ。
ブラックホール、イベントホライゾンは有限の時間内には形成されない。
反論するならまず相対論を論破してからにしてくれ。
粒子加速器で物質を光速に加速するには無限大の時間か無限大のエネルギーが必要になるのよ、そこには疑いの余地はないね、でないならタイムパラドックスが生じるという単純な論理によって導かれる、難しい数式は必要ない。
0325名無しさん@お腹いっぱい。
垢版 |
2017/03/09(木) 21:15:45.00ID:zKbfBvYR
hf(n)=n番目のゼロ点のy座標
hf(1)≒14
hf(2)≒21
hf(3)≒25
A=ダークエネルギー
(d/dE)*E=(d/dmc^2)*mc^2+(d/dhν)*hν+(d/dA)*A=0
E=1/2+hf(n)
E=[(d/dmc^2)*mc^2+(d/dhν)*hν+(d/dA)*A]*dE=1/2+hf(n)
(d/dmc^2)*mc^2+(d/dhν)*hν=0
E=[(d/dA)*A]*dE=1/2+hf(n)
A=Eのとき
E=A=1/2+hf(n)になる
質量に近いほどAは減少するがこのときAはとびとびに減少する
1/2+hf(n)→1/2+hf(n-1)→1/2+hf(n-2)→・・・→1/2+hf(1)=質量  ←ゼロ点の間隔に従って減少する
原子核エネルギー=1/2+hf(m)のとき
原子核エネルギー+ダークエネルギー=一定の時
1/2+hf(m)+1/2+hf(n)=一定
hf(m+A)+hf(n-B)=一定
原子核エネルギーのとる値もとびとびになる
ダークエネルギーが質量と光にわりふられダークエネルギーは減少するため
質量エネルギー+光エネルギー+ダークエネルギー=一定
0326名無しさん@お腹いっぱい。
垢版 |
2017/03/09(木) 21:33:05.83ID:zKbfBvYR
光は移動すると磁界を空間にばら撒き
質量が移動すると重力波を空間にばら撒く
磁石内部でs極に行くほど光エネルギーの配分がでかくなりn極に行くほど質量エネルギーの配分がでかい
磁石内部でs極からn極に電子が移動すると電子は空間に磁界をばらまき
n極からs極に電子が移動すると空間に重力波をばらまく
s極からn極に移動するときもn極からs極に移動するときも磁界と重力波はばらまいているが
エネルギーの配分に偏りがあるためs極から磁界がばらまかれるようにみえる
0327名無しさん@お腹いっぱい。
垢版 |
2017/03/09(木) 22:34:37.63ID:QInJmvUh
難しい話してんなあ
低レベルで申し訳ないけど宇宙は仮想空間で超新星爆発座標の物理演算が処理落ちして時空が歪んでるとか
プログラムのミスで時空自体がバグって破損してるって可能性はないの?
関係あるかわからんけどホログラムの物理演算は難しいらしいし
0329名無しさん@お腹いっぱい。
垢版 |
2017/03/10(金) 01:59:37.41ID:HHnyHPXV
重力と慣性力が何故か同じという等価原理

物体が移動すると重力波を放出するからなのか。
電車のブレーキで乗っている人が倒れるのも、電車が重力波を放出してからってことかな。
慣性力の正体は重力波
0330名無しさん@お腹いっぱい。
垢版 |
2017/03/10(金) 02:17:07.30ID:HHnyHPXV
超弦理論ってほとんど確定じゃね?これ理に叶いすぎて面白いな

空間は全て弦で満たされてるってことでしょ。
この弦はいろんな形に変わることができて振動もする。

ここから重力子や光子など素粒子(エネルギー)が現れる。伝える。
0331名無しさん@お腹いっぱい。
垢版 |
2017/03/10(金) 02:23:39.36ID:HHnyHPXV
人間の観測ってほとんど電磁波だけだけど、電磁波って素粒子であり波だよね。

だったら他の素粒子も観測できるはずだよね。
重力波は間接的な方法で検出できたけど、直接的に重力子を観測できないものかね。

あ、でも電磁波を観測できても、波になってない光子そのものは観測できてないのか。
宇宙から届く電磁波なく光子そのものを観測した宇宙を見たい

重力も、波なら観測できたけど重力子という素粒子では観測ができないのかな。
0332名無しさん@お腹いっぱい。
垢版 |
2017/03/10(金) 02:28:42.70ID:HHnyHPXV
映像化できるのは、波だけなのかな?

X線で見た宇宙
可視光で見た宇宙
マイクロ波で見た宇宙
重力波で見た宇宙
ここまで波

光子で見る宇宙
重力子で見る宇宙
ニュートリノで見る宇宙
ヒッグス粒子で見る宇宙
0334名無しさん@お腹いっぱい。
垢版 |
2017/03/10(金) 07:54:53.06ID:3sdMgPzk
セルンが2013年からマイクロブラックホールで重力子を見つけようとして未だに成果なし
本当に存在するかどうかも実際怪しい
数式上美しくまとまっているが、物理とはかけ離れてしまった
超弦(M)理論は一部でそう言われてるくらいだし
ブラックホールの特異点を証明したペンローズの主張では
量子の本質的な部分は計算不可能であるはずだと言うくらいだし
数式上綺麗まとまっててもまだ過信するには早すぎる
0335名無しさん@お腹いっぱい。
垢版 |
2017/03/10(金) 07:58:32.14ID:l2zc79wd
事象の地平面は有限の時間内には形成されない。
ビックバン以前からあったものでない限り、宇宙にブラックホールは存在しない。
だが超高密度の大質量はあるんだろ。
超新星爆発によって重力崩壊が起こるのも事実だろ。
だがそれでも有限の時間内には事象の地平面は現れない。
実時間内ではどれだけ時間が経っても重力崩壊途中なのだ。
ブラックホールがあるだの空間が膨張するだのと、どれだけデタラメがまかり通ってるんだろ?
0336名無しさん@お腹いっぱい。
垢版 |
2017/03/10(金) 08:33:55.30ID:l2zc79wd
ブラックホールがあるとか言ってたらレベルが低いまま何も進歩がない。
この際だから学者とかSF雑誌の言ってることは無視して自分で考えよう。
重力加速度が光速を超える場所、つまり時間の止まる場所がなぜ有限の時間内に現れる?
0340名無しさん@お腹いっぱい。
垢版 |
2017/03/10(金) 09:07:04.14ID:7Cq1jPut
>>322
サイズはまちまち
超ミニのマイクロブラックホールから
銀河の中心にあるという超巨大ブラックホールまで、様々らしい
0341名無しさん@お腹いっぱい。
垢版 |
2017/03/10(金) 09:11:59.83ID:7Cq1jPut
>>335
ブラックホールが形成されれば、イベントホライゾン事象の地平面は当然ながら存在する。
イベントホライゾンが形成される、という考えが間違い。
そこに存在する、と言うのが正しい。

三角形に頂点が3つ存在するのと同じ。
0344名無しさん@お腹いっぱい。
垢版 |
2017/03/10(金) 10:11:06.12ID:dutrkwmz
>>336
現在ブラックホールは直接観測では発見できてないけど、ブラックホールの活動の痕跡とか
間接的な観測結果はいくらでも見つかってるのね。
ブラックホールが存在しないとなると、それらの観測結果を一通り説明できる理論が必要に
なってくるわけで、単純に無かったことにしようってわけにはいかない。

ってか、普通は「事象の地平面は有限の時間内には形成されないような気がするけど、実際に
ブラックホールはあるという間接的証拠はたくさんあるから、その考え方がどこか間違えてる
んだな」って考えるんだよ。
0345名無しさん@お腹いっぱい。
垢版 |
2017/03/10(金) 10:30:47.58ID:l2zc79wd
>>344
だからビックバンより前からあったBHについては否定してないだろ。
それに重力崩壊途中で事象の地平面が現れてなくても、質量の分の重力作用は及ぼす。
重力崩壊途中の質量と重力崩壊が終了した質量で、そこから十分に離れた場所に及ぼす作用は質量が同じならほぼ同じ。

例えば、太陽質量の星雲があるとする、そこの重力中心から何光年も離れたところなら、そこに太陽があろうと太陽質量の星雲があろうと重力的には殆ど同じなんだよ。
それが太陽質量の星雲の中に入るほど近距離になると質量が拡散してる分だけ重力源が拡散してる。
星雲ほどの大きさでなくて恒星の大きさなら、その拡散はもっと小さい。
BHと恒星でも同じ、遠く離れた場合と近距離では重力による影響は違う。
だからBH質量がそこにあるなら、BHになっていようといまいと遠くの場所に及ぼす作用は同じ。
0346名無しさん@お腹いっぱい。
垢版 |
2017/03/10(金) 10:41:08.61ID:dutrkwmz
>>345
その話も宇宙背景放射の観測結果から否定される。
宇宙背景放射の観測から、宇宙の晴れ上がりの時点で宇宙はごくわずかに
ムラがある程度で、ほとんど均質な状態だったことが証明されてる。
ビッグバンより前にブラックホールが存在するような物質・エネルギーの
偏りがあれば、宇宙背景放射はまったく違った観測結果になる。

っていうか、ビッグバン以前のブラックホールって、自分が何を言ってるか
理解できてる?
0348名無しさん@お腹いっぱい。
垢版 |
2017/03/10(金) 11:03:26.13ID:h8W9EaFe
>>329
運動する質量が重力波をばらまくのではなく
運動する質量が通過した座標が重力波をばらまく
1枚の大きな布をイメージして布の上にエネルギーを分布させる
布をA点でつまみ上げるとエネルギーが一点に集まる
これが質量になる
質量が移動する際つまみ上げる点を移動させることになる
この時A点でつまみ上げた布を一旦離して移動したB点で布をつまみ上げる
このとき一度エネルギーが全方位に移動した後B点にエネルギーがまた集まる
A点から重力波がばらまかれ重力波がエネルギーを吸い寄せB点にエネルギーが収束し質量になる
0349名無しさん@お腹いっぱい。
垢版 |
2017/03/10(金) 11:07:41.74ID:h8W9EaFe
>>329
A点からB点に質量が移動する時
A点から重力波が空間に伝搬する
空間から等量の重力波をB点は吸収する(吸収しなければB座標に質量が存在できない
宇宙空間に対生成された2つの質量のみがある時
片方の質量のみが運動する場合ばらまかれた重力波をばらまいた質量が全て吸収しなければならないことになるが
これは運動する質量の通過した座標が重力波をばらまかないことと同義になるため不可能
片方の質量が動いた際もう片方の質量も同様の速度で運動するとき
2つの質量をどれほど離しても矛盾しない
量子テレポーテンションで対になった粒子の状態が常に同一なのは対になった粒子間を等量の重力波が行き来しているため
量子もつれは受信した重力波により規定され量子もつれに影響を与える重力波は量子もつれによってのみ生成される
ラジオのように特定の周波数の重力波しか粒子の量子もつれに影響を与えられないため対になった粒子の量子もつれが起こす重力波の影響のみを考慮すればいい
0350名無しさん@お腹いっぱい。
垢版 |
2017/03/10(金) 11:52:58.65ID:l2zc79wd
>>346
ビッグバン以前を語ることは空間の外側を語るに等しい。
それは物理じゃない、物理で空間を取り扱うことはできないから。
なら別宇宙があるとか言う話も物理じゃない。
別の宇宙があると物理的にいうことができないなら、この宇宙の物理定数が天体が存在するために都合のいい値になっている理由についても、別の無数の宇宙の中で偶然この宇宙がそのような物理定数となっているとは語れない、物理ではそれについてはなにも語れない。
ここまでOK?
物理定数、空間に言及するなら、その時点でもう物理じゃない。
なら、はっきり言って意識が宇宙を作り出してるという主張と同じ次元の話。
違うのは物質は観測することができるがクオリアは観測する方法がないというだけ。
意識、クオリアは確実に存在するが、おそらく物理量を持たない、だから物理として取り扱えない。
空間も意識も同じじゃないか、なら、物理定数の値をぐちゃぐちゃに組み合わせた結果の宇宙と、意識が作り出した結果の宇宙という話は、どっちかシンプルか。
オッカムのカミソリにあてはめて考えたら分かる。
0351名無しさん@お腹いっぱい。
垢版 |
2017/03/10(金) 11:58:05.74ID:l2zc79wd
>>346
背景放射は宇宙が限りなく平坦であることを示しているらしいな。
ただ個人的にはクエーサーの見かけの大きさから、遠いところでは宇宙の曲率は正になってるんじゃないかと想像してる。
つまり宇宙は閉じてる。
これは地球は球になっているが、生活の支障がない程度には地面は平坦であるのと似ている。
地球は球でなければ自明な形じゃなく、地球が存在するための物理法則はシンブルなものとはならない。
地球が存在するための方程式を最もシンプルに書くとするなら、地球は当然のことながら球になるんです。
宇宙が存在するための方程式の場合も同じでしょう、ただしそれは物理ではないことは分かってます。
0354名無しさん@お腹いっぱい。
垢版 |
2017/03/10(金) 12:11:21.28ID:l2zc79wd
>>352
それは「宇宙の外」とか言ってる人にも言ってくれ。
>>353
外からみると止まっているように見えるは、中から見たら外は無限大の時間が経過してるのと同義だぞ。
0355名無しさん@お腹いっぱい。
垢版 |
2017/03/10(金) 12:19:24.40ID:3sdMgPzk
自分で考えるのはいいが、
相対論と各種量子力学を全て理解した上で考えないと
それこそただの宗教だよね
0356名無しさん@お腹いっぱい。
垢版 |
2017/03/10(金) 12:31:51.99ID:l2zc79wd
自分で考えないから脱出速度が光速となる領域は有限の時間内に現れないということに気づかないんだろ。
ブラックホールとか事象の地平面なんてただの妄想じゃん。
0357名無しさん@お腹いっぱい。
垢版 |
2017/03/10(金) 12:52:41.83ID:3sdMgPzk
おまえが全て理解した上でそれを言ってるんならまだしも
単語を聞いたことある程度の知識でそんなこと言われても鼻で笑うしかないんだが
ハンッ
0358名無しさん@お腹いっぱい。
垢版 |
2017/03/10(金) 12:56:36.00ID:7Cq1jPut
宗教にはまる人は、自分の理解出来ない事柄に強く曳かれる。
宗主は、カリスマ性の他に、神秘性が必要で、その神秘は理解不能であるほど良い。
0360名無しさん@お腹いっぱい。
垢版 |
2017/03/10(金) 13:22:28.76ID:l2zc79wd
まあしかしあれだな、一時期取り上げられた第三セクターによる公共工事。
地方に行けば巨大な博物館とかたくさんある。
で、そういうところで教授なりそこないが就職してたりするわけで、ある意味アカデミズムのセーフティネットになってる側面もある。
はっきり言って物理的に起こることのみを発表してたら、たしかに夢はないかもしれないし、書籍も売れないかもしれない。
書籍が売れない講演も出来ないとなったら、アカデミズム全体のパイが縮小するかもしれないし、そうなると物理を志しても割に合わないとなるかもしれない。
だから話題を作って書籍を販売するなりして収入を確保する方法があるのはいいんだが、脱出速度が光速となるような領域が有限時間内に生じるみたいな明らかにおかしい話を一般人には式が分からないのをいいことに押し付けるのもどうかと思うね。
0362名無しさん@お腹いっぱい。
垢版 |
2017/03/10(金) 14:06:15.00ID:l2zc79wd
学者もたいがいだろw別宇宙とかw
星の生涯の現実は重力崩壊途中で時間が凍り付いて、周囲に重力による影響を及ぼす以外のま一切の物理現象が起こらなくなる。
こんな感じだろ。
0363名無しさん@お腹いっぱい。
垢版 |
2017/03/10(金) 14:19:01.17ID:DBtt3irN
アンドロメダ銀河までかたつむりが歩いたら何年ぐらいかかりますか
0365名無しさん@お腹いっぱい。
垢版 |
2017/03/10(金) 14:40:02.36ID:odSRB32z
>>319
BHの外部の遠方にいる観測者からは、シュバァルトシルト座標系で認識されます。
ここでは「シュバァルトシルト半径=特異点」となり、BHに落ち込んでいく宇宙船は最終的には
シュバァルトシルト半径で停止したように見えます。(実際には宇宙船からの光も段々赤方偏移してしてしまい、
可視領域ではなくなってしまうので、見えなくなってしまうでしょうが。。)
一方、宇宙船の乗組員はエディントン・フィンケルシュタイン座標系(それを引き直したもの)で認識されますので、
「シュバァルトシルト半径≠特異点」となり、有限時間でそれを通り抜けてしまいます。
http://teenaka.at.webry.info/200702/article_10.html
0367名無しさん@お腹いっぱい。
垢版 |
2017/03/10(金) 15:07:54.90ID:etbUs2lQ
>>366
宇宙に浮かぶと何でもカッコよく見えるんだよ
0368名無しさん@お腹いっぱい。
垢版 |
2017/03/10(金) 15:18:10.34ID:7Cq1jPut
そういえば、BHは完全な黒体ではあるが、シュバァルトシルト境界面の外側ではBHに落ち込むのを逃れて飛び出す光やX線があって、BHの輪郭が光って見えるらしい。
0369名無しさん@お腹いっぱい。
垢版 |
2017/03/10(金) 15:22:16.58ID:AupFOEy9
>>363
アンドロメダ銀河までの距離が変化しない物として、
かたつむりの移動速度で飛んだらどれだけかかるかを考えてみる

かたつむりの移動速度は一番速くて2.7mm/sだそうで、
計算しやすく3mm/sとすると光速の千億分の1となるから、
アンドロメダ銀河までの距離(254万光年)を千億倍すると…

25.4京年で合ってるだろか?2.7mmで考えると28京年くらいになる
…宇宙、終わってるかもしれんね
0373名無しさん@お腹いっぱい。
垢版 |
2017/03/10(金) 20:34:31.35ID:HHnyHPXV
>>371
車のスピード違反を調べるのに、アンドロメダ銀河と車両との相対速度を計測しても無意味でしょ。
それと同じ
0377名無しさん@お腹いっぱい。
垢版 |
2017/03/10(金) 21:04:44.44ID:hLdgWYFu
やっぱ、質量の大きな星って重元素の少ない軽い銀河とかに多いんじゃないの?
重元素が多いと星雲の冷却が速くなって、十分な質量が集まる前に星が形成されるから
0378名無しさん@お腹いっぱい。
垢版 |
2017/03/10(金) 21:28:42.64ID:AupFOEy9
>>372
Wikipediaの「光度の大きい恒星の一覧」の中ではR136a1とHD 269810の2つが
大マゼラン雲にある恒星だね
「R136」っていうのが星団の名前で、「質量の大きい恒星の一覧」の方だと
他にもいくつかランク入りしてるけど、同じページにある星団の写真を見ると
将来爆発しまくりそうな感じがするな
0379名無しさん@お腹いっぱい。
垢版 |
2017/03/10(金) 21:33:34.85ID:LHmvuy3b
>>374
ありがとうございます
銀河系以外の銀河の星の大きさも分かるのですね
ちなみに、大きさが分かっている星で最も離れているのは、地球からどれくらい離れているんですか?
0380名無しさん@お腹いっぱい。
垢版 |
2017/03/10(金) 21:40:01.24ID:l2zc79wd
有限の時間内には事象の地平面は生じない。
中性子星が質量降着によってBHになることを例に考えよう。
質量降着によって重力崩壊、潰れていくときに密度が高くなっていく。
密度が高くなることによって重力が強くなる。
その重力からの脱出速度が光速に等しくなる直前で時間が凍り付いて外から観察しても崩壊は全く進まなくなる。
重力崩壊によって脱出速度が光速に等しくなる領域が生じるまでにかかる時間は無限大です。
でなければ、外からみたBHに落ち込む物質は、BH表面に貼り付いて凍り付くことなく、そのまま事象の地平面内に突入していくでしょう。
しかしそれは矛盾なのです。
0383名無しさん@お腹いっぱい。
垢版 |
2017/03/10(金) 21:56:12.62ID:l2zc79wd
結論としてビッグバン以降に誕生したBHは存在しえません。
こうゆうSFネタは単純に物理学者の食い扶持のためにあります。
例えば学者のいうことを鵜呑みにする好奇心旺盛なスカポンチンがニュートンはじめ日経なんたらのSF雑誌を買うなら、そこには筆者や監修者や翻訳者といった多くの物理学出身者が携わるため、食い扶持になるんです。
僕はものの善悪を問うつもりはありません、物理学が発展する必要があるなら、詐欺師を鵜呑みにするしかない阿呆の一般人が騙されても仕方ないと思います。
でも自分は物理学には携わってはいません、真実を知りたい、鵜呑みにする阿呆ではいたくない、だから鵜呑みにすることのない同士を求めているのです。
僕の意見に賛同して下さい、BHに落ちていく物質がイベホラに貼り付いて動かなくなるなら、そもそもどうやってイベホラが生じるんでしょうか?
0386名無しさん@お腹いっぱい。
垢版 |
2017/03/10(金) 22:26:42.57ID:l2zc79wd
>>382
前半は正しいが残り半分がうそ。
そう対論が正しいならそもそも脱出速度が光速になる領域は有限時間内に現れない。
bhに落ちてくものがイベホラに張り付くように、重力崩壊も凍りついて止まる。
でないなら落ちてくものはそのままイベホラを通過してbhのイベホラ内の質量が増える。
0388名無しさん@お腹いっぱい。
垢版 |
2017/03/10(金) 22:43:18.75ID:l2zc79wd
崩壊途中で凍りついたbhも質量分の重力の影響を及ぼす
0389名無しさん@お腹いっぱい。
垢版 |
2017/03/10(金) 23:29:32.36ID:xyu8uAQk
まだブラックホールの流れなの?
ディスカバで小惑星やってるけどやっぱ小惑星はロマンだわ
0394名無しさん@お腹いっぱい。
垢版 |
2017/03/11(土) 08:56:07.75ID:rXUcxt9C
ブラックホール、そんなものは存在しない。
存在するのは重力崩壊途中で時間の止まった天体。
事象の地平面=脱出速度が光速になる領域が現れる直前で時間が止まる。
ただし質量はあるから周囲に質量の分だけの影響は及ぼす。
BH質量がある=BHがあるではなく、BHに崩壊途中の天体がある、だ。
事象の地平面が現れるのは無限大時間の経過後、自由落下視点では有限の時間内に事象の地平面を超える。
事象の地平面を到達した時点で事象の地平面の外側では無限大の時間が経過する。
つまり自由落下視点でも事象の地平面に到達する前にBHは輻射によって蒸発する。
0395名無しさん@お腹いっぱい。
垢版 |
2017/03/11(土) 09:26:49.89ID:zeMJX75+
>>364
こんな無駄に重量感あるもの地上から打ち上げられない。国際宇宙ステーションみたいに宇宙空間で組み立てるのもアリだけど、それでもこんなデザイン重視の機能性は二の次なんてモノを造るほど技術に余裕はない。
0396名無しさん@お腹いっぱい。
垢版 |
2017/03/11(土) 09:59:04.81ID:FsOanbok
BHは成長する。
つまり、有限時間内に物質を吸い込んで巨大化する。
吸い込まれる途中の時間経過が引き延ばされるのは、その物質から見た時間経過であって、外から見る者にはドンドン吸い込まれていくように見えている。
0398名無しさん@お腹いっぱい。
垢版 |
2017/03/11(土) 10:54:21.43ID:HQC9jLfo
>>394
> でも自分は物理学には携わってはいません、真実を知りたい、鵜呑みにする阿呆ではいたくない、だから鵜呑みにすることのない同士を求めているのです。
> 僕の意見に賛同して下さい、BHに落ちていく物質がイベホラに貼り付いて動かなくなるなら、そもそもどうやってイベホラが生じるんでしょうか?

じゃ教えてやる
相対論ではBHに落ちていく物質がイベホラに貼り付いて動かなくなる「外から見てそのように見える」ということしか言えない
見える時刻と実際の時刻は違う、そして相対論によると時刻はすべて相対的なもので、
自分自身の時刻以外は一意に決めることができない
つまり有限時間内にイベホラが生じているでも生じていないでもどちらも正しい
同じ場所で会話してる2人でも、ひとりは月が自分の右にあると言い、ひとりは自分の左にあると言う、どちらも正しいのと同じだ
0399名無しさん@お腹いっぱい。
垢版 |
2017/03/11(土) 11:35:56.89ID:rXUcxt9C
>>397
質問は、物理学者はみんな詐欺師なのか?です。
ありもしない天体をデッチあげて著作物の著作、監修、編集、翻訳費を騙し取っているのではないか。
>>398
イベホラは元々あるものじゃありませんから、重力崩壊が進んだとしてもイベホラが有限の時間内に現れることはありません。
重力崩壊前のイベホラの体積はゼロです。その体積が有限の値を持つまでに無限大の時間がかかると言ってるんです。
どんな物質もそもそもイベホラの内側に入ることが出来ません。
0400名無しさん@お腹いっぱい。
垢版 |
2017/03/11(土) 11:40:39.12ID:8ocXJ7O3
イベント・ホライゾンに張り付いて見えるのは、吸い込まれた物体が発する光であって吸い込まれた物体そのものではないことに留意しよう
あと、光円錐で考えるとわかりやすいが時間の方向がイベント・ホライゾンの中と外では違ってるんだよ
0401名無しさん@お腹いっぱい。
垢版 |
2017/03/11(土) 11:44:13.51ID:8ocXJ7O3
イベント・ホライゾンの外側では特異点は空間の座標を持つが、内側では特異点は時間座標に変わってるんだな
要するにブラックホールの中では特異点は場所ではなく未来に存在すると言えるわけだ
0402名無しさん@お腹いっぱい。
垢版 |
2017/03/11(土) 11:45:40.93ID:FsOanbok
言えないよ
なにそれ?
0403名無しさん@お腹いっぱい。
垢版 |
2017/03/11(土) 11:45:45.48ID:psQ7ho6J
ブラックホール愛が凄いな
まさに無限大の愛情
0404名無しさん@お腹いっぱい。
垢版 |
2017/03/11(土) 12:02:20.31ID:8ocXJ7O3
http://wondephysics.web.fc2.com/images/einsteinBHinside.bmp
この図が結構きれいに纏まっている

光円錐というのは光が取れる経路を示したもので、単純にいうと円錐の開いている方向が未来もしくは過去になる
円錐の面もしくは線分を世界面とか世界線とかいう

重力ポテンシャルが0に近い場所では光円錐は直立して描くことが可能で上方向が未来、下方向が過去になる

イベントホライズン上の光円錐を見てみると光のとれる経路がイベント・ホライゾンの内側にしかないことがよく分かる
つまり、光は出てこれない
また光円錐の世界線を見るとイベント・ホライゾンとキレイに重なっている
つまり、外からは時間が止まっているように見える
実際には光のとれる経路はイベント・ホライゾンの内側に確保されているので吸い込まれた物体の時間が止まっているわけではない

で、どんどん特異点に近づいていくと面白いことに光円錐は横倒しになっていってついには特異点方向に円錐が開く経路図になる
この状況では特異点は未来にあるということが出来るわけだな
イベント・ホライゾンは過去にあるといえるわけだ
0405名無しさん@お腹いっぱい。
垢版 |
2017/03/11(土) 12:08:35.75ID:rXUcxt9C
中性子星のイベホラの体積はゼロです。
そこへ質量降着が発生して重力崩壊に至ります。
重力崩壊によって密度が上がると、中性子星内部に脱出速度が光速に近くなる領域が現れます。
脱出速度が光速になる直前つまりイベホラが現れる直前で時間の経過が止まり、物理反応が起こらなくなります。
この時のイベホラの体積はゼロです、イベホラの体積がゼロから有限の値を持ったときに脱出速度が光速度になります。
加速器で言うなら加速している素粒子に無限大のエネルギーを投入したか無限大の時間加速したことを意味します。
そもそもイベホラの体積がゼロである以上は、どんな物質もゼロの体積から生じるイベホラの内側には入れないことになります。
ただしビッグバン以前から存在していたイベホラがあるならこの限りではないかもしれません。
しかしその場合でも事象の地平面に到達する瞬間に外部では無限大の時間が経過するのでブラックホールは蒸発してしまうか素粒子の寿命が終わるかもしれません。
物理的に何が起こるのかは言えないでしょう。
0406名無しさん@お腹いっぱい。
垢版 |
2017/03/11(土) 12:49:49.83ID:HQC9jLfo
>>399
重力崩壊が進んだとしてもイベホラが有限の時間内に現れることはありません。 → △(〇でもあり×でもある)
重力崩壊前のイベホラの体積はゼロです。 → 〇
その体積が有限の値を持つまでに無限大の時間がかかると言ってるんです。 → △(〇でもあり×でもある)


>>405
外から見ればイベホラの体積はゼロだ
見かけ上はどれもBHになる一瞬前の中性子星だ
では今からその中性子星にタッチして帰ってくることができるか?

未来永劫いつまで見ててもイベホラができないんだから、時間かければタッチしてくることができると思うだろ
答えはNOだ。
タッチする前のとある瞬間イベホラを通過する。
0407名無しさん@お腹いっぱい。
垢版 |
2017/03/11(土) 12:50:55.24ID:FsOanbok
人類のような奇形的突然変異での進化って、超珍しいのでは?
生命が誕生している星は沢山あるでしょうが、進化している種族がいるのは地球が最先端かもしれないね。
0409名無しさん@お腹いっぱい。
垢版 |
2017/03/11(土) 12:57:35.13ID:zeMJX75+
>>407
かもね。他の知的生命体が見つからない限り「その可能性はある」としか言えないけど。でも人類が宇宙トップだったら嬉しいね。
0411名無しさん@お腹いっぱい。
垢版 |
2017/03/11(土) 13:14:50.64ID:HSzLFNl4
>>407
肉体的、DNA的な視点で見れば人間は他の野生動物と比べ特別進化してるとは言えない。
僅か3万年前まで、類人猿や野生動物と生存競争していた。
そして現代人は、その当時とほとんど変わっていない。

劇的に進化したと錯覚するのは人間が文明を築いたから。
文明とは情報の蓄積。人間は情報をコントロールできるようになっただけ。

例えば、巨大隕石が落ちて全てが壊れたとしよう。
教科書も辞典も全て失う。
そこへ宇宙ステーションから戻ってきた数人の人間(生物学者)だけが生き残り、子供を作ったとする。
この人間の生き残りは、再び3万年前と同じような狩猟農耕の生活をしていく。
人間が蓄積した全ての知識を失ったからだ。
0413名無しさん@お腹いっぱい。
垢版 |
2017/03/11(土) 13:19:48.55ID:FsOanbok
>>410
なるほど
そのとおりだね
0414名無しさん@お腹いっぱい。
垢版 |
2017/03/11(土) 13:20:35.55ID:mdZjc6uD
>>399
>質問は、物理学者はみんな詐欺師なのか?です。

その可能性は限りなzくゼロに近く、統計学上では「ありえない」と断言して
差し支えない程度です。


>ありもしない天体をデッチあげて著作物の著作、監修、編集、翻訳費を騙し取っているのではないか。

陰謀論全体で言えることだけど、そんなリスクの高い事をして小銭を稼ぐより、まっとうな事をして
お金を稼ぐ方が遙かに低リスクで高リターン。


これで質問には回答されたわけだから、もう来ないでね。
0415名無しさん@お腹いっぱい。
垢版 |
2017/03/11(土) 14:36:58.34ID:rXUcxt9C
なぜありもしない天体があると言われるのでしょうか?
納得のできる回答があるまでは、まだまだ来させてもらわなければならないようですね。
0421名無しさん@お腹いっぱい。
垢版 |
2017/03/11(土) 21:09:56.26ID:osTbBpnB
というか脳や体の細胞が細かくなったり、大脳新皮質他の動物より圧倒的に大きくなったのが進化だと思うんだが。
脳の細胞が他の動物より細かかったりや大脳新皮質が大きいのは、人間らしい知能の原因だし。
筋肉の細胞が細かいのはチンパンジーのように力は出ないけど、道具などを使って細やかな行動をできるようにするため。
人間とチンパンジーが分岐したのは500年前から700年前ぐらいと言われている。
チンパンジーはチンパンジーとピグミーチンパンジーとしか分かれていないが。
人間はいくつもの種族が生まれては絶滅してを繰り返している。これはチンパンジーより人間の方が進化が早かった証拠にもなるだろう。
もし仮に地上の人間が絶滅したとしても(その仮定自体変だが)、また3万年が経てば(知識が少しでも残っているならもっと早く)元に戻ってるだろ
0422名無しさん@お腹いっぱい。
垢版 |
2017/03/11(土) 21:11:25.95ID:WPTqvYWh
三万年前と現代で人間の肉体はほとんど変わってないのか
なるほど、俺に新しい知識を与えてくれるレスだ
0423名無しさん@お腹いっぱい。
垢版 |
2017/03/11(土) 21:17:02.09ID:HSzLFNl4
>>421
3万年前に絶滅した他の数種類の類人猿と脳容積は大して変わらん。
そして現代人は3万年と比べ誤差レベルでしか脳容積が増えていない。
0424名無しさん@お腹いっぱい。
垢版 |
2017/03/11(土) 21:21:01.46ID:HSzLFNl4
>>421
300万年前のアウストラロピテクスがチンパンジーと分岐した。
そしてそのアウストラロピテクスを元に、北京原人ジャワ原人ネアンデルタール人ホモサピエンスなど20種が登場した。

寒冷地に適応したネアンデルタール人は人間よりも知能が高く体格もでかかった。
しかし肉体的な問題で「会話」という情報スキルが人間に及ばなかった
0425名無しさん@お腹いっぱい。
垢版 |
2017/03/11(土) 21:30:48.56ID:HSzLFNl4
子育てにおいて教育が大事とされるのは人間が情報を与えられなかったらただの猿であらからだ。
教育によって過去の人類が溜め込んだ情報を教え込む
それにより人間らしくなれる。

日本人の赤ん坊がアマゾンのジャングルで拾われれば土人になる。
文明とは情報集積なのである。

今はIT革命により情報のブレイクスルーの真っ只中。
更なる文明の進化がおきる。
米国は「情報」という資源の独占国だ。
ビッグデータがなければAI革命も量子コンピューターも実現できない。
日本はGPS衛星を米国に借りている時点で将来性無し。
中国は米国にビッグデータを与えていない。
残念だがこれが現実
0426名無しさん@お腹いっぱい。
垢版 |
2017/03/11(土) 21:34:12.69ID:HSzLFNl4
「宇宙を制する」とは、遠くの星に移住することではない。
「情報」を制することが、宇宙を制することに繋がる
0427名無しさん@お腹いっぱい。
垢版 |
2017/03/11(土) 21:36:27.09ID:FsOanbok
ネアンデルタール人がホモサピエンスより知能が高かったという科学的根拠はない。
言語能力も遥かに劣っていただろうと言われている。
0428名無しさん@お腹いっぱい。
垢版 |
2017/03/11(土) 21:41:40.66ID:HSzLFNl4
>>427
学会でも反論はあるが、脳容積はホモサピエンスより大きい傾向がある。
寒冷地に適応していた点や、遺跡発掘による考古学知見からもホモサピエンスより知能が高かったという説が有力。

言語能力がホモサピエンスより劣っていたのは当然。
そうでなかったら人間が絶滅していた。
0429名無しさん@お腹いっぱい。
垢版 |
2017/03/11(土) 21:57:29.72ID:zeMJX75+
>>426
人間に限らずあらゆる生物の本能と言うか存在意義は繁殖する、つまり生息域を拡大することなのだから、やはり移住が宇宙開発の最終的な目的だろうと思うけどな。
0430名無しさん@お腹いっぱい。
垢版 |
2017/03/11(土) 23:13:00.06ID:XhkssYQm
人間はなぜ生きてるのですか
0432名無しさん@お腹いっぱい。
垢版 |
2017/03/11(土) 23:29:29.09ID:HSzLFNl4
>>430
ホモサピエンスが他の類人猿との生存競争に勝ち、敵がいなくなった時、人類は同種間で争いを始めるようになった。
そして会話により討論し知識を高めていくと、君のような漠然とした多くの疑問を持つに至った。

人類はその疑問や不安を解決する為に、「神話」を作った。
最初は、当時の世界の中心であるエジプトで作られた。
後の宗教である。
0435名無しさん@お腹いっぱい。
垢版 |
2017/03/12(日) 00:54:50.46ID:3StgSr9q
>>431
じゃあ4んでみてぇえよ
0436名無しさん@お腹いっぱい。
垢版 |
2017/03/12(日) 00:55:01.04ID:mH9VfwTl
月が頭の上にあるときと、地球の真裏にあるときでは、重力加速度変わる?
0438名無しさん@お腹いっぱい。
垢版 |
2017/03/12(日) 01:00:50.23ID:3StgSr9q
>>435
そう事じゃなくて、4への恐れがないって事か?^_^;
そうだったらすまん
0439名無しさん@お腹いっぱい。
垢版 |
2017/03/12(日) 01:14:07.10ID:3StgSr9q
>>436
もちろん。
月が上にある時は、地球の重力が下に引っ張ってるけど、月の重力が上に引っ張ってる。
下にある時は両方引っ張ってて
それによって潮の満ち引きが起こってる。
質量mの物体の重力は質量M惑星との距離をrとし、万有引力定数をGとすると。
GmM/r^2
で表される。
惑星にも物体と同じ力の重力が働いてるけど、質量が大きすぎてびくともしないんだよね。
0440名無しさん@お腹いっぱい。
垢版 |
2017/03/12(日) 01:18:50.01ID:3StgSr9q
>>439
あ、重力加速度は運動方程式でma=GmM/r^2 より
a=GM/r^2で表される。
0442名無しさん@お腹いっぱい。
垢版 |
2017/03/12(日) 02:16:53.04ID:8xT5CpfP
>>437
これも論争になってるけど、個人的には交配はあったと思うね。
現代人でも獣姦マニアとかいるくらいだし。

でも、交配によって産まれた混血種は子孫を残せなくて淘汰されたと思う。
だってホモサピエンスの唯一の優位性だった「複雑な発音のできる声帯」を劣化させる行為(交雑)だからね。
0443名無しさん@お腹いっぱい。
垢版 |
2017/03/12(日) 02:20:32.47ID:8xT5CpfP
>>437
ああなるほど。
ネアンデルタール人の血が混ざった人類はほとんどいないが(残ってないが)
人類の血が混ざったネアンデルタール人はかなり前からいたってことか。
0444名無しさん@お腹いっぱい。
垢版 |
2017/03/12(日) 03:21:16.93ID:3StgSr9q
DNAも持っているという説もあるよ。(最近はそっちの方が主流)
http://logmi.jp/192860
0445名無しさん@お腹いっぱい。
垢版 |
2017/03/12(日) 03:54:06.46ID:Xe8ON3pr
ていうか、ネアンデルタール人が知り合いとか同級生の中に居るけど。
前頭葉部が後退していて、脳の容積が絶対的に少なく、言語もあまり明瞭じゃない奴だろ。
あいつだよ。
0446名無しさん@お腹いっぱい。
垢版 |
2017/03/12(日) 04:04:54.20ID:8xT5CpfP
いや、リアルのネアンデルタール人は寡黙で、話をしてもドモって何しゃべってるかよくわからない
0448名無しさん@お腹いっぱい。
垢版 |
2017/03/12(日) 04:07:48.31ID:hO1bVCBX
>>421
脳の進化は代償が多すぎ
脳梗塞、脳出血、クモ膜下出血、認知症、精神病
チンパンで止まってたほうが幸せだったかも
0449名無しさん@お腹いっぱい。
垢版 |
2017/03/12(日) 04:11:45.36ID:8xT5CpfP
つうか日本の教育ってネアンデルタール人を目指してるよな。
学校のテストで良い成績出すことより、コミュニケーション能力を磨くほうが人生において大事だと思うわ。
英会話とかもね。
0450名無しさん@お腹いっぱい。
垢版 |
2017/03/12(日) 04:19:36.26ID:ANcXPPxN
ネアンデルタールは捏造でしょ
0451名無しさん@お腹いっぱい。
垢版 |
2017/03/12(日) 04:26:27.20ID:hO1bVCBX
他人叩きは無意味だと気づけよ
競争がなくなれば進化のスピードは落ちるが争いもなくなる
宇宙の謎解明する前に戦争か疫病で文明崩壊濃厚なんだから
0452名無しさん@お腹いっぱい。
垢版 |
2017/03/12(日) 05:06:22.90ID:8xT5CpfP
星の成り立ちや年齢、星の一生などを物理現象として計算して把握できるけど
系外惑星の生命体が意図的にある星を破壊したら、それは物理現象と言えるのだろうか。

少なくとも、何故破壊されたかを計算はできない。
不可能だろう。

つまり生命体がいたら、物理が成り立たなくなるのでは?
計算不可能な現象を引き起こしてしまうから。
それとも惑星破壊程度なら、不確定性という物理法則の範疇に入るのでしょうか?
0453名無しさん@お腹いっぱい。
垢版 |
2017/03/12(日) 06:04:38.60ID:gGSAbj+d
>>452
突き詰めれば「私」という存在を物理現象として説明しきれるかという問題になるな
一言で言ってしまえば「そんなの無理」
0457名無しさん@お腹いっぱい。
垢版 |
2017/03/12(日) 08:03:36.85ID:C2/30fIj
>>456
じゃあ銀河の中心にあるあれはなんですか?
0458名無しさん@お腹いっぱい。
垢版 |
2017/03/12(日) 08:15:54.10ID:C2/30fIj
>>456
やっぱいいわ他のスレであったから
0459名無しさん@お腹いっぱい。
垢版 |
2017/03/12(日) 08:29:12.05ID:qQ6R6WbK
>生命体が意図的にある星を破壊したら、それは物理現象と言えるのだろうか。

不自然な爆発として観測されるだけだから、そういうのを物理現象じゃないとは言わない
0460名無しさん@お腹いっぱい。
垢版 |
2017/03/12(日) 08:46:03.98ID:BklaSo/t
>>457
重力崩壊途中の天体です。
外部から観測して事象の地平面が有限の時間内に現れることはありません。
脱出速度が光速度になる領域が現れる直前で時間が止まってしまいます。
脱出速度が光速度に到達する領域が現れることを観測するまでに無限大の時間が必要です。
ですからビッグバン以前から存在していたものを除き、この宇宙にはブラックホールなどは存在しません。
ただしブラックホールが存在した方が話題になるのは確かです。
本や映画などの著作物は売れるでしょう。
その著作者、監修者、翻訳者ぱ物理学出身の者なので、ブラックホールは物理学者にとっての飯の種です。
生活がかかっていますから無いとは言えないのです。
>>459
この世界は物理現象が全てではありません。
物理量を持たない意識・クオリアは実在しているのです。
私は寧ろ、この宇宙は意識が作り出していると考えた方がシンプルに説明できると考えます。
でなければ物理定数は天体が存在するために非常に都合のいい値になっており、これを説明する方法がありません。
別の宇宙を持ちだすこともできません。
なぜなら、それは物理ではないのですから。
物理で扱えるのはこの宇宙の中のことだけです。
この宇宙とは空間のことです。
空間があるから物質が運動することができ、その運動を物理として理解します。
重要なのは扱っているのは物質の運動であり、空間そのものではありません。
空間の膨張は斥力があるからとしか言えません、空間の歪みは電磁波にとっての最短経路が変化したとしか言えません。
なぜなら空間を直接、測定する方法がないからです。
その意味では空間も物理量を持ちません。
物理量を持たないものが実在して、我々はその中にいます。
もちろん我々自信も、物理量を持たない存在そのものです。
意識・クオリアを計測することはできないからです。
多くの物理法則がごちゃごちゃに成立していると考えるよりも、オッカムのカミソリに沿って言うならば、意識が宇宙を作り出していると考える方が合理的です。
0461名無しさん@お腹いっぱい。
垢版 |
2017/03/12(日) 09:13:12.51ID:BklaSo/t
そもそも「空間」は定義することができないでしょう。
空間を別の言葉で言い換えるなら「世界」、「真空」、「宇宙」などといった言葉しかないかもしれません。
しかしこれらは似て非なるものです。
空間は少なくとも「電磁場」と「重力場」という二つの言葉の合体でしか言い表すことが出来ないでしょう。
みなさんの頭の中にある「空間」とは実は非常にあいまいなものなのです。
そして未だ統合することのできていない電磁気理論と重力理論の合体が「空間」なのですから、言ってみれば空間とは「大統一理論の場」ということになるのかもしれせん。
私たちの曖昧さは良さでもあるのですが、そこには落とし穴があることも認識するべきでしょう。
また、これは物理世界を観測した結果としての空間の説明です。
純粋に論理的には「空間」とは距離と角度に分解できると考えています。
空間は距離と角度という因数の合体なんですね。
詳しくはまた後で書きます。
0464名無しさん@お腹いっぱい。
垢版 |
2017/03/12(日) 12:46:48.72ID:31+2AcQp
>>449
>学校のテストで良い成績出すことより、コミュニケーション能力を磨くほうが人生において大事だと思うわ。

ブラックホールが存在しない云々言ってる奴は他の人とコミュニケーションとれてないでしょ?
最低限の知識がないと他人とコミュニケーションなんてとれないし、とれてると思っても浅いレベルでしか
ないから、どっちの方が重要とかない。
両方大事。
ってか義務教育で成績良くない知的レベルだとコミュニケーション能力も低いよ。
0467名無しさん@お腹いっぱい。
垢版 |
2017/03/12(日) 14:07:55.12ID:nVaquIHk
>>439
月が頭の上にあるときと、地球の真裏にあるとき
「どちらも」重力加速度が小さくなるって知ってる?

> 惑星にも物体と同じ力の重力が働いてるけど、質量が大きすぎてびくともしないんだよね。
びくともしないことはない
質量が大きかったらその分働いてる重力も大きいんだから
0469名無しさん@お腹いっぱい。
垢版 |
2017/03/12(日) 15:42:34.53ID:qQ6R6WbK
>>460
空間が存在するなんて言う物理屋は居ませんよ

>物理量を持たないものが実在して、我々はその中にいます。
実在するものなら物理量として定義できるはずです
実在するのに物理量ではない←ちょっと意味が分からないですね

宇宙が意識で作られているという考え方はありえますが、
そう考える方が合理的であるかどうかは説得力に欠けますね
0470名無しさん@お腹いっぱい。
垢版 |
2017/03/12(日) 16:09:20.08ID:qQ6R6WbK
>>461
>私たちの曖昧さは良さでもあるのですが、そこには落とし穴があることも認識するべきでしょう。

空間は人の頭の中にしかないただの概念ですから、人によって想像するものが違ってくるのは確かにそうです

ですが、理論の中に登場する空間の概念はその違いをはっきりさせたものが使われてます
逆を言えば理論毎に空間の定義が違ってくるということですね
0471名無しさん@お腹いっぱい。
垢版 |
2017/03/12(日) 16:33:29.62ID:qQ6R6WbK
おそらく物理量を誤解してるのだと思いますが、

たとえそれが意識が作り出した幻だとしても、
幻の物体Aと幻の物体Bを比較すれば大きさがどれくらい違うのか 比べる ことが出来ますよね
つまり物理量それは、異なる二つを比べるということでしかないのです

幻の定規を基準にすれば、幻の距離という名の物理量を定義できますよね
ただそれだけのことです
0472名無しさん@お腹いっぱい。
垢版 |
2017/03/12(日) 16:36:37.07ID:M4jooLiw
この世には、大きなシャボン玉、中くらいのシャボン玉、小いさなシャボン玉が
ありますが、大きなシャボン玉には、人が中に入る空間があり、中くらいのシャボン
玉には、小さな数個のシャボン玉を入れることが可能であります。

宇宙は、中くらいのシャボン玉に小さなシャボン玉が数個入れているのに、少し似
ているかも知れません。

口元から放たられるシャボン玉は、あらかじめ中くらいのシャボン玉をつくり、
そして、小さなシャボン玉を数個入れようと思念の意識したときに、そのように、
中くらいのシャボン玉の中に数個の小さなシャボン玉を入れることができるとし
たら、創造主が思念の意識で、大きな宇宙空間を創り、星雲や銀河や惑星を創造
することが可能であると思いませんでしょうか?


一京無限
0473名無しさん@お腹いっぱい。
垢版 |
2017/03/12(日) 18:41:50.77ID:3StgSr9q
>>449
勝手に翻訳家でも目指してろやw
0480名無しさん@お腹いっぱい。
垢版 |
2017/03/13(月) 04:24:04.52ID:/4ZncbXP
ドキュメンタリーで放送されてない答をいうやつは無能
ってな感じですね
馬鹿呼ばわりする前に自分の無能を恥じなさい
0481名無しさん@お腹いっぱい。
垢版 |
2017/03/13(月) 11:42:16.10ID:SR4gufXl
>>475
そういうのよく見るけど、高価な機材を買わすためにやってるんだよ。
100万の機材をポンポン買えるということ金持ってるんだろうね。
盲腸炎になるのもそう、医者を儲けさせるため。
過払い金請求もそう、これは弁護士の小遣い稼ぎになる。
これをやったのはあとあとグレーゾーン金利が問題になると知ってる存在。
盲腸もそう、遠隔で体内にアクセスできる存在。
ハイテク嫌がらせ?これも遠隔でやれる存在。
ただし人間がやってるんじゃないし、宇宙人でもない。
変な組織に狙われて?24時間365日電磁波を浴びせられてるっていう可能性もゼロではないんだが、ちょっと有り得んよね。
現実はもっとシンプル。
0482名無しさん@お腹いっぱい。
垢版 |
2017/03/13(月) 11:58:49.25ID:sVGeCyWv
S(n)はn番目の素数
[Σ1/S(n)^s]^∞-X=Σ1/n^s=0
[Σ1/S(n)^s]=1 ←sがゼロ点のとき素数のみで構成されたゼータ関数は1になる

E=Σcos[mc^2*logS(n)]/√S(n)-i*Σsin[mc^2*logS(n)]/√S(n)

mc^2 ←原子核エネルギー

Eは空間に存在する振幅が1/√S(n)の波の合成
この波の位相が変動して質量や光になる
原子核エネルギーがゼロ点のときE=1になるがこれは空間のエネルギーが全て質量に変わったことを意味する
原子核エネルギーが光を含まない時虚部が存在しないため原子核エネルギーはゼロ点しか取ることができない

hν=0になるため
E=mc^2*Σcos[mc^2*logS(n)]/√S(n)-i*mc^2*Σsin[mc^2*logS(n)]/√S(n)=mc^2+i*hν ←静止した質量エネルギーはゼロ点のエネルギーしか取ることができない
ゼロ点いがいの重さの質量は虚部が存在するため運動した状態になる

mc^2*Σcos[mc^2*logS(n)]/√S(n)=0になるmc^2のとき虚部は存在するため運動する重さ0のエネルギーつまり光になる
-i*mc^2*Σsin[mc^2*logS(n)]/√S(n)=hν≠0 ←mc^2*Σcos[mc^2*logS(n)]/√S(n)=0になるmc^2を代入するとき0にならないため光になる
0488名無しさん@お腹いっぱい。
垢版 |
2017/03/13(月) 18:43:47.42ID:mIkEbzYX
mc^2=Σ1/n^(1/2+mc^2)*cos[(hf+hν)*logn] ←原子核エネルギー

E=m(n)c^2+i*hν(n) ←原子核のとびとびのエネルギー値

冦c^2=m(n+1)c^2-m(n)c^2=Σ1/n^(1/2+m(n+1)c^2)*cos[(hf+hν(n+1))*logn]-Σ1/n^(1/2+m(n)c^2)*cos[(hf+hν(n))*logn]
冑ν=hν(n+1)-hν(n)=Σ1/n^(1/2+m(n+1)c^2)*sin[(hf+hν(n+1))*logn]-Σ1/n^(1/2+m(n)c^2)*sin[(hf+hν(n))*logn]

m(n+1)≒m(n)  ←原子核のエネルギー値がとびとびになっても質量エネルギーの大きさは変わらないため  

僞=冦c^2+i*冑ν=[(1/2+m(n+1)c^2)+i*hν(n+1)]-[(1/2+m(n)c^2)+i*hν(n)]≒i*[hν(n+1)-hν(n)] ←y軸の間隔がエネルギー間隔として現れる
0490名無しさん@お腹いっぱい。
垢版 |
2017/03/13(月) 19:03:40.70ID:82wPCJgZ
hνってひょっとしてプランク定数と振動数のことか?
それやったらE=hvなんやから、普通にEでええやないかw
0491名無しさん@お腹いっぱい。
垢版 |
2017/03/13(月) 19:03:53.34ID:cx/YvE7k
h hayasa 速さ
v 速度  速さ
0492名無しさん@お腹いっぱい。
垢版 |
2017/03/13(月) 19:04:19.58ID:cx/YvE7k
hv = 速さ二乗!?
0493名無しさん@お腹いっぱい。
垢版 |
2017/03/13(月) 19:14:43.79ID:NhrZ0x7u
hνの光を放つ2つの光源を一定の間隔をたもってvの速度で運動させる
  →→□〜〜〜→ ←〜〜〜□→→
           光源間の距離Rは一定
光を吸収する際の光源間のエネルギー
E = 2*√(hν*[1-(v/c)/√(1-(v/c)^2)]*hν*[1+(v/c)]/√(1-(v/c)^2)) + i*(hν*[1+(v/c)/√(1-(v/c)^2)-hν*[1-(v/c)]/√(1-(v/c)^2)) = 2*hν+ i*2*hν*(v/c)/√(1-(v/c)^2) =mc^2+i*hν'
光を放出する際の光源間のエネルギー
E = 2*√(hν*√(1-(v/c)^2)/[1-(v/c)]*hν*√(1-(v/c)^2)/[1+(v/c)]) + i*(hν*√(1-(v/c)^2)/[1-(v/c)]-hν*√(1-(v/c)^2)/[1+(v/c)]) = 2*hν+ i*2*hν*(v/c)/√(1-(v/c)^2) =mc^2+i*hν'
光源の運動方向とは同じ向きにhν'=2*hν*(v/c)/√(1-(v/c)^2)のエネルギーが光源間を流れる

運動方向と同方向に光を放つ光源からhν+hν1の光を放射し逆方向に光を放つ光源からhν+hν2の光を放射する時
E = 2*√((hν+hν1)*[1+(v/c)/√(1-(v/c)^2)]*(hν+hν2)*[1-(v/c)]/√(1-(v/c)^2)) + i*((hν+hν1)*[1+(v/c)/√(1-(v/c)^2)-(hν+hν2)*[1-(v/c)]/√(1-(v/c)^2))
hν=mc^2/2
E = √((mc^2+2*hν1)*(mc^2+2*hν2)) + i*mcv/√(1-(v/c)^2) + i*[(hν1-hν2)+(v/c)*(hν1+hν2)]/√(1-(v/c)^2)
hν1=hν2=hf
E = √((mc^2+2*hf)*(mc^2+2*hf)) + i*mcv/√(1-(v/c)^2) + i*[(hf-hf)+(v/c)*(hf+hf)]/√(1-(v/c)^2)
E = (mc^2+2*hf) + i*(m+2*hf/c^2)*cv/√(1-(v/c)^2) =Mc^2 + i*Mcv/√(1-(v/c)^2)
hν2=0
E = √((mc^2+2*hν1)*mc^2) + i*mcv/√(1-(v/c)^2) + i*hν1*[1+(v/c)]/√(1-(v/c)^2)
v'=c*{mcv+hν1*[1+(v/c)]}/√{ ((mc^2+2*hν1)*mc^2)*(1-(v/c)^2)+{mcv+hν1*[1+(v/c)]}^2 } ←vで運動する物体に運動方向後方からhν1の光を吸収させる時この速度で運動する
E = √((mc^2+2*hν1)*mc^2) + i*mcv/√(1-(v/c)^2) + i*hν1*[-1+(v/c)]/√(1-(v/c)^2)
v'=c*{mcv+hν1*[1+(v/c)]}/√{ ((mc^2+2*hν1)*mc^2)*(1-(v/c)^2)+{mcv+hν1*[1+(v/c)]}^2 } ←vで運動する物体に運動方向前方からhν1の光を吸収させる時この速度で運動する
v=0
E = √((mc^2+2*hν1)*mc^2) + i*hν1
v'=c*{mc0+hν1*[1+(0/c)]}/√{ ((mc^2+2*hν1)*mc^2)*(1-(0/c)^2)+{mc0+hν1*[1+(0/c)]}^2 }=c*hν1/(mc^2+hν1)←静止した物体にhν1の光を吸収させる時この速度で運動する
0494名無しさん@お腹いっぱい。
垢版 |
2017/03/13(月) 19:20:56.25ID:07fttzN4
なるほど。
質量ゼロの光が物体を動かせる理由がわかった気がする
押してるんじゃなくて電磁波のエネルギーが空間の作用で運動エネルギーに変換されてるという感じか
0496pyu-rakkusu
垢版 |
2017/03/13(月) 20:49:57.42ID:T8+Lf0l1
質量無くなってますよ。
0499名無しさん@お腹いっぱい。
垢版 |
2017/03/13(月) 23:24:24.62ID:3z0/CFqm
ついに、AI 人工知能も参戦してきたようですねぇ〜
0502名無しさん@お腹いっぱい。
垢版 |
2017/03/14(火) 15:24:12.72ID:Z46SqiDP
脳に知識を直接アップロードする実験に成功 勉強は不要に
http://tocana.jp/images/uploadingknowledge_02.jpg
http://tocana.jp/images/uploadingknowledge_04.jpg
http://tocana.jp/2017/03/post_12610_entry.html
電気信号を素人の脳に電極付きのヘッドキャップを通して送り込み、
フライトシミュレーターで操縦技術を学習させたところ、電気刺激を
受けなかった被験者よりも学習が早く、課題を33パーセント増でこなしたという。
0504名無しさん@お腹いっぱい。
垢版 |
2017/03/14(火) 15:42:49.11ID:TnJGBVE6
2*√(hν*[1-(v/c)/√(1-(v/c)^2)]*hν*[1+(v/c)]/√(1-(v/c)^2)) + i*(hν*[1+(v/c)/√(1-(v/c)^2)-hν*[1-(v/c)]/√(1-(v/c)^2)):mが無い
= 2*hν+ i*2*hν*(v/c)/√(1-(v/c)^2):mが無い
=mc^2+i*hν':mがいきなり出てきた

mはどっから湧いて出た
0505名無しさん@お腹いっぱい。
垢版 |
2017/03/14(火) 17:31:16.06ID:QP+hWnC4
E=e^(i*arcsin(hν1/√((mc^2)^2+(hν1)^2))*√((mc^2)^2+(hν1)^2)=mc^2+i*hν1
E=e^(i*arcsin(hν2/√((mc^2)^2+(hν2)^2))*√((mc^2)^2+(hν2)^2)=mc^2+i*hν2

i*hν1=i*mcv/√(1-(v/c)^2)
i*hν2=i*mc√(2GM/R)/√(1-(2GM/(Rc^2)))

i*∫hν1dv=[i*mc^2*√(1-(v/c)^2)-i*mc^2*√(1-((v+dv)/c)^2)]≒i*mc^2*[1-(1/2)*(v/c)^2-1+(1/2)*((v+dv)/c)^2]=i*[mvdv+m(dv)^2]≒i*mvdv
i*∫hν2dR=[i*mc^2*√(1-(2GM/(Rc^2)))-i*mc^2*√(1-(2GM/((R-dR)c^2)))]≒i*mc^2*[1-GM/(Rc^2)-1+GM/((R-dR)c^2)]=i*(GMm/R^2)dR

i*∫hν1dv≒i*∫mvdv=i*(1/2)*mv^2
i*∫hν2dR≒i*∫(GMm/R)dR=i*-GMm/R

mc^2+i*∫hν1dv+i*∫hν2dR≒mc^2+i*(1/2)*mv^2-i*GMm/R

hν=mc^2*[√(1-(v0/c)^2)-√(1-(v1/c)^2)] (v0<v1)
hν=mc^2*[√(1-(2GM/(R0c^2)))-√(1-(2GM/(R1c^2)))] (R0>R1)

質量mがv0からv1まで減速するとhν=mc^2*[√(1-(v0/c)^2)-√(1-(v1/c)^2)]の光を進行方向前方に飛ばす
質量mが質量Mの半径R0から半径R1の位置まで移動するとhν=mc^2*[√(1-(2GM/(R0c^2)))-√(1-(2GM/(R1c^2)))]の光をMの鉛直線に沿ってmからみてMと逆方向に飛ばす

この2つの光はvとRの変化域の周波数帯の光が合成されたもの
mを光速で運動する状態から静止させる時
Mから無限遠に離れた場所に存在するmをMに限界まで近づける時
v0=c v1=0
R0=∞ R1=2GM/c^2
hν=mc^2*[√(1-(0/c)^2)-√(1-(c/c)^2)]=mc^2
hν=mc^2*[√(1-(0))-√(1-(1)]=mc^2
mc^2のエネルギーが完全に光に変わる

E=mc^2*(1+i*[1-√(1-(v/c)^2)]+i*[√(1-(2GM/(Rc^2)))-1])=mc^2*(1+i*[√(1-(2GM/(Rc^2)))-√(1-(v/c)^2)])≒mc^2+i*(1/2)*mv^2-i*GMm/R
0510名無しさん@お腹いっぱい。
垢版 |
2017/03/15(水) 01:02:43.32ID:pbUoQpT2
>>504
hν=mc^2/2の光が円運動をしている
静止しているときは円状の全ての部位で光の周波数は一定
円それ自体が特定方向に運動する時
つまり右回りの円を右に動かす時
○→
上の円の上部では円の回転方向と運動方向の向きが一致するため周波数が増え
下部では減少する
周回積分して平均値を取る時いかのようになる

1/(2π)*∫[0→2π] hν/(1-(v*sinθ/c)) dθ=[arctan((v-ctan(0))/√(c^2-v^2)))-arctan((v-ctan(π))/√(c^2-v^2)))]*2*hν/√(1-(v/c)^2)
arctan((v-ctan(0))/√(c^2-v^2)))=arctan(v/√(c^2-v^2))+2nπ
arctan((v-ctan(π))/√(c^2-v^2)))=arctan(v/√(c^2-v^2))

1/(2π)*∫[0→2π] hν/(1-(v*sinθ/c)) dθ=n*2hν/√(1-(v/c)^2) ←になる
hν=mc^2/2なのでn=1
hν=mc^2/2の光が円回転しており円自体がvの速度で特定方向に運動する時
1/(2π)*∫[0→2π] hν/(1-(v*sinθ/c)) dθ=mc^2/√(1-(v/c)^2) ←全体として光の周波数が増える
0511名無しさん@お腹いっぱい。
垢版 |
2017/03/15(水) 01:07:07.91ID:Bcf9kKPt
光が質量になる時、電子の周りに光が重力場の影響でまきつき光が円を描く
この状態で中心の電子が運動して抜けると光の円のみが残されこれが新たな電子になる
0512名無しさん@お腹いっぱい。
垢版 |
2017/03/15(水) 07:41:26.68ID:hTYUOvBJ
ドップラー効果か
で運動方向の空間が伸び縮みしてんの?
違う慣性系になることによる単位時間の変化で単位時間あたりエネルギーが変化すんの(・・?
0514名無しさん@お腹いっぱい。
垢版 |
2017/03/15(水) 12:59:54.51ID:KQPwMYXK
光から質量への転化(または逆も)って詳細はどうなっているんでしょうね
超スロー再生は不可能で結果がわかっているだけなの?

素粒子衝突→光と新素粒子発生
→光エネルギーの反作用で新素粒子が別方向にとばされる

素粒子から光か別の素粒子に変わる以外の変わり方はあるのかしらん
単極磁気、スピン
0517名無しさん@お腹いっぱい。
垢版 |
2017/03/15(水) 15:56:39.26ID:e0fOmHOs
やっぱりビッグバン説破綻してね?
100歩譲って偽の真空か真の真空にあった原子が相転移時に核融合で物質化したとして
ビッグバン前の真空の原子がどうやって無から発生したか説明付いてない
結局無限宇宙説を無限真空説にしただけじゃねえの?
0518名無しさん@お腹いっぱい。
垢版 |
2017/03/15(水) 16:48:18.45ID:hTYUOvBJ
>>517
真空って圧力単位のゼロ端じゃねない?
それ空間って言いたいんだろうけどそもそも空間があるという前提でそこの圧力がないと言ってるのと同じ。
空間は多項式であって因数に分解できるんじゃないかな。
ちなみに空間の因数は距離と角度。
距離は情報の差異。
差異とは違いのことで二つ以上の違う情報がある時にだけ必然性を伴って「違い」が生じる。
例えばエネルギーとかでもいい、全く同じエネルギーしかないなら「違う」という概念は必要がない。
でもここで言う情報は物理的なものじゃなく概念上のものであって、この世界にあるものに例えるならエネルギーとかが近いか。
二つの素粒子の間の距離とかじゃなくて、そもそもその素粒子の状態を表す情報の違いこそが概念上の距離であって物理世界で扱う距離より遥かにスケールが小さい。
つまり空間とか(物理世界の)距離とか言う時点で色々なものがくっついた多項式のようなものじゃないかな。
根本的には世界は情報で成り立ってると考えないと根本的には何も説明できないような気がする。
0519名無しさん@お腹いっぱい。
垢版 |
2017/03/15(水) 16:53:04.81ID:e1fkxatv
ブラックホールの超近くを星が周回する「X9」、チャンドラX線観測衛星が観測
http://headlines.yahoo.co.jp/hl?a=20170315-00010001-sorae_jp-sctch
>長い間、球状星団にブラックホールが存在するケースはまれ、あるいは無いと考えていました。

なぜ、これまで球状星団にブラックホールは存在しないと考えられていたのですか?
0520名無しさん@お腹いっぱい。
垢版 |
2017/03/15(水) 18:11:52.18ID:WFYdCgAC
>>517
強い力、弱い力という核力が生まれて初めて物質ができる。
ビッグバン以前には4つの力が揃ってなかった。
目に見えない大量のエネルギーだけが存在していた
0523名無しさん@お腹いっぱい。
垢版 |
2017/03/15(水) 23:46:45.28ID:e1fkxatv
>>522
返答ありがとうございます。

球状星団をつなぎとめている重力源は、星ではなくダークマターとは考えないのだろうか?
0524名無しさん@お腹いっぱい。
垢版 |
2017/03/16(木) 00:28:02.50ID:XDxzvsfv
宇宙も生物と一緒説
オチンチンがなぜかゆらいじゃって溜まってたエネルギーがオマンマンに相転移して宇宙君が生まれた可能性
0526名無しさん@お腹いっぱい。
垢版 |
2017/03/16(木) 11:11:43.14ID:Y2EDZdO8
>>525
物理では答えられない。
なにしろ物理は空間があることが前提だから。
正確に言うなら電磁場と重力場。
みんなが無意識に空間と呼んでるものは電磁場と重力場の合体だから大統一理論の場があると言ってることになる。
そうじゃないなら、みんなが呼ぶ空間というのはあくまで目で見たものでしかない。
自分と壁の間には空間があると。
だが空間というのが何なのか説明できなかったな、だから正確には空間じゃなくて距離があるんだ。
説明できないものが目の前にあって、説明しようとするとどうしても距離としか言えなくなる。
>>460で書いてるが、空間も宇宙も意識が作り出してると考えるのがシンプルで合理的。
意識は説明のつかないものを違和感なく受け入れて、目の前にあるものが現実のように錯覚するためにある。
この世界にある全てのものは物理量を持つ(と錯覚している)が、意識はこの世界に存在するはずなのに物理量を持たない。
これは幽霊がいると言ってるのと同じ。
全部は心霊現象なんだと考えた方が大統一理論の式よりシンプルになるのは確実。
たがこんな考えには反論があるのも事実。
要はこの世界は幻であって、なぜ幻が存在する(または存在すると錯覚する)のかを説明するよりも、物理法則の方がシンプルなら、物理法則の方が真となれる。
究極的には両者はイコールなんじゃないかと思ってる。
0527名無しさん@お腹いっぱい。
垢版 |
2017/03/16(木) 11:26:00.45ID:Y2EDZdO8
宇宙には重力が働いてる。
ものは高いところから低いところに落ちると。
要は重力が働くことによって落差が生じる。
力が働いてるということはカオスでなくて秩序立ってるということ。
こういった力は意識にも働いてる。
例えば物理を理解できる人と理解できない人がいる。
そこには難易度の壁があるはずだ。
つまり理解することの難しさの程度の落差がある。
重力定数が定まっているように、実のところ誰が物理を理解できて誰が理解できないかも実のところ定まってる。
実のところ猿山にいるニホンザルが突然シェークスピアを朗読し始める可能性もあるし、東大入試をすらすら解くことだってあるんですよ。
あるのは難易度の落差ですから、それは起こり得る。
でも実際にそんなもの見たことがないのは、反重力装置を見たことがないのと同じ話なんですよ。
賢い動物がいると、人も驚くような知恵を働かせる、単純な話ですよ。
彼らにはそれだけの知恵が解放されているということです。
おかしいな、なんでこの分野の勉強を始めたら思考停止してしまうんだろ?
それは理解することが解放されていないからですよ。
0528名無しさん@お腹いっぱい。
垢版 |
2017/03/16(木) 11:40:51.98ID:XDxzvsfv
結局行き着く先は量子力学なんだなw
観測しない限り不確定
天動説が有力だった頃は実際に地球が宇宙の中心だったかもしれんし
0529名無しさん@お腹いっぱい。
垢版 |
2017/03/16(木) 12:13:58.42ID:Y2EDZdO8
何が観測してるかというと、観測機器じゃないんだな。
人の意識が観測してる。
観測機器も量子の集まりだからあらゆる観測結果の重ね合わせで存在してる。
それを確定した(と錯覚する)のが人の意識、つまり人が知る瞬間に一つの状態になる。
もちろん見て知るのでなくて聞いて知るでもいいし触って汁でもいい、全ては人の五感、つまりクオリア。
人はクオリアを通してしか知る方法がない。
0531名無しさん@お腹いっぱい。
垢版 |
2017/03/16(木) 12:28:30.03ID:Fhz2sATi
空間が全ての源であり全てである
物質は力(エネルギー)が作り出した偶像
中身はスカスカで実体のないものが物質
この世にあるのはエネルギーのみである
0532名無しさん@お腹いっぱい。
垢版 |
2017/03/16(木) 12:33:36.96ID:Fhz2sATi
空間は素粒子と密接に関係。
空間がエネルギーの一形態として素粒子を自在に表示する。
素粒子は物質ぽいが本質的には物質ではない
概念や数式でしか奴らを表せなきゃ
0535名無しさん@お腹いっぱい。
垢版 |
2017/03/16(木) 12:47:13.21ID:Fhz2sATi
>>533
空間のどんな場所でも、重力が働き、物質があれば質量が存在し、磁石があればくっつく。

これらが起きるには力の作用の媒介となる光子、重力子、ヒッグス粒子が必要

つまり全ての空間には、普遍的にこれら素粒子が存在する(現れる)ことになる。
0537名無しさん@お腹いっぱい。
垢版 |
2017/03/16(木) 12:50:40.34ID:3b3sgG49
>>535
場の最小単位と物質の最小単位を同一に捉えてるところがまだまだだな
0538名無しさん@お腹いっぱい。
垢版 |
2017/03/16(木) 12:52:23.88ID:3F9JhK5i
今この時はまだ宇宙じゃなくても、明日その場所は宇宙となっている
これが膨張だと思うんですが合ってますか?
合っていた場合、今この時宇宙じゃない場所に野球ボールを投げたらどうなりますか?
私の考えではどう考えても、ボールが空間を広げるとしか思えません
つまり、物質、物体が(光含む)が宇宙を膨張させている
と考えます
それにより歪な感じで膨張してるという理論になります
0539名無しさん@お腹いっぱい。
垢版 |
2017/03/16(木) 12:52:49.48ID:10h3u5pC
>>526
> みんなが無意識に空間と呼んでるものは電磁場と重力場の合体だから大統一理論の場があると言ってることになる。
大統一理論は電弱力と強い力の統一で重力は関係ないぞ。
0540名無しさん@お腹いっぱい。
垢版 |
2017/03/16(木) 12:54:24.34ID:y4XISfKK
>>525
それはいわゆる無ってやつだけど、無は空間や時間も含めて何も存在しない状態を言うが、存在しない無を存在すると言うと矛盾するから、無は有るとも無いとも言えるかな?どちらとも言えないかな?みたいな感じ。
0541名無しさん@お腹いっぱい。
垢版 |
2017/03/16(木) 12:55:49.67ID:Fhz2sATi
電界と磁界が共振して初めて電磁波として空間内を電波する光として人間が目視できるようになる。
電磁波になっている空間内の光子は極僅かに過ぎない

電界のみ磁界のみでも光子がその力を伝達してるが見ることができない
電磁気力は無限遠に作用する。
つまり光子も無限遠に存在する
0542名無しさん@お腹いっぱい。
垢版 |
2017/03/16(木) 12:56:36.44ID:3b3sgG49
無についてはせいぜい妄想することしかできないよね
検証することも理解することも多分できないから
0544名無しさん@お腹いっぱい。
垢版 |
2017/03/16(木) 12:57:35.80ID:3b3sgG49
>>543
物質の最小単位なんだから物質なんだよ
0547名無しさん@お腹いっぱい。
垢版 |
2017/03/16(木) 12:59:44.21ID:3b3sgG49
前期量子論からやり直し
0548名無しさん@お腹いっぱい。
垢版 |
2017/03/16(木) 13:00:01.20ID:Fhz2sATi
>>544
あくまでも観測行為によって物質として確定するだけ。
物質として考えるならその事象の素粒子全てを観測しなきゃならんが不可能
つまり実際には波として素粒子は活躍している
0549名無しさん@お腹いっぱい。
垢版 |
2017/03/16(木) 13:04:58.10ID:Fhz2sATi
物質は強い力、弱い力で結合されて初めて物質らしくなる。
素粒子単体は物質とはかけ離れた存在。
素粒子の塊だろうと見ることもできない
0550名無しさん@お腹いっぱい。
垢版 |
2017/03/16(木) 13:05:14.83ID:y4XISfKK
>>538
それは違う。宇宙空間ってのは球の表面みたいな物で、果ては無いけど大きさは有限。宇宙が拡がるというのはつまりこの球が膨張してるということ。
どう頑張ってもボールを宇宙の膨張方向に投げることはできない。どんな方向にボールを投げてもボールはこの球の表面(宇宙空間内)を進んで行く。
0552名無しさん@お腹いっぱい。
垢版 |
2017/03/16(木) 18:42:04.51ID:qer73ykK
この世界の本質ってなんなの?
空間ってほんとに存在するの?

「横」「高さ」「奥行き」が見えるから、脳が勝手に空間という概念を認知してるだけじゃないの?
0553名無しさん@お腹いっぱい。
垢版 |
2017/03/16(木) 18:42:27.49ID:qer73ykK
哲学で「なぜ何もないのではなく、何かがあるのか?」という難問があるらしいけど、
この世界が無いと「無い」という概念も無いんじゃないの?
0554名無しさん@お腹いっぱい。
垢版 |
2017/03/16(木) 19:02:02.58ID:EXEz4Eqo
アンドロメダ星雲って120万光年の距離って記憶にあるけど、最近の写真集を見たら250万光年って書いてあった。
いつの間にそんなに遠くなっちゃったですか。そもそもどうやってそんな遠くのものの距離がわかるのか不思議。
どっかで適当に決めてる訳じゃあないよね。
0555名無しさん@お腹いっぱい。
垢版 |
2017/03/16(木) 20:08:58.50ID:Fhz2sATi
>>554
三角測量。
ググれば小学生向けのサイトの質問コーナーでよく紹介されてる。

3世紀頃に中国で最初に利用された。
西洋では16世紀に地図作りの精密測量で三角測量され始めた。
0557名無しさん@お腹いっぱい。
垢版 |
2017/03/16(木) 20:54:21.94ID:ViT3h3xX
>>555
年周視差で測れるのは精々1000光年くらい迄
明るさが決まってる星の種類ってのがあってそのタイプの星がどれくらい明るく見えるかで距離を割り出す
0558名無しさん@お腹いっぱい。
垢版 |
2017/03/16(木) 21:26:31.47ID:c1uaI6vI
ボイジャーが太陽系外行こうとしてるけど、まだ通信出来てることに驚きなんだけど、今の技術だったら衛星にカメラつけて飛ばしてほしいな。それでyoutubeでも中継して欲しいんだけど可能?
0562名無しさん@お腹いっぱい。
垢版 |
2017/03/16(木) 22:41:53.41ID:10h3u5pC
>>558
伝送速度の関係で動画はおろか静止画でもリアルタイムはきついと思う。
それ以前に適切な被写体がないと思うけど、どんな映像を期待しているの?

>>561
原子力電池なら崩壊熱が出る限り発電できるから大丈夫。
時間の経過と共に出力が減少していくのは避けられないけど。
ただ必要なプルトニウムが枯渇しつつあるんだよな。
0563名無しさん@お腹いっぱい。
垢版 |
2017/03/16(木) 23:20:49.13ID:Fhz2sATi
距離が遠くても伝送速度は関係ないのでは?
ボイジャーも原子力電池だけど、ほとんど電源切りっぱなしで超節電してるけど
0565名無しさん@お腹いっぱい。
垢版 |
2017/03/16(木) 23:51:33.07ID:10h3u5pC
>>563
>距離が遠くても伝送速度は関係ないのでは?
遠くなる程、高速通信が困難になると言う意味で、んなわけはない。
例えばニューホライズンズの場合だと木星付近では37kbpsぐらいだったのが
冥王星付近では1kbpsぐらいにしている。
0566名無しさん@お腹いっぱい。
垢版 |
2017/03/17(金) 00:39:58.88ID:JF8t55n6
子供のころ宇宙や恐竜にハマるというのは致し方ない
しかし成長すれば、子供騙しだと気づく
大人になったら宗教をやるべきだ
宗教にしか答えはない
0568名無しさん@お腹いっぱい。
垢版 |
2017/03/17(金) 05:05:44.74ID:rMdin4XR
>>563
送信先に届くまでの間にノイズが紛れ込むと読み取り出来なくなっちゃうから

1010 →111000111000

という感じにして多数決を取れば、1つノイズで失われても大丈夫という考え方
(細かい事は冗長性でググれば出てくる)
データの保全性は高まるけど、量が増える分実質的な転送速度は落ちる訳だ
惑星探査機の場合は距離に応じて冗長性を変更して対応してるんだと思う
0569名無しさん@お腹いっぱい。
垢版 |
2017/03/17(金) 07:07:32.88ID:SUd2Y3uu
>>526
>>540
ありがとうございます
やはり 人間そのものが認識出来ない領域は 人間からしたら無に近いんですかね?

もう一つ質問なんですが
銀河同士や銀河団が お互いに離れたりする話しを聞いたことあるのですが、
よく例え話しででますが

膨らませる前の風船に マジックで点を何点か付け 膨らませた時に似ているって言いますが その 表面のゴムの強度や伸縮性みたいに 宇宙空間にも 空間の強
度や伸縮性の概念はあるのですか?

例え話しと混同してしまうのは 申し訳ないのですが あと長文も
0570名無しさん@お腹いっぱい。
垢版 |
2017/03/17(金) 10:08:09.34ID:lVFel17a
>>569
風船の喩えは風船に書いた点も膨らんで風船と点は相似形になる。
比率は同じなのでモノサシが変わっただけ。
ただこの場合に言いたいのは空間としての風船表面が膨らむから点同士が離れると言いたいんだろ。
だがそれは簡単に否定できる。
点が空間としての風船表面に固定されているなら点は空間の動きによってしか運動ができないことになるよ。
要は空間と物質は相互作用しないということだ。
でなければ慣性の法則は破れる。
言えるのは引力と斥力が働いてるというところまで。
ここでもまた空間の認識が間違ってることが明らかになった。
0573名無しさん@お腹いっぱい。
垢版 |
2017/03/17(金) 12:14:48.42ID:Eg/ZOQMe
超弦理論
https://youtu.be/7y_BlA3ZTeQ

この動画でも物理学者が言ってるな
人間が知覚できる次元は縦横高さと時間だけ。
生物としてその能力しか持ってなきからと。

超弦理論ではこの世には11次元が存在する。
ミクロの世界なので我々には知覚できない
0574名無しさん@お腹いっぱい。
垢版 |
2017/03/17(金) 12:24:22.42ID:LVSUR5QK
4次元しか知覚できないのに11次元はトンデモだろ
現状わからないことは全部量子力学のせいにすりゃいいよ
0575名無しさん@お腹いっぱい。
垢版 |
2017/03/17(金) 12:37:22.93ID:Eg/ZOQMe
>>574
人間が知覚できない現象は世の中には多い。
でも知覚できない現象を理論や実験で把握できている。
GPSの時間補正だってそうだし、量子テレポーテーションとかも
0577名無しさん@お腹いっぱい。
垢版 |
2017/03/17(金) 13:14:43.21ID:mD2TqJjI
11次元の要素で計算すると上手いこと説明できる体系ができそう、ってのと実際に11次元あるというのは似てるようでちょっと違うんやで
0578名無しさん@お腹いっぱい。
垢版 |
2017/03/17(金) 13:37:49.51ID:3FeuVUeA
東京駅に直径1mの原子核を置くと、パチ玉大の電子が回っているのは
千葉の銚子駅だという。その間の空間はスカスカで何もないらしい。
想像するだけでゾッとする
0579名無しさん@お腹いっぱい。
垢版 |
2017/03/17(金) 13:45:48.86ID:Eg/ZOQMe
相対性理論の発表時も、理論先行でトンデモ理論だった。
ましてブラックホールがあるだの時間が変わるだの妄想まで書かれていた。
でも後年、観測や実験で相対性理論に矛盾しない事実が次々と分かってきた。

今の超弦理論も同じ。理論は素晴らしい完成度。
超弦理論を否定することを考えるよりも、信じてその先を妄想するほうが人生においては有意義でなかろうか?

空間はエネルギーの輪で満たされていて、その輪の変化で光子になったり重力子になったり素粒子が産まれる
4つの力は空間エネルギーの一部。
力として現れないエネルギーは、ダークエネルギーだ
0580名無しさん@お腹いっぱい。
垢版 |
2017/03/17(金) 13:48:09.16ID:Eg/ZOQMe
>>578
その1mの原子核は、陽子と中性子が強い力で結ばれていて
陽子や中性子といった素粒子は、弦の形をしたヒモのエネルギーである
0581名無しさん@お腹いっぱい。
垢版 |
2017/03/17(金) 13:56:18.73ID:Eg/ZOQMe
>>578
そのスカスカの空間も見えないエネルギーの輪で満たされている
銚子駅にある電子が動くのも、エネルギーの輪が電子として変化し伝達していくだけ。

陽子中性子電子といった素粒子が現れるのは、大量にある弦の一部。
だから観測した時しか「点」として現れないし
電子の移動スピードも光速度という限界がある。

見えない弦エネルギーは、別次元にあるダークエネルギー
0582名無しさん@お腹いっぱい。
垢版 |
2017/03/17(金) 14:25:06.02ID:iSJtTn08
まあ最小単位が色々矛盾のでる点ではなくひもであるとしたのはひも理論の卓見だと思うな
完全な点や球なんかは概念だけだと思うし
0583名無しさん@お腹いっぱい。
垢版 |
2017/03/17(金) 14:37:25.85ID:1b+ZnaJk
俺は多世界理論を信じたい!!!
0584名無しさん@お腹いっぱい。
垢版 |
2017/03/17(金) 14:40:30.20ID:K/hyeta4
>>552
その通り
人は誰でも教育の過程で空間があって中身があると、まず最初に理解するが
人は知識を積み重ねることで本質を悟り、空間が存在しないことを理解する

空間が存在するかのように発言する人にこれを説明するのは難しいので、スルー推奨
0585名無しさん@お腹いっぱい。
垢版 |
2017/03/17(金) 14:45:10.56ID:K/hyeta4
宇宙は物質や波動の連なりでしかないが、それを脳が効率的に把握するために空間を設置する。
空間は人の脳内にしか存在しない。
0586名無しさん@お腹いっぱい。
垢版 |
2017/03/17(金) 14:51:51.25ID:K/hyeta4
ちなみに物体の動きを3次元座標+時間の4次元で表現するとき、
ここに鼻息の力で物体が動く計算を考えると+1次元
ここに吐息の力で物体が動く計算を考えると+1次元
ここにオナラの力で物体が動く計算を考えると+1次元
という具合に次元はいくらでも必要なだけ増やすことが出来る

何が言いたいかというと、
計算上の次元と、空間の次元は似てるようで全然違うという事
0587名無しさん@お腹いっぱい。
垢版 |
2017/03/17(金) 17:44:04.01ID:JF8t55n6
>>573
オカルトに通じてないとこういうのにすぐ騙される
アポロ11号、9.11、3.11、超弦理論11次元ってこと
0591名無しさん@お腹いっぱい。
垢版 |
2017/03/18(土) 07:18:56.39ID:i5T6GVRg
モノや情報が移動してたわけじゃないから相対論には抵触しない
宇宙という入れ物が広がっていただけ
0592名無しさん@お腹いっぱい。
垢版 |
2017/03/18(土) 08:07:49.16ID:uB1XEISK
疑問をとことんまで突き詰めるために宇宙とか空間という単語を使うのを止めるのはどうだろ?
広がったのは電磁場と重力場と。.
それだけで説明がつく。
にもかかわらず空間という言葉を出す必要があるのは例の宇宙膨張というやつかな。
でもこれは斥力場のはずなんだが。
そんなにたくさんの力と場があったらおかしいというのはある。
天体が存在するために都合が良すぎるというか天体が存在するために絶妙に力と場が成り立ってるとしか思えなくなるから。
それを分かってるから敢えて論理的矛盾を含む空間の膨張などという言葉を使ってるんじゃないかな。
物質が空間に引っ張られて離れていくなら、物質は空間にくっついてないとならない。
そうすると今度は自由に動けなくなる(筈だよね?)
0593名無しさん@お腹いっぱい。
垢版 |
2017/03/18(土) 11:04:30.43ID:bcUKASE2
超弦理論的に重力場の時空の歪みを考えると
エネルギーの弦の密度も、重力の強い所ほど高いということなのかな

弦の密度が高いから情報伝達(エネルギー伝達)も時間かかって結果的に時間が遅くなると。

空間の膨張も、弦が増えてるってことじゃないかな。
空間に流れる光や熱が、経年劣化で失われることによってダークエネルギーの弦に変換されて、膨張していると。
0594名無しさん@お腹いっぱい。
垢版 |
2017/03/18(土) 11:12:17.46ID:bcUKASE2
空間が広がるってことは、より時間がかかるってことでしょう?

例えば死兆星から地球へと光の届く時間が長くなるということが、結果的に空間が広がっているというだけ。

中性子星も強い重力場により時間の流れが遅くなる。
つまり、中性子星がある所だけ空間が伸びているとも言える。
ただ、中性子を取り囲む周りの空間は伸びてないので、空間が伸びたと表現されないだけでは?
0596名無しさん@お腹いっぱい。
垢版 |
2017/03/18(土) 11:19:26.35ID:bcUKASE2
天の川銀河の全体を1つの星として考えた時、
銀河の中心に近いほど物質密度が高く、重力場が強い=時間が遅くなる。ですよね?

だったら、銀河の回転曲線問題は当たり前の現象ではないのですか?
中心に近い星ほど銀河密度が高い場所であり時間が遅くなるからゆっくり回転してるように見える。

星単位の時間の影響は計算していても、銀河全体の場所による時間の差異を計算してるのかな
0597名無しさん@お腹いっぱい。
垢版 |
2017/03/18(土) 11:24:50.54ID:bcUKASE2
GPS衛星は地上より時間が速いので補正している。
これは、GPS衛星が地上より狭い空間にいるということ。
空間が狭いから時間が早く流れる
0598名無しさん@お腹いっぱい。
垢版 |
2017/03/18(土) 11:28:27.29ID:0iNOFM4k
>>594
地球から遠ざかるほど遠ざかる速度が速いという事は、君の理屈で言うと
地球から遠ざかるほどだんだん星の質量が大きくなるということ。
つまり地球が宇宙の中で一番軽い質量が集まってる中心で、360度均等に
地球から遠ざかるに従ってどんどん質量が大きくなる事になる。
要するに地球が宇宙の中心って事だな。

そんな都合の良い宇宙の姿より、空間が膨張してると考える方が自然だよ。
0599名無しさん@お腹いっぱい。
垢版 |
2017/03/18(土) 11:31:06.58ID:0iNOFM4k
>>596
銀河の質量程度では、そこまで時間は遅くならない。
まずはどれくらいの重力があるとどれくらい時間の進みが遅くなるのか調べよう。
0600名無しさん@お腹いっぱい。
垢版 |
2017/03/18(土) 11:41:20.57ID:bcUKASE2
>>598
重力場の強い場所は、重力によって空間密度(弦密度)が上がって、結果的に時間が遅くなる。空間が広がったと同じ解釈ができる。

空間膨張については、重力に関係なく、ダークエネルギーの増加により
空間密度(弦密度)が上がって、結果的に距離が遠いほど時間がかかる。

空間が広がったと考えずに、時間がかかる(空間内の情報伝達が遅くなる)と考えれば
弦エネルギーの集積や増加で表現できるのではないのかなと
0601名無しさん@お腹いっぱい。
垢版 |
2017/03/18(土) 11:43:00.48ID:uB1XEISK
>>595
物理学者は安易に空間などという言葉を使う前に言葉の定義をするべきだと思いませんか?
それとも真空とでも言い換えますか?宇宙?世界?
みんな監修料が欲しいだけの詐欺師なのでしょうか?
0606名無しさん@お腹いっぱい。
垢版 |
2017/03/18(土) 12:40:43.95ID:0iNOFM4k
>>601
>物理学者は安易に空間などという言葉を使う前に言葉の定義をするべきだと思いませんか?

現行の定義でなんら問題はないです。


>みんな監修料が欲しいだけの詐欺師なのでしょうか?

そういう妄想は典型的な統合失調症の症状なので、薬を増やしてもらってください。

マジレスすれば、監修料とか教科書利権で動くお金より、実際の宇宙の様子を隠したり
フィクションの姿を提示する方が試算するまでもなくコスト・リスクが大きいから、その
妄想は無意味。


というわけで質問には回答されたわけだから、もう来ないでね。
0608名無しさん@お腹いっぱい。
垢版 |
2017/03/18(土) 13:17:57.49ID:unMUVQ5t
理解できない事に耐えられないんじゃなくて、だいたいこう

理解できているという妄想 → その妄想と合致しない表現を見た → それは陰謀だからみんな俺の妄想を聞こうぜ
0609名無しさん@お腹いっぱい。
垢版 |
2017/03/18(土) 13:20:18.77ID:uB1XEISK
>>606
では、現行の定義とやらを教えてもらえますでしょうか?
それに答えてもらうまでは、まだまだ、来させてもらわなければならないようですね。
0611名無しさん@お腹いっぱい。
垢版 |
2017/03/18(土) 13:51:03.74ID:uB1XEISK
>>610
ぐぐったところ、このように書いてありました。

1.
何も無くあいている所。?「―をうずめる」
2.
地上で言う上下・左右・奥行きといった方向に(限りなく)広がる世界。

限りなく広がる世界で十分とはお笑いですな^o^
0612名無しさん@お腹いっぱい。
垢版 |
2017/03/18(土) 14:02:17.68ID:unMUVQ5t
>>611
わざわざ報告ありがとう。
やればできるじゃないか、カスとか言ってすまんかったね
疑問が解決してよかったしググり方を覚えたことはキミの何よりの成長だ。
0613名無しさん@お腹いっぱい。
垢版 |
2017/03/18(土) 17:24:19.44ID:Vrsuke4O
https://goo.gl/znnSuW
これは、普通にショックでしょ。。
本当なの?
0614名無しさん@お腹いっぱい。
垢版 |
2017/03/18(土) 18:07:53.85ID:uB1XEISK
初投稿が2017/3/9 ? IP覗きかw
どこから記事朴ってきた?

ときどき一記事しかないのにずっと前から続いてる風に装ってるブログあってびびるw
0616名無しさん@お腹いっぱい。
垢版 |
2017/03/18(土) 19:03:13.27ID:fjPudMb2
一ミリ程度の大きさの重さ1那由多トンくらいある物質を手のひらにそっとのっけるとどうなるんですかね?
手ごと地面に落ちるんじゃなく手のひらに穴があいて物質だけ落ちる予想なんですけど
0617名無しさん@お腹いっぱい。
垢版 |
2017/03/18(土) 19:18:56.98ID:unMUVQ5t
>>616
地球の重さが60垓トンだから、まず君がその物質に吸われて氏んだ上から
地球やら月やら太陽やらが間髪入れず落下してくるイメージだね
0620名無しさん@お腹いっぱい。
垢版 |
2017/03/18(土) 20:39:21.63ID:z/p2/4Th
1ミリで地球よりも僅かに軽かったらシュバルツシルト半径を形成してるから地球がバラけながら吸い込まれていって最後に小さなブラックホールが出来るよ
0621名無しさん@お腹いっぱい。
垢版 |
2017/03/18(土) 21:10:10.61ID:fjPudMb2
地球の方が重いのに地球がすわれるんですか?ブラックホールはこれくらいの質量でもでかるんですか?
0622名無しさん@お腹いっぱい。
垢版 |
2017/03/18(土) 21:19:31.86ID:unMUVQ5t
>>619>>621
キミが一瞬で吸い込まれて氏ぬところまでは一緒だ。
その後1秒未満でキミのまわりの地表を引っぺがし、すさまじい衝撃波で地球を粉砕しながら落ちていく。

地球の質量なら0.9cm以下に詰め込めばブラックホールになる。
0623名無しさん@お腹いっぱい。
垢版 |
2017/03/18(土) 21:25:12.87ID:hs88w2Ea
>>621
計算上は地球の質量だとおおよそ9mmより小さい範囲に収めるとブラックホール化する事になるけど
そのサイズのブラックホールが出来るための説得力のある理論は提唱されてないはず
もっと小さな素粒子サイズのブラックホールであればビッグバン直後に
形成された可能性があるとされているものの、実在の証拠は観測されてない
0625名無しさん@お腹いっぱい。
垢版 |
2017/03/18(土) 21:48:05.34ID:fjPudMb2
宇宙のおわりについて科学者の中で一番の定説は永久に膨張なんですかね?
それともクランチ・リップ・フリーズどれかなんですかね?
0627名無しさん@お腹いっぱい。
垢版 |
2017/03/18(土) 22:19:02.69ID:6IcuynR0
>>592
理論Aに登場する空間や時間の概念
理論Bに登場する空間や時間の概念

この2つは同じではないので何も矛盾は無いです
これが同じものだとか、あいまいだとか言っちゃうのは素人だけ
0628名無しさん@お腹いっぱい。
垢版 |
2017/03/18(土) 22:22:23.68ID:6IcuynR0
そもそも空間や時間は人の脳内にしか存在しないただの概念なので
(考えるのを便利にするために用意するただの道具なので)

理論が違えば必要に応じて用意する道具も変わるのは当たり前
0629名無しさん@お腹いっぱい。
垢版 |
2017/03/18(土) 22:53:37.23ID:6IcuynR0
>>589
>ビックバンのとき光より早く膨張してたのなら光より早いものはあるんじゃないですか?

相対論だけだとそういう矛盾が出るから、別途空間が膨張すると考える
なぜ膨張するか?そんなことは知らんけど、とりあえず膨張すると考えて話を進めましょうという事

理論というのは根拠があって存在するというイメージがありますが、
なぜそうなのか?と聞かれると根本的な理由を答えられる理論は一つもないです

矛盾を解消するために根拠は無くても説明ができる考え方を用意する
理論の正体はこれです↑

今は分からなくても将来的にその根拠が見つかれば良いだけなので、理論はそういう風に作られます
というか宇宙の心理なんて知りようが無いのでそうするしかないんです
0630名無しさん@お腹いっぱい。
垢版 |
2017/03/18(土) 23:39:54.57ID:hs88w2Ea
>>624
古典物理の範囲でならある程度出来なくもないけど、
基本的には雑学としての知識だから物理に詳しい訳じゃないよ
0631名無しさん@お腹いっぱい。
垢版 |
2017/03/18(土) 23:52:02.34ID:unMUVQ5t
>>629
逆。
相対論から空間膨張が導き出される。
膨張宇宙は「ハッブルが発見する前に」相対論から導き出された一つの宇宙モデルだ。

しかしアインシュタインはあり得ないと思って相対論を一部修正した。
アインシュタイン自身が生涯最大の失敗と言った有名な話だ。
0632名無しさん@お腹いっぱい。
垢版 |
2017/03/19(日) 00:41:57.21ID:6H3/xMm7
なるほど、相対性理論が禁止してるのは光速を超えることではなく、
光速を超えて情報が伝わることだから矛盾はしてないということですかね
この掲示板で誰かに教えられたことをそのまま間違って伝えてしまったようです、申し訳ない
0634名無しさん@お腹いっぱい。
垢版 |
2017/03/19(日) 01:42:37.05ID:JIuHTqi4
>>632
いやちょっと誤解があると思う。
物が動いてるなら情報も伝わったことになるから。

相対論が禁じてるのは、光を追い越して物が動くまたは情報が伝わること。
299,792,458m/s以上で遠ざかるのは空間自体が動いているためで、
その空間に乗っかってる光はもっと速いんだから、光を追い越してるわけじゃない。
0637名無しさん@お腹いっぱい。
垢版 |
2017/03/19(日) 04:15:23.62ID:uq9T5Ya0
空間の膨張とは、時間の情報伝達の経過時間が伸びるということ。

同じ部屋内でも、情報伝達のスピードが遅くなれば、その部屋は広いと解釈される。
情報伝達とは光速や電子や素粒子のスピード。
0638名無しさん@お腹いっぱい。
垢版 |
2017/03/19(日) 04:20:31.13ID:uq9T5Ya0
光子は実際には飛んでない。
波であるから。エネルギーの振動伝達に過ぎない。

時間が伸びるということは空間が伸びるということ。
つまり光子のエネルギー伝達スピードが遅くなったということ。
遅くなるのは、エネルギーの最小単位である弦(輪)の密度が高まったから。

超弦理論
0640名無しさん@お腹いっぱい。
垢版 |
2017/03/19(日) 05:34:48.20ID:6H3/xMm7
>>634
一般相対性理論の重力場方程式には様々な厳密解があり、(シュヴァルツシルト、フリードマン、)
宇宙膨張もその内の一つだということは分かりました

でも物体が動いてなくて空間が動いてるからという説明はどうなんでしょうね?
重力で物体が動かないのか?ということになりますし

物体が動く動かないではなく、
加速するその場では光速を超えてないから問題が無い、ということですよね
遠方の異なる系にとっては光速を超えてるが、加速で光速を超えているわけではない
0641名無しさん@お腹いっぱい。
垢版 |
2017/03/19(日) 05:54:23.91ID:6vxg42ly
すみません学の無い全くの素人です。
疑問がボヤッとしてるので質問が解りづらいかも知れません。
よろしくお願いします。

1、天体が自ら発する光と反射した光とでは発した後に特徴の違いはありますか?

2、遥か遠くの天体(天体群?)(以後対象)の画像がありますが、あれは対象から放たれた(他の天体からの光を反射したもの?)光(電波)が光年分の時間を掛けて地球に辿り着いたって事で良いんですよね?
それをなんらかの技術で目視出来る画像にしてるということで良いですか?

3、球状である対象から発せられた光は進むほどに放射状に広がりますか?
もしそうなら対象が離れているほど地球を外れて進んでしまい観測出来ない可能性が出てきますか?

4、対象から光が発せられる時、対象表面の無数にある地点の一つずつから全方向に発せられるのですか?
それとも地点の一つずつから一つの方向に発せられるのですか?
または反射の場合はボールが弾むように入射角?によって発する方向が変わりますか?

5、光はどんな距離でも消失または減退?することなく進み続けますか?
天体の重力は影響しますか?

6、天体を光が掠める時、天体重力の強弱に関わらず光は曲がりますか?

7、重力により光が曲がる時、重力の強さと曲がる角度は比例しますか?
天体との距離が一定の場合

8、対象と地球の間に対象の光を遮る位置に天体が有った場合、天体の数に関係無く光が地球まで届きますか?
沢山の天体の重力の影響を受けても地球に届く光は残りますか?

9、観る(観測?)ことの出来ない天体はありますか?
ある場合どんな理由ですか?
0642名無しさん@お腹いっぱい。
垢版 |
2017/03/19(日) 05:54:57.25ID:6vxg42ly
10、宇宙は膨張していて宇宙の端?ほどそのスピードが速いと聞きました。
正しいですか?

11、10が正しいとして対象との距離は光が地球に届くまでの時間の膨張分が加味されてますか?

12、対象までの距離はいつの時点の距離になりますか?

13、膨張スピードは最も速い箇所でどの程度ですか?

14、遠くの対象ほどNASA画像などがボヤけているようですが何故ですか?

15、普通?の望遠鏡で月などを観ると近く大きく見えます。
あれは焦点を先の方へずらしてるってことですか?
もしそうなら目に届く前のずっと先の方にある光を観ているってことですか?
それは目視よりもほんの少しだけ過去の月を観ているってことですか?

16、電波望遠鏡も15の望遠鏡と同じ事が言えますか?

17、色と赤外線?と電波(他にも?)は全て光で波長が違うだけと聞きました。
離れるほど波長が長くなり、ある距離を超えると目視出来ない波長になると聞きました。
正しいですか?

18、目視出来る波長の限界は光のうちどれですか?

19、どの程度の距離までが目視出来る波長の限界ですか?

20、なんか解らなくなってきたんですけど、近いのに小さいものが見えないのは何故ですか?
遠くてもとてつもなく大きければ目視出来ますよね?
大きさに拘わらず光は発してますよね?
あれ?なんでだ?

疑問が浮かび過ぎてしまったので2レスに分けます。
これから少し寝て午後から予定があるので張り付けるのは夜か明日になってしまいますm(_ _)m
よろしくお願いします。
0643名無しさん@お腹いっぱい。
垢版 |
2017/03/19(日) 08:59:06.39ID:/+4arVUA
空間が膨張する結果、星が離れていくなら二つの可能性がある。

1. 星は空間に固定されている

2. 星は空間に固定されてはいないが星と空間の間に摩擦がある

1.の場合、星は空間の膨張する方向にしか動けない

2.の場合、慣性の法則が成り立たない
0644名無しさん@お腹いっぱい。
垢版 |
2017/03/19(日) 12:46:46.67ID:3HbsbnOE
>>643
数学で考えてみよう
x∈Z_0, y∈Z_0の2次元ベクトル空間を考える。
Z_0は初期の空間でこれは有限か無限の整数環としておきましょう。
x方向だけ空間が膨張しているとすると
単位時間tが経過するとZ_kの現在の単位元を2で割ったものを新たに単位元としてZ_{k+1}としよう(有理数からとってきてください)

さてこのx方向だけ膨張する空間上でnt時間後の星の運動は……
初期速度を(a, b)とするとy軸方向は変わらずbだけどx軸方向は
星は整数の世界しか知らないので自身はa動いているのは変わらないはずだと思っているが
神の視点というか有理数の世界の住人からみるとa*2^{-n}しか動いてないです
当然星からすればx軸方向の距離も増えてますよね
0645名無しさん@お腹いっぱい。
垢版 |
2017/03/19(日) 12:47:42.18ID:NS9+nLZH
宇宙ってなんで暗いんだろうな。くっそでかくて太陽より光ってる星なんかいっぱいあるはずなのに
0646名無しさん@お腹いっぱい。
垢版 |
2017/03/19(日) 12:55:47.76ID:dVBfOXxL
「空間の膨張」と「物体が一斉に遠ざかる」は同義
空間と物体を別々に考えるのは無意味
物体の存在が時空間を決め、時空間が物体を動かす
0648名無しさん@お腹いっぱい。
垢版 |
2017/03/19(日) 12:59:30.29ID:GR6dP18z
>>643
慣性の法則によれば、物体は外部から力が加わらない限りその場にとどまる。
「その場」自体が動くわけだから、なにも矛盾しない。

空間を物質と同じように考えるから変な考えになるんだよ。
ってか、可能性を2つしか思いつかないのが君の限界。
0649名無しさん@お腹いっぱい。
垢版 |
2017/03/19(日) 13:01:35.24ID:JIuHTqi4
>>640
物体が動いていないと説明したつもりはありません。
動く空間に乗っかる形で物体も光も情報も動きます。

物体は、遠方の異なる系にとっては光速を超えてるが、乗っかってる空間の上では光速を超えていない。
0650名無しさん@お腹いっぱい。
垢版 |
2017/03/19(日) 13:03:40.69ID:JIuHTqi4
>>641
1、ある
 物体が光を反射するとき、物質固有のスペクトルを吸収してしまうため

2、Yes

3、放射状に発せられているんだからどの方向からでも観測できる
 ただ光が弱くなりすぎたり、他の天体に隠れたり、重力レンズで見えないことはある

4、全方向に発せられる
 反射についてえは入射角と反射角の関係で見る方向によってある程度変わる。

5、基本的に中高で習った通り距離の2乗に反比例した強度で進む
 ただ宇宙規模になると
 ・宇宙膨張によるドップラー効果でエネルギーが減衰する
 ・強すぎる光子は途中でエネルギーが吸収される。
 ・星間ガスや他の天体により吸収されたり見えなくなったりする
 ・天体の重力により方向がゆがめられたリ明るく見えたり暗く見えたりする

6、7、どちらももちろん天体重力の強弱に影響を受ける

8、天体に遮られて全く見えないこともある

9、ある。3,5,8に述べた理由。
0651名無しさん@お腹いっぱい。
垢版 |
2017/03/19(日) 13:05:04.11ID:JIuHTqi4
>>642
10、Yes

11、Yes

12、3パターンある。
 ・光が発せられた時点の距離
 ・光が進んできた距離
 ・光を発した物体が今いるであろう距離

13、理論上は無限
 見える範囲ならおよそ光速の3倍

14、遠いから小さく暗くしか見えないし、それを拡大してるんだからぼやける

15、No
 大きく見えるからといって、近くに見えているわけではない

16、Yes
 電波望遠鏡だって本来目で見えない波長の光が大きく見えるだけ

17、正しいが、ある距離を超えると見えない波長になるってわけではない
 もともと波長が短い光なら可視光になる

18、赤い光。約750 nm

19、距離ではいえない。理由は17。

20、小さければ発する光が少ないため。
 人間の目に入らない、または少なすぎて人間の視細胞が反応しない。
0652名無しさん@お腹いっぱい。
垢版 |
2017/03/19(日) 13:09:52.88ID:GR6dP18z
>>641
長いのでかいつまんで回答。

>1、天体が自ら発する光と反射した光とでは発した後に特徴の違いはありますか?

なんの特徴を想定してるのか不明。
とりあえず惑星などで反射した光は恒星の光に対して圧倒的に暗い。
あと、それぞれ主に構成されている物質のスペクトラムが含まれてる。


>2、遥か遠くの天体(天体群?)(以後対象)の画像がありますが、あれは対象から

それで良いです


>3、球状である対象から発せられた光は進むほどに放射状に広がりますか?

放射状に広がります。

>もしそうなら対象が離れているほど地球を外れて進んでしまい観測出来ない可能性が出てきますか?

意味不明


>4、対象から光が発せられる時、対象表面の無数にある地点の一つずつから全方向に発せられるのですか?

全ての発光する物質から全方位に照射される。


>5、光はどんな距離でも消失または減退?することなく進み続けますか?

今のところ消失も減衰もしないと言われてる。


>6、天体を光が掠める時、天体重力の強弱に関わらず光は曲がりますか?

大なり小なり重力の影響で曲がる。
0653名無しさん@お腹いっぱい。
垢版 |
2017/03/19(日) 13:16:06.39ID:GR6dP18z
>>641
>7、重力により光が曲がる時、重力の強さと曲がる角度は比例しますか?

比例する。


>8、対象と地球の間に対象の光を遮る位置に天体が有った場合、天体の数に関係無く光が地球まで届きますか?

天体の数に関係なく、十分な質量があれば重力レンズ効果によって後背の天体が
観測される事がある。
重力レンズ効果で検索すべし。


>9、観る(観測?)ことの出来ない天体はありますか?

ブラックホールは直接観測できない。
全ての光が事象の地平面から出ていかないから。


>10、宇宙は膨張していて宇宙の端?ほどそのスピードが速いと聞きました。

基本的に正しいけど、宇宙の端を持ち出すのは間違い。
端があるかどうかはまだ不明。
遠くなるほど速い、程度に考えておくべし


>11、10が正しいとして対象との距離は光が地球に届くまでの時間の膨張分が加味されてますか?
>12、対象までの距離はいつの時点の距離になりますか?

場合による


>13、膨張スピードは最も速い箇所でどの程度ですか?

宇宙の広さが不明なので不明
0654名無しさん@お腹いっぱい。
垢版 |
2017/03/19(日) 13:20:46.03ID:GR6dP18z
>>642
>14、遠くの対象ほどNASA画像などがボヤけているようですが何故ですか?

遠いと小さくしか観測できないから。
画像を拡大していくとぼやけてしまうのと同じ。


>15、普通?の望遠鏡で月などを観ると近く大きく見えます。
あれは焦点を先の方へずらしてるってことですか?

何を言ってるのか意味不明。


>16、電波望遠鏡も15の望遠鏡と同じ事が言えますか?

質問15が意味不明なので不明。


>17、色と赤外線?と電波(他にも?)は全て光で波長が違うだけと聞きました。
離れるほど波長が長くなり、ある距離を超えると目視出来ない波長になると聞きました。

間違ってます


>18、目視出来る波長の限界は光のうちどれですか?

波長の長い方は赤外線以下、短いものは紫外線以下。
赤外線と紫外線の波長は検索して。


>19、どの程度の距離までが目視出来る波長の限界ですか?

距離と波長は無関係なので質問自体が意味不明
0655名無しさん@お腹いっぱい。
垢版 |
2017/03/19(日) 13:22:58.99ID:GR6dP18z
>>642
>20、なんか解らなくなってきたんですけど、近いのに小さいものが見えないのは何故ですか?
>遠くてもとてつもなく大きければ目視出来ますよね?
>大きさに拘わらず光は発してますよね?

目の細胞にも分解能があるから。
これは宇宙関係ない質問だな
0656名無しさん@お腹いっぱい。
垢版 |
2017/03/19(日) 13:28:41.44ID:U+7A4DcY
光が全方向に放射されていて減衰しないのなら
宇宙はもっと明るくていいんじゃないのって素人には感じる、質問者じゃないけど。
0657名無しさん@お腹いっぱい。
垢版 |
2017/03/19(日) 13:43:09.79ID:6H3/xMm7
>>649
じゃあ空間が動くとか関係ないですよね?
遠方の異なる系から見ると光速を超えてるが、加速するその場で見れば光速を超えていない、ただそれだけ

>>646
>「空間の膨張」と「物体が一斉に遠ざかる」は同義
>空間と物体を別々に考えるのは無意味

空間は人の脳内にしか存在しない、計算を便利にするためのただの概念だから当たり前ですよねそれ
0658名無しさん@お腹いっぱい。
垢版 |
2017/03/19(日) 13:51:07.42ID:6H3/xMm7
時間と距離は観測者ごと異なり信頼できない
だから、唯一信頼できる速度を基準に変形する時空を想定し、その状態を計算可能にしたのが相対性理論
0659名無しさん@お腹いっぱい。
垢版 |
2017/03/19(日) 13:56:43.02ID:JIuHTqi4
>>657
> じゃあ空間が動くとか関係ないですよね?
> 遠方の異なる系から見ると光速を超えてるが、加速するその場で見れば光速を超えていない、ただそれだけ

いや「加速するその場で」ってのが関係ない。
加速してようがしていまいが「空間が動いている」のが関係ある。
遠方の異なる系から見ると空間の移動速度が光速を超えてるが、空間に乗っかっているその場で見れば光速を超えていない
ただそれだけ。

>>658
時間と距離が観測者ごと異なるんなら、速度だって観測者ごとに異なるでしょう。
0660名無しさん@お腹いっぱい。
垢版 |
2017/03/19(日) 14:27:57.20ID:qhLZI3uY
アホな質問ばかりですいませんけどまた質問させてもらいます

地球上にいる人は限りなく光速に近いスピードで動いてるんじゃないのですか?地球は自転しててそれプラス太陽の周りを回っててさらに太陽系も銀河系の周り回っててその銀河もといふうに凄い数を重複しまくってるとおもうんですけど

この宇宙を外側からみれるひとがいたとすると人間も光の速度とほぼ変わらないくらいはやくみえないのですか?
0661名無しさん@お腹いっぱい。
垢版 |
2017/03/19(日) 14:48:59.69ID:6H3/xMm7
>>659
>空間の移動速度が光速
相対論の表現に移動する空間なんていうのは無いですよ、紛らわしいのでその説明はやめた方が良いです
相対論を知ってる人から見れば言わんとしてることが分かりますが
相対論を知らない人が見ると勘違いします
0662名無しさん@お腹いっぱい。
垢版 |
2017/03/19(日) 15:01:43.14ID:JIuHTqi4
>>660
地球の自転は赤道上でも、約0.46km/s
地球が太陽の周りを回る速度は、約28km/s
太陽系が銀河の周りを回る速度は、約217km/s
銀河も含めて我々が宇宙に対して動いている速度は、約630km/s

光の速度は、300000km/s

桁が3つぐらい違う
0663名無しさん@お腹いっぱい。
垢版 |
2017/03/19(日) 15:04:53.38ID:JIuHTqi4
>>661
「慣性系の引きずり」と言いかえればよいですか?
空間自体が動くという概念は一般相対論から導かれることです。
0664名無しさん@お腹いっぱい。
垢版 |
2017/03/19(日) 15:08:57.93ID:6H3/xMm7
離れたところから見れば動いてるように見えるだけで、近くで見れば動いてない
つまり空間は動いてない
膨張によって間の距離が伸びてるだけ
0665名無しさん@お腹いっぱい。
垢版 |
2017/03/19(日) 15:32:44.99ID:6H3/xMm7
空間が動くという話を聞いたことが無いのですが、もしかしてそういう考え方もあるのかな?
具体的にどんなのがあるか教えてもらえます?
0666名無しさん@お腹いっぱい。
垢版 |
2017/03/19(日) 15:38:36.76ID:JIuHTqi4
>>664
そんなこといえば、どの星もその上に立って見れば動いていない。

>>665
一般相対性理論では質量が動けば空間も動くと言ってる。
光速以上で離れていく空間からは光さえこちらに向かってくることはできない。
距離が離れて言ってるだけなら、こちらに向かってロケット飛ばせば帰ってくることができるはずです。
0667名無しさん@お腹いっぱい。
垢版 |
2017/03/19(日) 15:56:30.30ID:6H3/xMm7
>>666
>そんなこといえば、どの星もその上に立って見れば動いていない。

天体の運動と宇宙膨張が同じだといってます?全然違いますよね
空間が移動することの説明になってないです
0668名無しさん@お腹いっぱい。
垢版 |
2017/03/19(日) 16:01:58.07ID:6H3/xMm7
>距離が離れて言ってるだけなら、こちらに向かってロケット飛ばせば帰ってくることができるはずです。
距離と膨張率しだいで戻ってこれないケースもありますよ
0669名無しさん@お腹いっぱい。
垢版 |
2017/03/19(日) 16:11:39.79ID:6H3/xMm7
>光速以上で離れていく空間からは光さえこちらに向かってくることはできない。

この理由を知らない人のために補足しますけど、
それは速度が光速を超えてるから戻ってこれないのではなく
間の距離が次から次へと伸びて延々とゴールが遠のくから戻ってこれないだけです
0670名無しさん@お腹いっぱい。
垢版 |
2017/03/19(日) 16:29:11.68ID:6H3/xMm7
光速を超えてるために作用することが出来ないタキオンのような粒子もありますが、
それと宇宙膨張とでは、「届かない」ことの理由が違うということです
0671名無しさん@お腹いっぱい。
垢版 |
2017/03/19(日) 16:50:01.30ID:JIuHTqi4
>>667
それは何かを説明したわけじゃない。
「近くで見れば動いていない」そんなの何にだって当てはまる当たり前という指摘です。

>>668
つまり星々が離れて言ってるだけじゃなく、何かが膨張していると思ってるわけですよね?
その膨張してるのが空間という土台なんです。
全体が膨張してるということは、各部分に着目すれば動いてるということなんです。

>>669
ゴールが遠のくというのは、距離(つまり空間)が伸びていくからですね?
そして地球から見れば、光や物体は「その伸びに引きずられる」ように後退していくわけですね?
何にも引きずられなければ、1光年先から出発した光は絶対に1年で帰ってこれますので。

土台が伸びていくということは、
土台の各部分が別々の速度をもって動いてるということです。
0677名無しさん@お腹いっぱい。
垢版 |
2017/03/19(日) 17:21:17.93ID:6H3/xMm7
むしろ、間の空間が伸びるという説明は沢山見ましたが
空間が移動するという説明はめずらしいですよ・・・なんでわからないんだろう?
0678名無しさん@お腹いっぱい。
垢版 |
2017/03/19(日) 17:21:20.27ID:/+4arVUA
相対論は視点によって話が変わるからな。

例えば物質は加速されると質量が増えるというのがある。
だがそれはあくまで喩え。
加速する側から見て質量が多くなったように振る舞うだけ。
加速器で光速に近くなるほど加速される量より加速に費やすエネルギーの方が大きくなる。
これを加速される粒子の質量が大きくなったとして計算しても観測結果と齟齬がないということ。

だがより実際的に言うならこうなる。
加速する系も加速される系も慣性系も、系によって時間経過が異なる。
加速器によって光速近くに加速された系の時間経過は加速する視点から見て遅くなる。
逆に加速される視点から見るなら加速する系の時間経過は早くなる。
その結果どうなるか、加速される系に届く単位時間あたりのエネルギーは小さくなる。
逆に加速される系から届く単位時間あたりのエネルギーは大きくなる。

まあこう説明した方が分かり易いかもしれない。
光速近くで運動する物体の進行方向の空間が縮むのでなくて、系ごとの時間経過が異なるのであると。
0679名無しさん@お腹いっぱい。
垢版 |
2017/03/19(日) 17:29:21.61ID:6H3/xMm7
空間座標的には互いに移動してないが、過去と未来で系がことなり、間の距離が伸びる
この説明なら分かりますかね?

まあ、膨張の歪みを絶対的な視点から認識できるような系がかりにあるなら、
どちらも移動してるように見えるかもしれませんが
0680名無しさん@お腹いっぱい。
垢版 |
2017/03/19(日) 17:29:35.10ID:JIuHTqi4
>>677
それは部分に着目して話しているからです。
一般的に使われている言葉で知りたいのなら、さっき言ったように「慣性系の引きずり」で検索してください
0681名無しさん@お腹いっぱい。
垢版 |
2017/03/19(日) 17:38:09.25ID:6H3/xMm7
そもそも系といわれて、特定の空間座標である地球を持ち出してる時点で、何も理解してないですよね
0682名無しさん@お腹いっぱい。
垢版 |
2017/03/19(日) 17:48:03.54ID:JIuHTqi4
>>681
地球を原点とする系と言ってます。地球付近の静止系ととらえてもらいたいのですが、
宇宙の膨張を扱う規模の話でそんな注釈はつけなくても一般的には「地球」の一言で通じます。

で、あなたがまだごねているポイントは何ですか?
0684名無しさん@お腹いっぱい。
垢版 |
2017/03/19(日) 17:54:02.24ID:6H3/xMm7
宇宙が一様に膨張すると考えるならこんな感じ
空間座標的には移動してないが過去と未来で系が異なる
だから、どちらも動いてるといえば動いてるが

一般的には互いの見え方で考えるから、間の距離が延びると考える
0685名無しさん@お腹いっぱい。
垢版 |
2017/03/19(日) 17:57:58.80ID:6H3/xMm7
ブラックホールみたいに宇宙の中である範囲だけ歪んでるなら、↑の図のように見えるけど

いずれの観測者も宇宙の一部であるからして、宇宙全体の歪みを認識できる系なんて無いから、膨張が↑の図のように見えることは無い
0686名無しさん@お腹いっぱい。
垢版 |
2017/03/19(日) 17:58:11.83ID:JIuHTqi4
>>683
それってどれ?
お前の思う相対性理論て何?

>>684
その怪しいURLは何?
「このWebサイトから、このコンピュータのアプリを開けるようにしますか?」と出るんだが
0687名無しさん@お腹いっぱい。
垢版 |
2017/03/19(日) 18:10:54.12ID:JIuHTqi4
>>684-685
お絵かき見れたよ。
おまえこそ特定の座標に点打って「A系」とか書いとるやないかw
系を何だと思ってるのか説明してくれ。

なんか突然ブラックホールとか重力とか歪みとか出てきちゃってるし
もはや絵も文章もなにが言いたいかわからんくなってきてるぞ。
すこし休め。
0689名無しさん@お腹いっぱい。
垢版 |
2017/03/19(日) 18:18:19.04ID:JIuHTqi4
>>688
俺は答えた覚えがあるが、君の説明は聞いていない。

系と座標は違うって人に難癖付けたことを、
そっくりそのままやってのけるってどういうこと?

何もわかってないのは君一人だよ。
0691名無しさん@お腹いっぱい。
垢版 |
2017/03/19(日) 18:26:45.87ID:JIuHTqi4
>>690
では君のお絵かきの、黒丸書いて「A系」って書いてあるのは
どういうことか説明してもらいましょかw

ちなみに「慣性系」って「慣性座標系」のことだよ。
座標系だから当然原点とかもある。
0692名無しさん@お腹いっぱい。
垢版 |
2017/03/19(日) 18:35:34.04ID:6H3/xMm7
現実の地球やバスの中の系に原点なんて無いだろ
原点は計算する際に自由に設定できるから今は関係ない

それと、地球付近とか言ってるが
系といえば、無限遠方まで限りなく続く空間を想定するから
地球付近とか限定する必要はない
0693名無しさん@お腹いっぱい。
垢版 |
2017/03/19(日) 18:45:43.95ID:6H3/xMm7
もっと分かりやすく言うなら

A系は宇宙Aを表し無限遠方までの空間を想定する
B系は宇宙Bを表し無限遠方までの空間を想定する
A'系は宇宙A'を表し無限遠方までの空間を想定する
B'系は宇宙B'を表し無限遠方までの空間を想定する
0694名無しさん@お腹いっぱい。
垢版 |
2017/03/19(日) 18:47:56.18ID:6H3/xMm7
加速運動するバスの系から見ると世界が小さく見えるとか聞いたことないの?
それはつまりそういう事
0695名無しさん@お腹いっぱい。
垢版 |
2017/03/19(日) 18:53:49.34ID:6H3/xMm7
この問題をあなたに説明するためには、おそらく
双子のパラドックス当たりの例題で時空がどうなるのか説明する必要があるけど、
そんな気力は無いので勘弁してほしいです
0696名無しさん@お腹いっぱい。
垢版 |
2017/03/19(日) 18:56:33.30ID:JIuHTqi4
>>692
じゃあなんで君は「点」を2つ打ってA系B系を表したの?
便宜上そこを原点にした、そしてそれで通じると思ったからでしょう?

どこを原点にとってもいいけど、当然自分の居るとこを原点にしますよね。
でないと膨張で原点が動いて行ってしまうんだから扱いにくい。

>>693
宇宙4つあったの?

>>694-695
君は既にだいぶ意識が飛んでる。もう休め。
0697名無しさん@お腹いっぱい。
垢版 |
2017/03/19(日) 19:04:40.04ID:6H3/xMm7
時間と空間の長さが違うという意味では、無数に考えられる系のいずれも
同じ長さの宇宙ではないと言えますね、そしてこれは見え方の問題だけではない

あと、その図は距離感を描いた図じゃないです、過去と未来で重力によって系が変わるということを説明するための図

もし自分で勉強すなら、2つのロケット間の紐が切れるか切れないかのパラドックスがこの問題に近いのでお勧めです
0698名無しさん@お腹いっぱい。
垢版 |
2017/03/19(日) 19:22:11.19ID:JIuHTqi4
>>697
過去と未来で系が変わるってのも、系を何だと思ってるのか疑問だ。
今使ってる静止系とか加速系とかその手の「系」とは全部「座標系」のこと。
そして相対論では時間も含めた「時空間」上の座標を扱う。

あと、双子のパラドックスや2つのロケット間の紐は「特殊相対論」の範囲だ。
そこから宇宙の膨張や慣性系の引きずりを説明するのは無理がある。
0699名無しさん@お腹いっぱい。
垢版 |
2017/03/19(日) 19:27:44.51ID:6H3/xMm7
加速系が過去と未来で変わるという話がそんなに不思議ですか?
それが分からないなら、双子のパラドックスも紐のパラドックスも分からないはずですが
0700名無しさん@お腹いっぱい。
垢版 |
2017/03/19(日) 19:56:47.55ID:JIuHTqi4
>>699
座標系ってのは、いってしまえば誰かが計算のために勝手に引いた座標を表すものでしかないんです。
そして特定の特徴を持つ座標系を「静止系」とか「加速系」とか言うんです。
座標系が過去と未来で変わるって?それはあなたが2つの異なる座標系を作ったからにすぎない。

特殊相対論で「座標系を乗り換える」とかいうけど、
あれは計算のために2つの慣性座標系を用意し、カットしてつなぎ合わせるという意味です。
物体が物理的な何かを乗り換えたわけではないことはお分かりと思うが。
0702名無しさん@お腹いっぱい。
垢版 |
2017/03/19(日) 20:07:10.54ID:6H3/xMm7
>>700
ずっとそれを言ってるんですが?
互いが移動せずとも過去から未来へと系の乗り換えが行われてるから間の距離が延びる(←宇宙膨張)
カットしてつなぎ合わせる?←この部分だけはちょっと何言ってるか分かりません

そんなにおかしいというなら、あなたも相対論でなぜ宇宙が膨張するのか説明してもらえますか?
0703名無しさん@お腹いっぱい。
垢版 |
2017/03/19(日) 20:35:40.90ID:JIuHTqi4
>>702
> 互いが移動せずとも過去から未来へと系の乗り換えが行われてるから間の距離が延びる

もう一回繰り返しますよ。
系とは「座標系」、それはあなた(もしくは私)が勝手に引いた頭の名もしくは紙の上の線です。
座標系の乗り換えが行われるのは、あなた(もしくは私)の頭の中です。
実際の物体は何も乗り換えませんよ。

頭の中で座標系を切り換えたら実際の距離が伸びるんですか?
そう思ってるならご愁傷様。
0704名無しさん@お腹いっぱい。
垢版 |
2017/03/19(日) 20:40:21.02ID:6H3/xMm7
>>703
重力で系の乗り換えが行われる以外に何があるんですか?
ブラックホール付近と遠方では収縮率が異なるのは知ってますよね?
ただそれだけの話です、妄想?ではありませんよ

面倒ならいつでも終わってもらって結構ですが、
おかしいというなら、なぜ宇宙が膨張するのか相対論的に詳しく説明してもらえます?
そちらが言ってることが正しそうならそちらを信じますよ
0705名無しさん@お腹いっぱい。
垢版 |
2017/03/19(日) 20:43:26.61ID:6H3/xMm7
あなたは膨張で光速を超えることが矛盾しないという説明しかしてない、
膨張は相対論で予言されてたことだと言いますが、じゃあその膨張は
相対論的にどういうメカニズムなのか説明お願いします
0707名無しさん@お腹いっぱい。
垢版 |
2017/03/19(日) 21:30:45.28ID:JIuHTqi4
>>704
> 重力で系の乗り換えが行われる以外に何があるんですか?

もう一回繰り返しますよ。
重力だろうが何だろうが
系の乗り換えというのは、我々の思考の中で行われてる数学上の計算作業にすぎません。
実際の宇宙に座標系という物理的存在はありませんし、実在の物体がそれを乗り換えるわけでもありません。
あくまで数学上の道具にすぎないんです。

ブラックホール付近と遠方では収縮率が異なる?
「シュヴァルツシルト座標系で」計算すると近傍ほど「収縮することになる」のは知ってますよ。

なぜ宇宙が膨張するのか?
それは現時点で世界の誰も知らないんじゃないでしょうか?
相対論的には膨張している状態が計算できているにすぎません。
0709名無しさん@お腹いっぱい。
垢版 |
2017/03/19(日) 21:44:16.01ID:6H3/xMm7
>相対論的には膨張している状態が計算できているにすぎません。

いやだからそれを合理的に説明しろと言ってるんだが?
空間も時間も座標系も頭の中にしかないというのは、最初にオレが言ったセリフだよ

>なぜ宇宙が膨張するのか?
>それは現時点で世界の誰も知らないんじゃないでしょうか?
相対論でそれが予言されてたとオレに教えてくれたのはあなただろ(笑)
つまり、膨張には相対論的な理由があるはずなんだよ
それが本当かどうかは今は考えなくていいから、相対論的なメカニズムを説明して
0711名無しさん@お腹いっぱい。
垢版 |
2017/03/19(日) 22:06:43.00ID:JIuHTqi4
>>709
「フリードマン方程式」「フリードマン・ルメートルモデル」でググレカス。

> 空間も時間も座標系も頭の中にしかないというのは、最初にオレが言ったセリフだよ
どこで?
過去から未来へと系の乗り換えが行われてるから間の距離が延びるとか
意味の解らんことを言ってたのによういうわ。

> 相対論でそれが予言されてたとオレに教えてくれたのはあなただろ(笑)
> つまり、膨張には相対論的な理由があるはずなんだよ
宇宙の膨張に相対的論的な理由なんてものはない。
相対論では宇宙は定常的に存在できないことが予言されていた。
だったら収縮してるか膨張してるかしかないだろうよ。
0712名無しさん@お腹いっぱい。
垢版 |
2017/03/19(日) 22:11:04.12ID:JIuHTqi4
>>708>>710
私は物理学科卒ですが、専門家ではありません。
まったく関係のない職業です。

2chで出る質問などあんまりバリエーションがなく、
レベルも低いし、多少煽っても大丈夫なとこなので
調べながらなんだかんだ遊んでいるうちに対応できるようになります。
0713名無しさん@お腹いっぱい。
垢版 |
2017/03/19(日) 22:13:22.31ID:6H3/xMm7
物理学科卒業なのに説明できないんですか?
ちょっとググって出てきたヤフー知恵袋とかの方が短くて分かりやすい説明してる人多いですよ
つまり、あなたはそれを知らずに相手を否定していたということですね
0714名無しさん@お腹いっぱい。
垢版 |
2017/03/19(日) 22:18:37.80ID:JIuHTqi4
>>713
何を説明でいないって?
世界中の誰も知らんことをわかるわけないでしょうが。

> ちょっとググって出てきたヤフー知恵袋とかの方が短くて分かりやすい説明してる人多いですよ

では、ちょっとググって出てきたヤフー知恵袋とかで短くて分かりやすい説明聞いた結果
ここでずっと頓珍漢なこと言い続けてる理由を聞かせてくれ。
0716名無しさん@お腹いっぱい。
垢版 |
2017/03/19(日) 22:26:54.95ID:6H3/xMm7
空間が伸びるとか膨張とかはよく聞くけど、
彼の言う空間が移動するというのがどういう意味か分からん
0717名無しさん@お腹いっぱい。
垢版 |
2017/03/19(日) 22:31:27.36ID:MXYMgVhY
>>710
たぶん>>708のいう「こんなになれるの?」っていうのは、日曜日とはいえ昼から
延々と半日近く質問スレで質問と関係ない議論を続けるくらいのキチガイに
なっちゃうの?って話だと思うよ。
0719名無しさん@お腹いっぱい。
垢版 |
2017/03/19(日) 22:32:22.77ID:dz2v06Co
熱的死したエネルギーの最終形態がダークエネルギーではないの?
だからダークエネルギー増加で空間が加速膨張していると。





>エネルギー保存の法則が成立したと仮定し、宇宙がエネルギーの総量が一定の閉鎖系だと仮定すると、
"質"の良い(エントロピーの小さい)エネルギーは時間とともに減少していくことになる。エントロピーは局所的には増加も減少もするが、宇宙全体としては常に増加している(→熱力学第二法則)。
エントロピーの増加はエネルギーの"質"(そのエネルギーからどれだけ仕事を得られるか)の低下を意味し、
宇宙に存在する全てのエネルギーは最終的にはもはや外部に対して何ら仕事をすることができない均一な熱エネルギーとなる(熱的死)。
0720名無しさん@お腹いっぱい。
垢版 |
2017/03/19(日) 22:32:40.08ID:6H3/xMm7
伸び縮みは相対論で説明できるけど、
座標が飛び越えることはどうやって説明するの?

それって単に、移動する物体に原点設定してるだけじゃないの?
0723名無しさん@お腹いっぱい。
垢版 |
2017/03/19(日) 22:35:08.78ID:dz2v06Co
宇宙空間を飛び交う光の光子と光子が衝突することはあるのですか?
衝突するとどうなりますか?
0724名無しさん@お腹いっぱい。
垢版 |
2017/03/19(日) 22:43:18.27ID:JIuHTqi4
>>717
私はこんなことやってるのは年に1、2回だ。
宇宙にも興味あるので、趣味で周辺知識を調べる勉強がてら。
ほとんどの日は2chに出入りすらせんよ。

休みは1日中ゲームやってる時も、1日中ぶらぶらしてる時も、1日中釣りしてる時もある。
それなりに年を重ねると人に会うのも億劫になる。
たまにはこんな1日の使いかたもいいだろう。
0725名無しさん@お腹いっぱい。
垢版 |
2017/03/19(日) 22:47:22.49ID:6H3/xMm7
>>724
異なる系から見て伸び縮みする現象を空間が移動すると言っちゃうキチガイが何言ってるの
人の話聞いてないのはあなただよ
0728名無しさん@お腹いっぱい。
垢版 |
2017/03/19(日) 22:53:58.03ID:6H3/xMm7
空間が移動するとはどういう意味で言ってるのか、教えてと言ってるだけなのですがね?
そんなに難しいこと聞いてますかね??
0729名無しさん@お腹いっぱい。
垢版 |
2017/03/19(日) 22:55:08.30ID:A+k0i4Mm
>>639
遠くの初期銀河は周辺のダークマターが近隣の銀河より薄く広がっていたために
周縁部分では回転が遅くなってるみたいだね
0730名無しさん@お腹いっぱい。
垢版 |
2017/03/19(日) 22:57:18.89ID:6H3/xMm7
もしかして、これのことを空間の移動と言ってます?

これは移動じゃなくて伸びてるだけですよ
0731名無しさん@お腹いっぱい。
垢版 |
2017/03/19(日) 22:59:25.12ID:JIuHTqi4
>>727
知ってて同じことを聞いてるんですか?
毎回やってるのにそれですか。

>>728
質問には1回は全部答えてますよ。
あなたは私の質問全部スルーしてますけどね
0732名無しさん@お腹いっぱい。
垢版 |
2017/03/19(日) 23:04:18.20ID:JIuHTqi4
>>730
それ、明らかに右の四角は移動してるように見えますね
移動してないの?
それが移動してないように見えるの?目か頭大丈夫?
0735名無しさん@お腹いっぱい。
垢版 |
2017/03/19(日) 23:06:26.30ID:6H3/xMm7
空間のある一点に物体を設置して動きを見れば移動してるように見えるだけだろそれw
空間は移動してないよwwwwwwwwwwwwwww
0736名無しさん@お腹いっぱい。
垢版 |
2017/03/19(日) 23:07:44.18ID:dz2v06Co
>>729
ああなるほど!
遠いってことは過去の姿の銀河だから、昔はダークマターが銀河に濃くなかったってことか!
やっぱ時間かかってるってことは重力で吸い寄せられてんのかな
0738名無しさん@お腹いっぱい。
垢版 |
2017/03/19(日) 23:09:22.93ID:JIuHTqi4
>>734
移動してないとしたら静止してるんか?
違うでしょ、明らかに移動してるよね。

試しに「増えるワカメ」全く動かさずに膨張させてみてや
できるもんなら
0739名無しさん@お腹いっぱい。
垢版 |
2017/03/19(日) 23:10:02.15ID:6H3/xMm7
物理学科卒業でここまで頭が悪いとは、信じがたい
さんざんこっちの間違いを指摘し続けてそれかよ
0742名無しさん@お腹いっぱい。
垢版 |
2017/03/19(日) 23:15:19.59ID:JIuHTqi4
>>740
自分で空間とはっきり書いてますよね?

中心の位置が明らかに右に動いてる。
最初に右端があった場所よりさらに右に動いてるのに動いてないとは
あたまの残念な子だ。
0743名無しさん@お腹いっぱい。
垢版 |
2017/03/19(日) 23:17:07.64ID:6H3/xMm7
お前の間違えを知っててそれを気づきやすいずにしてるだけだよwwww

移動するのは空間じゃなくて物体だからね?OKですか?wwww
0744名無しさん@お腹いっぱい。
垢版 |
2017/03/19(日) 23:18:22.11ID:6H3/xMm7
物体は移動してないから矛盾しないとかも言ってなかった?www
いやいや物体は移動してますよwww
情報の伝達が光速を超えないから矛盾にならないだけw
0746名無しさん@お腹いっぱい。
垢版 |
2017/03/19(日) 23:22:06.47ID:JIuHTqi4
>>743
くどい
一般相対論の宇宙モデルでは、物体とともに慣性系が引きずられて動くるといっておろうが
いい加減ググれや

>>744
「物体はその空間の上で光速を超えて移動していない」な
適当に省略してねつ造すんな、人間のカス
そんなに負けを認めるのが嫌か?
0747名無しさん@お腹いっぱい。
垢版 |
2017/03/19(日) 23:24:11.18ID:6H3/xMm7
>物体とともに慣性系が引きずられて動く
それ当たり前なんだけど?何が言いたいのかな?w

しかもそれ、物体移動してますやん
物体は移動してない、空間が移動してるとか言ってたのはどうなったの?w
0748名無しさん@お腹いっぱい。
垢版 |
2017/03/19(日) 23:28:08.36ID:JIuHTqi4
>>747
> それ当たり前なんだけど?何が言いたいのかな?w
当たり前?
一般相対論で説かれ、近年まで実験レベルで検証できなかった事実が?

> 物体は移動してない、空間が移動してるとか言ってたのはどうなったの?w
きみは記憶障害?
まず1レス前を見ろw
0749名無しさん@お腹いっぱい。
垢版 |
2017/03/19(日) 23:30:47.38ID:6H3/xMm7
遠方から見た物体は光速を超えてるけど、
それは移動する空間に乗ってるからというのが、彼の主張
だけどそうじゃない

空間に乗ってるとか関係なく、移動してるのは空間じゃなくて物体だし
実際に光速を超えている
0750名無しさん@お腹いっぱい。
垢版 |
2017/03/19(日) 23:33:07.21ID:6H3/xMm7
問題は空間どうのこうのじゃないってこと、分かるかな?

遠方から見ても近くで見ても、いずれの場合も物体が光速を超えることがないから
矛盾にならないだけ
0752名無しさん@お腹いっぱい。
垢版 |
2017/03/19(日) 23:34:54.48ID:JIuHTqi4
>>749
つまり一般相対論の宇宙モデルは間違っていると?
宇宙はほぼ平坦らしいのに、この理論なくして光速を超えちゃってたら特殊相対論とも矛盾するんだが。
0753名無しさん@お腹いっぱい。
垢版 |
2017/03/19(日) 23:36:00.21ID:6H3/xMm7
物体は実際に移動して光速を超えるけど、
その超える理由が違うから矛盾にならないだけ
そんなことも分からんのか
0758名無しさん@お腹いっぱい。
垢版 |
2017/03/19(日) 23:43:06.12ID:6H3/xMm7
物体と光が一緒に伸びるから光を超えることが無いってだけ

空間が移動するとかキチガイな発想は要らない
0760名無しさん@お腹いっぱい。
垢版 |
2017/03/19(日) 23:52:09.05ID:6H3/xMm7
すみませんでした^^;

でも、さすがに彼も理解したでしょうからもうスレを荒らす必要なくなると思います
0761名無しさん@お腹いっぱい。
垢版 |
2017/03/19(日) 23:57:41.79ID:JIuHTqi4
>>756
無理クソだな
こっちのツッコミは完全スルーして、同意したことにしてしまうなんてヤクザしか通用せんぞ

>>758
一般相対論を否定すんの?
光が伸びるのはいいとして、お前ん中では物体も伸びるの?

こちらも、バカの妄想に付き合うほど好奇心旺盛じゃないのでね。

>>760
ほんとおまえみたいなのって例外なく
ボロクソに言いくるめられてても無視して、勝ち宣言だけしたがるよな
0762名無しさん@お腹いっぱい。
垢版 |
2017/03/19(日) 23:58:30.24ID:A+k0i4Mm
>>736
遠方銀河はダークマターのむらが十分成長する前にガスがダークマターの作る重力ポテンシャルの穴に落ち込んで
銀河が作られてる
近隣銀河ではダークマターのむらが成長してるところで作られてるので銀河周縁部でもダークマターが詰まってる
そのために速度が落ちない
0763名無しさん@お腹いっぱい。
垢版 |
2017/03/20(月) 00:00:32.06ID:w9uBS6H2
ちなみに、中身の物体は空間の膨張に取り残される感じで
天体とかなら、中央に縮もうとする力が働くそうです
空間と中身が一緒に膨張しちゃったら宇宙膨張なんて感じることすらできないので
0764名無しさん@お腹いっぱい。
垢版 |
2017/03/20(月) 00:04:03.64ID:w9uBS6H2
>>761
自分が正しいと疑いもせずに相手を煽りつづけたあなたの自業自得ですよ
こちらは、最初に紳士的に謝罪して終わらそうとしたのに蒸し返すでしょ?
ああ、こいつはキチガイだなと、その時点で切り替えましたよこちらも
0765名無しさん@お腹いっぱい。
垢版 |
2017/03/20(月) 00:14:29.81ID:xu2UeICK
>>764
こちらも最初は敬語で紳士的な対応してましたよね?
答えた話を蒸し返し続けて終わらそうとしてないのはあなたでしょう。
私はあなたのレスがあるまでは追いレスは書き込んでないですよ。
そしてすべてあなたの質問に答えてきた。

自業自得の意味わかってますか?
ヤクザが暴れまわった後、自分を正当化するためのセリフじゃないんですよ。
こちらも途中からあなたをキチガイと見ています。
0766名無しさん@お腹いっぱい。
垢版 |
2017/03/20(月) 00:20:47.84ID:w9uBS6H2
言いくるめられなければいいじゃないですか?
気に入らないならまだ付き合いますが、何が問題なんです?
0767名無しさん@お腹いっぱい。
垢版 |
2017/03/20(月) 00:22:16.85ID:w9uBS6H2
ちなみに、あなたが間違ってるとは思ってませんよ?
概念が整理できてないので、十分に身についてなくて
説明が下手、あるいは的をはずれているとは思いますが
0768名無しさん@お腹いっぱい。
垢版 |
2017/03/20(月) 00:31:19.03ID:xu2UeICK
>>766
> 言いくるめられなければいいじゃないですか?

えっ?どういうこと?
あたたが私を言いくるめてる状況と思ってんの?
こっちの指摘や返答一切無視して、意味なくひたすら喚き散らしてただけに見えるが

おまえに勉強する気がなく聞く気もない、妄想まき散らしたいだけなら
もうこちらも続ける気はないよ。

>>767
あなたは基本的な相対論の概念をまだまだ理解していない、
あるいは間違って理解しているとは感じます。
0769名無しさん@お腹いっぱい。
垢版 |
2017/03/20(月) 00:38:09.72ID:w9uBS6H2
>あたたが私を言いくるめてる状況と思ってんの?
それあなたが自分で言ったことですよ

>>761
>ボロクソに言いくるめられてても無視して、勝ち宣言だけしたがるよな

煽りは要らないから何が問題なのか話を進めてくれるかな?
0770名無しさん@お腹いっぱい。
垢版 |
2017/03/20(月) 00:39:19.39ID:w9uBS6H2
ああ、こっちが言いくるめられてるって意味でしたかw
勘違いwwww

ものすごくポジティブなんですねwww

何度も他の片から間違いを指摘されてるのにw
0771名無しさん@お腹いっぱい。
垢版 |
2017/03/20(月) 00:44:01.71ID:xu2UeICK
>>770
妄想癖がある方は強いですね。

どう考えても言い負けてるのはあなたです。
私はあなたの疑問にすべて返してると思いますが、
あなたは私のツッコミに何か反応できてましたか?

多く喚き散らした方が勝ちというルールはないですよw
0773名無しさん@お腹いっぱい。
垢版 |
2017/03/20(月) 00:55:22.98ID:xu2UeICK
私はこれまで相対論ではどう考えられるという説明をしてきたが
あなたが相対論を否定したい、相対論に独自理論を持ち込みたいというスタンスなら、
それに対して議論したいとは思わない。

あなたの何が間違ってるって、ところどころあるので指摘してきましたが。
もしかして私の書いたレス、議論の流れ、覚えてないのですか?
0776名無しさん@お腹いっぱい。
垢版 |
2017/03/20(月) 00:59:03.78ID:w9uBS6H2
あとは、空間が移動するとか言っちゃう当たりが
誰から見ても発言そのものが明らかにおかしい
0777名無しさん@お腹いっぱい。
垢版 |
2017/03/20(月) 01:23:51.11ID:xu2UeICK
>>774
その説明は>>682で回答済みです。
あなたの図>>684も特定の座標に黒丸を打って座標系と表現しています。
簡略した表現としては間違いではないんですが、
私がやるのは間違いで、自分でやるのはよいというのは、フェアではありませんね。

さらに貴方の系に会する概念>>693
系が4つあったとき、4つの宇宙を書いていますね。
また>>697で「過去と未来で重力によって系が変わる」という謎の表現をしています。
それを受けて、私が>>698-707で、「系」についての詳細な概念を説明しています。

それでもあなたは、いまだに自分が言い負かされていることに気づいていない。
0778名無しさん@お腹いっぱい。
垢版 |
2017/03/20(月) 01:24:36.04ID:xu2UeICK
>>775
どれも非常に具体的な指摘でしたよ。
あなたにはまるで見えてないようでしたが。

>>776
表現が気に食わないなら「慣性系の引きずり」と言いかえると言いましたよね?
それは一般相対論で使われている一般的な言葉です。
砕けた表現をすれば、まさに空間が引きずられて動くんです。
0780名無しさん@お腹いっぱい。
垢版 |
2017/03/20(月) 01:39:43.00ID:xu2UeICK
>>779
具体的な指摘をしてもらえますか?
地球を原点とする座標系を考えてほしいとき、「地球」と簡略表現することはよくあります。

しかしあなたの図>>684はなんとも説明が付きません。
あなたはググっても自分がおかしいことすらまったく気づかないでしょう。
そのレベルだと思います。
0782名無しさん@お腹いっぱい。
垢版 |
2017/03/20(月) 01:46:40.48ID:xu2UeICK
>>781
地球を原点とする座標の中に、地球という原点座標があるんです。

それをいうなら何度も言うように、座標と座標系の区別が全くついていないあなたの図>>684の解説こそ必要です。
またスルーですか?
0783名無しさん@お腹いっぱい。
垢版 |
2017/03/20(月) 01:49:11.00ID:xu2UeICK
まちがい、

>>781
座標系の一つとして、慣性座標系というものがあるんです。
前も説明しましたよね?聞いてました?
0784名無しさん@お腹いっぱい。
垢版 |
2017/03/20(月) 01:52:04.57ID:w9uBS6H2
>>676
>ガリレイの相対性原理から始めないと
初歩中の初歩すら勘違いしてると、他の人からも言われてるの気づいてる?

>>684
その説明しても君じゃ理解できない
0785名無しさん@お腹いっぱい。
垢版 |
2017/03/20(月) 01:56:27.29ID:w9uBS6H2
はあ・・・慣性系なんていう初歩をオレが説明するのか、めんどくせー
まじでなんなのおまえ・・・
0786名無しさん@お腹いっぱい。
垢版 |
2017/03/20(月) 01:58:06.41ID:w9uBS6H2
この図を見てもらえば分かるが

ただの座標系じゃ、↑の図の重なりを表現できないだろ?
つまり慣性系はそういうことじゃない
0787名無しさん@お腹いっぱい。
垢版 |
2017/03/20(月) 02:14:12.13ID:xu2UeICK
>>784
あなた以外、誰がそんなこと言ってましたか?
ご自分の妄想癖には気づいていますか?

>>785
もしかして、わざとやってる?
「バスの中の系」とか言ってるあたり、
きみはひょっとして太陽系とかの「系」の意味とごっちゃになってるんじゃないかと思うよ。

これ以上の議論は不毛なのでお開きにしていいだろうか?
0790名無しさん@お腹いっぱい。
垢版 |
2017/03/20(月) 02:32:20.23ID:xu2UeICK
>>788
そのレスにはレスアンカーついてませんね。
誰にどんな意図で言ったのかわからないと思いますが、
あなたは全部自分に都合の良い方向に解釈するんですね。便利な思考回路をお持ちだ。

「空間の引きずり」という概念。
知らない人には一見、相対性原理と反する過去のエーテル論のように見えます。
そのため知らない人には一時的に疑問に思われることがあります。

その後私の話を聞いていて、間違っていると思うならすぐに傍観者から反論が付くはずです。
そういのみんな大好きですからね。
でも静観している。どうしてかわかりますよね。
0795名無しさん@お腹いっぱい。
垢版 |
2017/03/20(月) 02:46:01.89ID:w9uBS6H2
バスの中で観測者が違ったら系も違うとか言いそうだな、あいつ・・・
原点を設定するとか言ってたし
0800名無しさん@お腹いっぱい。
垢版 |
2017/03/20(月) 07:26:30.13ID:6dnUMoDu
>>797
わかんね。極限まで潰れたコアって有るんかな。
1mmより小さいんかな。
中心は重力無限大とか聞いてたが、そうなったら、
重力だけの場なんかな
0801名無しさん@お腹いっぱい。
垢版 |
2017/03/20(月) 10:55:42.02ID:w9uBS6H2
慣性系A:地上と一緒に動く世界の系、無限遠方まで続く空間を想定する
慣性系B:バスと一緒に動く世界の系、無限遠方まで続く空間を想定する
(一緒に動くという意味で、◯◯から見た静止系とも言う)

観測者A1と観測者A2はどちらも慣性系A、ただし相対論の計算をする際に原点の位置が違ってくる
観測者B1と観測者B2はどちらも慣性系B、ただし相対論の計算をする際に原点の位置が違ってくる

ID:xu2UeICK が言ってるのは相対論の計算をする際の原点の取り方、つまり観測者の違いを言ってるだけなので
それは、ただの座標系であり、慣性系ではない
系と観測者がごっちゃになるのは、相対論入門者によくある間違え、本当に物理学科出身なんだろうか?この人は


慣性系は慣性の法則が成り立つ系
(加速も減速もせず一定の速度で曲がらず真っ直ぐ進む系)

方向が途中で変わる場合は、必ず加速が必要になるので
その場合は、慣性の法則が成り立たない系になる(非慣性系)

仮にバスが等速直線運動してない場合は、加速や減速をしているということなので、
慣性の法則が成り立たない系になる(非慣性系)

※地上やバスは厳密にいえば慣性系ではないけど、今は説明のためにそこは省略
0802名無しさん@お腹いっぱい。
垢版 |
2017/03/20(月) 11:12:13.10ID:w9uBS6H2
ちなみに、加速によって速度が増加する場合、速度が違えばそれば別の系ということになる
(系を乗り換えるともいう)

観測者と系を区別することは加速を扱う問題を考えるときにとても重要になる
ID:xu2UeICK は双子や紐のパラドックを知ってる風な口ぶりだったけど、理解しているとは思えない
0806名無しさん@お腹いっぱい。
垢版 |
2017/03/20(月) 12:41:05.65ID:dtbkmhPA
>>804
相対論語ってるんなら速度とはなんぞやということ教えてくれ。
理論としての相対論はあくまで計算結果と観測結果が一致するというだけだ。
実際に世界がアインシュタインの説明の通りになってるというわけじゃない。
とりあえず分かるのは世界を精度良くシミュレートする理論ということだけ。
理論の話になると難しいから、言葉として「速度が増加する」の速度を定義してみれ。
0808名無しさん@お腹いっぱい。
垢版 |
2017/03/20(月) 12:45:48.26ID:w9uBS6H2
その前にまず

>理論としての相対論はあくまで計算結果と観測結果が一致するというだけだ。

これがどういう意味で言ってるのか説明してくれ
計算結果?は何が言いたのか大体わかる
けど、観測結果は何を意味してる?
0810名無しさん@お腹いっぱい。
垢版 |
2017/03/20(月) 13:36:43.53ID:w9uBS6H2
>理論 >だけだ
とか言うあたりからして理論の存在理由を勘違いしてそう
理論は根拠があって証明されて存在するものではないよ?
そう考えるのが最も合理的でかつ、実用的な計算方法が得られればそれで十分なんだよ
だけと言えばそうだけども、そんなことを言えば他の理論だってすべて、ただそれだけってことになる


>理論としての相対論はあくまで計算結果と観測結果が一致するというだけだ。
これはどういう意味で言ってる?
0811名無しさん@お腹いっぱい。
垢版 |
2017/03/20(月) 13:48:10.50ID:w9uBS6H2
物理量としての速度、
絶対、相対の速度
概念としての速度、

速度を定義しろと言われても、速度の何について話したいのかが分からん
速度の概念が一つしかないと勘違いしてないか?
0812名無しさん@お腹いっぱい。
垢版 |
2017/03/20(月) 13:50:26.88ID:w9uBS6H2
時間や空間の概念だっていろいろあって理論毎に違うだろ

>定義
とかいうあたりからして
一つの言葉が一つの概念しか持たないという初心者的な発想をしてるようにしか見えない
0813名無しさん@お腹いっぱい。
垢版 |
2017/03/20(月) 13:53:19.18ID:dtbkmhPA
理論には数式と能書きがあるだろ。
数式が現実世界をうまくシミュレートしても何故なのかを説明しないとならないよな?
数式が合ってるならそれは実際に上手くシミュレートできる数式でいい。
だけど能書きの方はそうとは限らないだろ?
数式の方は議論の余地なし、でも説明の方はある。
だから相対論を語る人に聞きたいわけだ。
速度が増加するというのはどの視点の話?
0814名無しさん@お腹いっぱい。
垢版 |
2017/03/20(月) 13:58:12.00ID:w9uBS6H2
数式と能書きは違うという事かな?

それも初心者的な勘違いですね
概念の視点で見ればどちらも同じで、その二つに違いは無い

例えば、意味もなくあなたは三角関数を使いますか?使わないよね
概念が言葉の形をとるのか、概念が数字の形をとるのか
ただそれだけしか違いは無い
0816名無しさん@お腹いっぱい。
垢版 |
2017/03/20(月) 14:13:23.64ID:w9uBS6H2
おそらく相対論を否定したいのかな?

ちなみにね、相対論の理解者のほとんどは相対論を嫌っている
あるいは、もともと嫌っていた人達なんですよ
ムカつくけど、ほかに合理的な説明が見当たらないから相対論を仕方なく使ってる人がほとんどです
0817名無しさん@お腹いっぱい。
垢版 |
2017/03/20(月) 14:16:28.11ID:w9uBS6H2
人ならだれでも成長するにつれて見につく常識や概念
その普通の考え方から見れば相対論はただの矛盾のかたまりでしかないので
誰だって受け入れがたいのは同じです

自分の常識や概念をねじ曲げるだけの理由がそこにあり、
仕方なく受け入れることで初めて理解できるようになる理論ですね
0818名無しさん@お腹いっぱい。
垢版 |
2017/03/20(月) 14:23:49.81ID:w9uBS6H2
ただしそれは、相対論を使いたい人だけがすればいいことなので

相対論を使うときはねじ曲げた方の概念を使い
他の理論を使うときはねじ曲げる前の概念を使う感じで
使い分ければよいだけなのです

(理論毎に同じ言葉でも概念が違う可能性がある)
0819名無しさん@お腹いっぱい。
垢版 |
2017/03/20(月) 14:34:16.99ID:w9uBS6H2
理論は正しい正しくないではなく、

プラスネジにはプラスドライバーを
マイナスネジにはマイナスドライバーを使うのと同じで、
何が正しいかではなく、何が便利か、合理的か、納得できるか、という基準で存在しています

なので、相対論さんは私は正しいですよなんて言ってないので、
使いたくないなら無理に理解する必要はありません
理解するためには自分の概念をねじ曲げる必要があり、
それでもどこかに矛盾を感じるなら、概念のねじ曲げ方が足りないということです
彼は自分が正しいとは言ってないので、自分の概念をねじ曲げるかどうかはあなた次第

じゃあ、相対論は正しくないのか?と聞かれると少し違う
大勢の人がそれを利用してるとすれば、それはもう正しいと言っても差支えが無いということです
0820名無しさん@お腹いっぱい。
垢版 |
2017/03/20(月) 14:52:52.88ID:w9uBS6H2
余談ですが、正しいとは何か、これを考えることは宇宙に関係が無さそうで意外と関係が深いです

結論から言うと、正しさそれは

A と B が同じであるとき→正しい
A と B が同じじゃない時→正しくない

ただそれだけの概念なんです
ここでいうAとBというのは片方は自分の常識や概念で、もう片方は自分が見る世界です

つまり、同じ現象を見ても人の概念(解釈)が違えば正しさは揺らぐということです
なので、正しさは幻のようなものですね

「それは正しくはない!」というのはよく見かける光景ですが、
正しさを知ってる人からすれば「う、うん、そうだね、で、それが何?」となるわけです
0821名無しさん@お腹いっぱい。
垢版 |
2017/03/20(月) 15:01:18.28ID:dtbkmhPA
下らないゴタクはもういい

「ちなみに、加速によって速度が増加する場合、速度が違えばそれば別の系ということになる」

この場合の速度とは誰からみた速度?
0823名無しさん@お腹いっぱい。
垢版 |
2017/03/20(月) 15:24:02.80ID:w9uBS6H2
加速するバスの中から見てもいいし、加速するバスを見つめる地上から見てもいいし、
なんなら、地球の外から見てもいい、どういう意味ですか?
0825名無しさん@お腹いっぱい。
垢版 |
2017/03/20(月) 15:30:33.50ID:xu2UeICK
うわ、気持ちわるいw
案の定、収まりつかなくて独演会、たまに人きたら延々と絡んどるんかいな
長文で自己満講義したいなら他所に行かんか?

いちおう1レスだけしといてやるわ
0826名無しさん@お腹いっぱい。
垢版 |
2017/03/20(月) 15:32:11.69ID:xu2UeICK
>>801
慣性系はもともと「慣性座標系」を省略したもの、つまり座標系。
いい加減理解できてくれ。

あときみレベルはすぐに座標系と観測者を結び付けたがるが、座標系に観測者なんて本来ないぞ。
高校で慣性系は習っても観測者なんて出てこないのはそういうこと。
緯度経度だって座標系の一つだが、観測者なんて知ったこっちゃないでしょう。

>>802
加速度系も知らんのか

>>804
啓蒙書で特殊相対論をかじった高校生にしか見えとらんよ

>>805
これは昨晩お開きにしたと思ったのだが?
もう宇宙板に書き込む必要ないのでは?
0828名無しさん@お腹いっぱい。
垢版 |
2017/03/20(月) 15:38:40.44ID:w9uBS6H2
>>826
慣性系に原点や観測者がいらないというのはオレが最初から君に説明してる事なんだけどw
何でオレがそれを説明されてる側になってるの?

系といわれて、原点とか、地球から見たとか、答えてたのはあんただよwwwwキチガイか?
0830名無しさん@お腹いっぱい。
垢版 |
2017/03/20(月) 15:45:47.80ID:xu2UeICK
>>828
うそこけ、延々と頓珍漢な事ばかり言ってたくせに
座標系なんだから観測者はいらんが、原点もあるし、どっかの座標に地球もあるわ
0831名無しさん@お腹いっぱい。
垢版 |
2017/03/20(月) 15:47:29.20ID:w9uBS6H2
慣性系は君の言うように座標系の一つだけど、

原点や観測やは自由に設定できるから、慣性系はどんな概念?と聞かれて
原点や地球を上げる必要はないwwwww
0832名無しさん@お腹いっぱい。
垢版 |
2017/03/20(月) 15:49:01.64ID:w9uBS6H2
>>673 ID:JIuHTqi4
>特定の系(地球上)から見て

>>682 ID:JIuHTqi4
>地球を原点とする系と言ってます。地球付近の静止系ととらえてもらいたいのですが、

ID:JIuHTqi4 = ID:xu2UeICK
0834名無しさん@お腹いっぱい。
垢版 |
2017/03/20(月) 15:55:49.04ID:xu2UeICK
>>831-833
ええと、みんなめんどくさがってるのでどっか行ってもらっていいですか?

---------------------------------
Wikipedia 「慣性系」より抜粋

太陽系の重心に原点を取り、恒星系に対して回転しない座標系は慣性系に近いことが現在分かっている
---------------------------------
0835名無しさん@お腹いっぱい。
垢版 |
2017/03/20(月) 15:58:48.81ID:w9uBS6H2
予想通りそれ出してきたかwwwアホすぎるw

それは、厳密にいえば慣性系じゃない系を慣性系として扱う際に原点を設定するという話だよw
つまり、慣性系の説明ではないwwwwアホすぎるわ
0836名無しさん@お腹いっぱい。
垢版 |
2017/03/20(月) 16:01:46.38ID:ovoqk3u0
慣性系とはなんですか? 慣性の法則が成立する系です
でいいだろ
あとは物理板でもっと詳しい人とお話しておくれ
0837名無しさん@お腹いっぱい。
垢版 |
2017/03/20(月) 16:03:26.77ID:xu2UeICK
>>835
もうきみの妄言をりかいできる者はここにはいない。
さあ、宇宙板から足を洗おうな

-----------------------------------------------
805 名前:名無しさん@お腹いっぱい。[sage] 投稿日:2017/03/20(月) 12:04:13.80 ID:w9uBS6H2 [24/41]
これ終わったらもう宇宙板に書き込みするのやめるわ・・・
-----------------------------------------------
0838名無しさん@お腹いっぱい。
垢版 |
2017/03/20(月) 16:05:18.81ID:w9uBS6H2
>>836
です、それが正解です

なのに、あの人は >>832 みたいな答えを延々と繰り返し、間違ってるのはお前だと言い続けてるんですよ
頭おかしいでしょう?
0839名無しさん@お腹いっぱい。
垢版 |
2017/03/20(月) 16:08:50.12ID:xu2UeICK
>>838
文脈見たらわかりますが、そこ慣性系の説明をしてるとこじゃないですよね

さあ、出てけ!ウソツキ!出てけ!虚言癖!きもちわるーいwww
0840名無しさん@お腹いっぱい。
垢版 |
2017/03/20(月) 16:12:10.17ID:w9uBS6H2
ID:xu2UeICK = ID:JIuHTqi4

>ググレカス。

>ぶつぶつ意味わからんこといわずもう休めw
>おまえはもうとっくに限界だ

>それが移動してないように見えるの?目か頭大丈夫?

>あたまの残念な子だ。

>なにそれw
>ボクの妄想したうちゅうりろんを聞いてくれというサイン?

>無理クソだな
>こっちのツッコミは完全スルーして、同意したことにしてしまうなんてヤクザしか通用せんぞ

>私は物理学科卒ですが、専門家ではありません。
>まったく関係のない職業です。

>2chで出る質問などあんまりバリエーションがなく、
>レベルも低いし、多少煽っても大丈夫なとこなので
>調べながらなんだかんだ遊んでいるうちに対応できるようになります。
0843名無しさん@お腹いっぱい。
垢版 |
2017/03/20(月) 16:26:29.16ID:xu2UeICK
おまえも「キチ〇〇」「キチ〇〇」連発してたことは、とりあえず内緒にしといたほうがいいんかな?
0844名無しさん@お腹いっぱい。
垢版 |
2017/03/20(月) 16:36:12.26ID:w9uBS6H2
>地球を原点とする系
>空間が移動する

こんな表現する人は誰から見てもキチガイですよ?違うんですか?
0847名無しさん@お腹いっぱい。
垢版 |
2017/03/20(月) 17:22:41.71ID:BixnVEUD
これって僕の質問の地球も自転しててそらんなものの周りを回ってるからほぼほぼ光速のはなしからはじまったんじゃないですよね
0850名無しさん@お腹いっぱい。
垢版 |
2017/03/20(月) 19:23:03.30ID:xu2UeICK
>>847-849
いちおうフォローしときますが、
元になった彼の最初の発言が>>627-629あたりから
それに対する俺の発言が>>631あたりから

つまりどちらの方とも関係ありませんのでご安心を。
0851名無しさん@お腹いっぱい。
垢版 |
2017/03/21(火) 00:36:18.04ID:CLoko3pn
>>800
前に触れられてたけど「重力無限大」ってのは無いからね
(無限大の質量が必要)

密度無限大についても、その場合大きさ0の点(特異点)にならないといけないけど
それだと説明できない現象がある(詳細はホーキングパラドックスでググろう)

だから今では素粒子レベルの大きさの領域に折りたたまれた
「11次元の膜」とやらに質量が取り込まれてる、という認識になってるようだ
0852名無しさん@お腹いっぱい。
垢版 |
2017/03/21(火) 00:57:20.57ID:2BveRBC+
お前、何もわかってないな
ブラックホールの底は重力無限大だぞ
説明のつかない事になってるからこそ『特異点』という
0855名無しさん@お腹いっぱい。
垢版 |
2017/03/21(火) 02:30:30.33ID:ZchOb7m0
>>854
極限まで突き詰めると、素粒子が動いたり変化することが時間の経過と思う。
この電子とかは移動すんのにワープしたり、テレポーテーションしたりするから滑らかとは言えないんじゃないかな
0856名無しさん@お腹いっぱい。
垢版 |
2017/03/21(火) 02:35:05.37ID:ZchOb7m0
心配なってググッたら俺と同じ考えは既にあるようだ

最小時間間隔をプランク時間と呼ぶらしい
0861名無しさん@お腹いっぱい。
垢版 |
2017/03/21(火) 04:19:45.35ID:gyuPSLY6
>>855
素粒子の変化や動きで 時間経過し
滑らかではないのであれば
仮に パラパラ漫画の中身が 徐々に右手を上げる中身で 1ページだけ左手を上げてるページにしたら
そこから 左手を上げてる中身にすり変わってしまうとか あるんですかね?

素粒子の変化で 時間が分岐したり
右手上げる本 途中から左手を上げる本
はたまた 一瞬 左手を上げてるバクみたいな本 になるんかなー と 疑問なんですが 。
0864名無しさん@お腹いっぱい。
垢版 |
2017/03/21(火) 08:04:46.08ID:9UiiRDyD
物質の定義が問題になる。
簡単にテレポートとか言うけど定義が重要、つまり深い知識のバックグラウンドがあってこそ考えられる。
その深い知識とやらはネット探してもどこにも書いてない。
書いてあるのは通りいっぺんの科学マンセーばっかり。
空間が膨張するだのなんだなのと。
で全く同じ状態の素粒子が二つあったら、その素粒子に違いがあるといえるか?
全く同じ物体なら?全く同じ人体なら?
なにかしらヒントをもらいながら自分で考えるしかない。
自分の答えはテレポートできる。
0865名無しさん@お腹いっぱい。
垢版 |
2017/03/21(火) 08:34:55.52ID:a2gIGUP9
素粒子の状態が全く同じでも不確定性原理によって全く同じにはならないよねwww
0866名無しさん@お腹いっぱい。
垢版 |
2017/03/21(火) 08:44:02.95ID:a2gIGUP9
人間をテレポートさせるっていうことは
意識感情記憶全て転送しなきゃいけないけど
人の体内でも常にフェムト秒単位で電子の重ね合わせが発生していて、それが意識と関連性があるのではないかというペンローズの理論がある
意識感情記憶など全て転送するにはエンタングルした状態の素粒子をそのまま転送しなければ完全には同じ状態にならない
単純に離れた場所に全く同じ素粒子の構成を作っただけではただのコピー人間しか生まれないよね

そして量子テレポートはエンタングルしている状態に対して観測することで初めて発生するので
現状の技術では遠隔コピーまでで、どうやっても人間のテレポートは不可能だな
0868名無しさん@お腹いっぱい。
垢版 |
2017/03/21(火) 08:52:52.67ID:a2gIGUP9
ああ、スピンの存在を知らない人かwww
0869名無しさん@お腹いっぱい。
垢版 |
2017/03/21(火) 08:53:45.48ID:9UiiRDyD
>>866
人間のテレポートは意識のテレポート。
テレポート先に説明のつく肉体があればいいんだよ。
例えば人間は赤ん坊から生まれるだろ?
その赤ん坊はどこから来たかは問わないが、大人になって自分が存在する理由が分かったら説明がつくだろ?
つまり両親がいるから自分がいる。
当たり前すぎて灯台下暗しとでもいうか。
意識を考えるにはまず自分自身を疑わなければならないんだよね。
0870名無しさん@お腹いっぱい。
垢版 |
2017/03/21(火) 08:56:48.49ID:a2gIGUP9
>>869
あっはいwwwwwwそですねwwwww
(意識の転送にはエンタングルした状態をコピーする必要があるけどそれが現状不可能だって言ってるんだけどなwwwww)
0871名無しさん@お腹いっぱい。
垢版 |
2017/03/21(火) 08:58:58.27ID:a2gIGUP9
まああと2,30年くらいしたらエンタングルについて解明されるんじゃないかな そしたらエンタングルのままコピーできたりもできるんじゃない?
そもそも不可能ってこともありえるけど
0873名無しさん@お腹いっぱい。
垢版 |
2017/03/21(火) 09:02:52.56ID:Nn2c72+Y
>>866
人間の脳の動作なんて細胞単位で行われるものだから、電子の状態なんて細かいものに影響を受ける様なものではないと思うけどな
0874名無しさん@お腹いっぱい。
垢版 |
2017/03/21(火) 09:08:14.61ID:9UiiRDyD
まず前提として物理は空間があることが前提になってることを受け入れるか受け入れないか。
その空間の中の物質の運動を記述するのが物理。
空間の外とか別宇宙は物理じゃない。
物理量を持たないものは物理では扱えない。
0876名無しさん@お腹いっぱい。
垢版 |
2017/03/21(火) 09:57:14.25ID:9UiiRDyD
他人にも意識があると断言できるかな?
断言できるのは自分には意識があるということだけでしょ。
人間が人間の意識を物理で解明するのは原理的に無理。
意識のテレポートは原理的に可能。
0877名無しさん@お腹いっぱい。
垢版 |
2017/03/21(火) 10:07:21.45ID:a2gIGUP9
そうっすねwwww
(飛躍しすぎじゃね?wwwwww)
0879名無しさん@お腹いっぱい。
垢版 |
2017/03/21(火) 10:21:40.47ID:9UiiRDyD
物理的に可能かじゃなくてそれが必要かどうかで考えてる。
意識が物理的に解明されるのは必要じゃない。
テレポートは遠い未来には必要になるんじゃないかな。
原理は分からなくても遣えるものはあるんだと思います。

喩え話をするなら石油が人類の存続にとって絶対に必要だから
水よりも軽い油が長期間地中に存在し続ける物性が可能になってるんだと思います。
ちなみに温暖化も人類の存続にとって絶対必要。
原発の爆発も絶対に必要、ただしこっちは人類じゃないかもしれない。
0880名無しさん@お腹いっぱい。
垢版 |
2017/03/21(火) 10:31:33.24ID:9UiiRDyD
相対論は人工衛星を飛ばすために原子力発電をするために必要だった
量子論は半導体を作るのに必要だったとか言ったら怒られるんだろうな
0881名無しさん@お腹いっぱい。
垢版 |
2017/03/21(火) 10:50:32.29ID:a2gIGUP9
いっぱい文字を書いてるのに
「物理で意識が扱えないなら、どうして意識のテレポートが可能といえるのだろうか」
という質問に全く触れてすらいないwwwwwwww
0882名無しさん@お腹いっぱい。
垢版 |
2017/03/21(火) 10:52:40.55ID:a2gIGUP9
ID:9UiiRDyD こいつは就職面接で経歴やスキルを聞かれてるのに志望動機とか答えちゃうタイプだわwwwww
0883名無しさん@お腹いっぱい。
垢版 |
2017/03/21(火) 11:09:31.80ID:9UiiRDyD
違うね。就職面接にすら辿り付けないが正解。
経歴は自宅警備しかないし。
スキルはタイピングですw
志望動機は恋人にケツを叩かれたからですwww

物理で扱えなくてもテレポートの必要があるなら原理が解明されないまま使われることになるかもと言ってるんですw
物理と物性だけ解明してれば良かった過去と、これからの未来は違うんですよ。
なぜかと聞かれても論理では答えられません。
なぜなら論理は時間が一方向にだけ経過する場合にしか保証されないからですw
時間の一方向への経過は因果律を保証しますが、時間が一方向にしか経過しないなら宇宙は誕生しませんwww
0884名無しさん@お腹いっぱい。
垢版 |
2017/03/21(火) 11:17:17.61ID:9UiiRDyD
物理は空間があることが前提になってる。
その空間の中の物体の運動を記述するのが物理。
だから物理物理言ってるあいだは空間のことなど分かりはしない。
意識のことなどもってのほか、物理でそれを齎すものを解明することはできません。
0887名無しさん@お腹いっぱい。
垢版 |
2017/03/21(火) 11:54:14.37ID:9UiiRDyD
>>885
まあそういわず、暇だから付き合ってくれやw
>>886
その生命体って微生物だよね?
DNAはそもそも如何様にも変化できるものなら、その微生物がいるということは必然なんだよなw
でも如何様にも変化するためには障害があって、それは環境。
生物の進化変化は環境に左右される。
で、今その微生物がいるということは、DNAがそういう風に変化することが可能だったということ、環境があったということだ。
同じことが人間にも言える。
人間まで話を拡大したら、そもそも天体が存在しなければならないってなる、もちろんハビタブルソゾーンがある太陽系もそう。
で、あまりに偶然が重なり過ぎてるとなった時に、科学者のする説明と言えば別の宇宙が無数にあってたまたまこの世界が・・・というものだ。
だがそもそも空間は物理では扱ないから、別の宇宙があるなんてことも言えないんだが、まあ仕方ない。
これは過去と未来が干渉すると説明するなら、結果は同じだし、もっと簡潔になる。
だが過去と未来が干渉したら因果律は成り立たないし、どっちにしても物理的なことを言ってることにはならないんだがな。
論理は一次元の一方向にしか進まない時間軸でしか保証されない、だから過去にも戻れる時間軸の世界を論理で説明することは原理的にできないって分かるだろ。
そういうことであって、物理も全く同じ、物理の外にあるものを解明することはできない。
0888名無しさん@お腹いっぱい。
垢版 |
2017/03/21(火) 12:16:58.04ID:a2gIGUP9
せめて量子論勉強してから書いてほしい
0889名無しさん@お腹いっぱい。
垢版 |
2017/03/21(火) 12:36:05.54ID:a2gIGUP9
ふふふ
0890名無しさん@お腹いっぱい。
垢版 |
2017/03/21(火) 12:45:36.63ID:9UiiRDyD
理論の勉強して分かるのは結局のところ、この宇宙が取り得る物理定数だからな。
要は目の前の世界が成り立つにはどうゆう物理定数になってるかということだし。
目の前にあるものだから感覚で捉えた方が分かり易いしw自分は専門じゃない以上は理論には入りたくないなw
宇宙のことだけ考えていたいから、遠慮しとくわw
0893名無しさん@お腹いっぱい。
垢版 |
2017/03/21(火) 14:21:16.85ID:5xZ4r6wm
流れる時間というのは人の頭の中にしかないです
それを言えば時間も人の頭の中にしかないわけですが
どちらかと言えば、流れると考えるよりはパラパラ漫画と考える方がシンプルで合理的です

あとプランク時間は、
なぜか分からないけど、宇宙のどこを探しても一番短い時間はそれになるというだけなので
その長さが元の材料となって時間が存在するというわけではないです
確かにその長さを基準に宇宙が作られてるようにも見えますが、そうではない可能性もありますし
時間は不連続だとか言う人いますが、それはあくまでその理論の中だけの話です
0894名無しさん@お腹いっぱい。
垢版 |
2017/03/21(火) 14:28:46.16ID:9UiiRDyD
>>892
そのサイトの人はかなり考えたようだけど、間違ってるところもある。
自由意志があるかどうかを考えるところでコップを選ぶ話がある。
だけどこの場合はコップに入った何かを貰うということで、それて自由意志があるかどうかを判断することはできない。
貰うのじゃなくて捨てる場合を考えなければいけない。
0896名無しさん@お腹いっぱい。
垢版 |
2017/03/21(火) 18:18:18.91ID:ZchOb7m0
その空間内の全ての物質や素粒子が静止したら、その空間内は時間が止まっていると言える。

しかし絶対零度でも素粒子は運動しているのでそのような空間を作ることが不可能。
つまり、宇宙の全ての空間にエネルギーがある限り、時間が存在し止まらない
時間とはエネルギーである
0897名無しさん@お腹いっぱい。
垢版 |
2017/03/21(火) 18:21:33.75ID:ZchOb7m0
エントロピーの増大で宇宙の全てのエネルギーが最終形態(ダークエネルギー?)になり作用をしなくなったとき、
時間が止まったと言えるであろうか
0899名無しさん@お腹いっぱい。
垢版 |
2017/03/21(火) 19:02:26.35ID:knDXwt0a
LHCが稼働してから大発見といえるものはヒッグス粒子だけ。
あれだけでかい装置使って世界中の優秀な素粒子物理学者たちが取り組んでるのに
めぼしい成果がない。今彼らは相当焦っていて、今後数年以内に
白旗を上げるかどうかが決まるかもしれないと考えている

一方で、統一理論として有力視されていた超ひも理論が予言するのは
多元宇宙で、つまり我々はたまたま、ほとんどあり得ないような確率で
物質の存在が許される物理法則に支配されたハビタブルな宇宙に
生まれてきただけだという。もしそれが事実なら、これ以上の観測も
証明も不可能ということになる。
0900名無しさん@お腹いっぱい。
垢版 |
2017/03/21(火) 19:10:31.69ID:5xZ4r6wm
一緒に運動する物体やエネルギーの集団は、外から見れば動いて見えるけど中から見れば止まって見える
外から見ると慣性運動+相互作用を感じ取れるけど
中から見ると相互作用しか感じ取れない
この中からしか感じ取れない相互作用が中の人にとっての時間や距離

時間や距離は人の頭の中にしかないけど、その本質は一緒に動くことにある
時間や距離という考え方は一緒に動かないものの方が多い宇宙ではあまり意味を成さない
0902名無しさん@お腹いっぱい。
垢版 |
2017/03/21(火) 19:17:29.51ID:5xZ4r6wm
絶対零度でも動いている、仮にそれらが相互作用をしているとしたら、
その相互作用がどんなにゆっくりだとしても、
中の人にとっては何気ない日常の時が流れる
0903名無しさん@お腹いっぱい。
垢版 |
2017/03/21(火) 19:20:15.97ID:a2gIGUP9
あのー
絶対零度に達するには無限のエネルギーが必要なんですがそれは
0904名無しさん@お腹いっぱい。
垢版 |
2017/03/21(火) 19:23:48.27ID:5xZ4r6wm
光速の手前まで加速するには無限のエネルギーが必要になりますよね
でも中の人にとっては何気ない時が流れる、それと同じ仮定の話です
0905名無しさん@お腹いっぱい。
垢版 |
2017/03/21(火) 19:36:24.98ID:5xZ4r6wm
あれ?そもそも温度で相互作用って変化するのかな?
しないなら温度と時間は関係ないですね、ごめんなさい
0906名無しさん@お腹いっぱい。
垢版 |
2017/03/21(火) 21:16:19.39ID:9UiiRDyD
>>904
有限のエネルギーしかなくても無限大時間加速すればOK
つまりブラックホールが有限の時間内に形成されるというのは嘘でもなんでもなく無限大のエネルギーがあれば可能なんですね
物理学者は決して詐欺をしているわけではなく、有限の質量、有限のエネルギーがありさえすれば何らかの魔法によって有限の時間内にBHが出来ると信じ込んでるんですね
信じ込んでいるので詐欺には当たりません、騙す意図はないですし、そもそも立証不可能です、記憶に御座いませんと同じ理屈です
どれもこれも一般大衆からの売り上げがあるから可能なのです
0907名無しさん@お腹いっぱい。
垢版 |
2017/03/21(火) 21:21:46.75ID:LY+aRQ6w
多くの科学者が検証してきた理論と自分の理論の矛盾を考えてなぜ前者が間違っていると考えてしまうのか
0909名無しさん@お腹いっぱい。
垢版 |
2017/03/21(火) 22:08:14.24ID:9UiiRDyD
>>907
理論て数式だよね?
問題なのは数式が何を意味するかであって、これはどんな説明でも可能。
言葉は厳密に定義されてなんぼなので世界とか宇宙とか言い換えられる空間とかいう都合のいい言葉を持ちだせばみんな黙ってしまうんです
数式が完璧でツッコミどころがないから、くいっばぐれないためには黙っておくしかないのですな
だけど、なんで重力があるかって考えたことある?重力から逃れられるかって考えたことある?
はっきり言って無理なんだよね、IS国とか問題になってる今アインシュタインが発見wwwwwwした重力理論は完璧でなければならないんだよなww
はいはい、空間は膨張しますし、天体は空間に固定されていると思いますよ、みんな誰でも信じます
そう、エヂソンは偉い人、そんなのじょーしきなんですwwwwwwwwwwwwww(おおおおおおうそ
0911名無しさん@お腹いっぱい。
垢版 |
2017/03/21(火) 22:15:53.92ID:9UiiRDyD
科学も民主主義も女性の権利も、みんな欧米人の発明です
なぜか?
すまんがこれを説明するには本一冊書くエネルギーが必要だか、それに見合う対価は絶対にwww得られないので書かない
だから結論だけ書くが、世界は全部嘘です
大嘘
世界で起きてきたこと全部に-1をかけるといいです
自分の持っているイメージ、自分の持っている記憶全てを反転するのです
それが正しい世界の描像てず、ただし自分自身にも多少の反転はあるでしょう
これについては本10冊書くEが必要になる
0912名無しさん@お腹いっぱい。
垢版 |
2017/03/21(火) 22:20:18.24ID:ZchOb7m0
ブラックホールってのは物理的に直接観測することが不可能だから
有限の時間内に形成されるか否かは関係ない。

間接的にでもそこに在るから有るのである。
0913名無しさん@お腹いっぱい。
垢版 |
2017/03/21(火) 22:22:40.16ID:ZchOb7m0
>>907
>>908
カルト指導者は自分の主張や考えが正しいか嘘かは問題にしない。
ただ自分の主張や妄想を、他人に信じ込ませることだけを目的とする。
0914名無しさん@お腹いっぱい。
垢版 |
2017/03/21(火) 22:26:12.47ID:ZchOb7m0
量子力学的には、誰も月を見ていなかったら、そこに月は存在しない。
観測して初めてそこに月が在る。

ブラックホールも同じ。
間接的には存在するが、直接観測できないので存在してないとも言える。
何も矛盾していない
0916名無しさん@お腹いっぱい。
垢版 |
2017/03/21(火) 23:06:43.91ID:c9CrP4xv
個人的な思考実験から、ブラックホールの周囲は遠くから見た見た目よりずっと広いのではないかと思える事が有る。
ブラックホールの特異点に向かって一直線に無限に伸びる紐を垂らした場合、
紐は潮汐力で引き伸ばされていく筈なのに、遠くの観測者から見て
垂らした紐の先端が事象の地平面で硬直し後から続いてきた部分に
追いつかれると言う矛盾現象が起きるからだ。
0918名無しさん@お腹いっぱい。
垢版 |
2017/03/21(火) 23:20:54.15ID:LY+aRQ6w
>>909
理論と観測結果が一致しなかったら否定できるでしょ

重力は今研究されてるらしいM理論が完成したらわかるんじゃないかな
今どんな状況かは知らないけど

IS国?もしかしてイスラム国の事?それならISでイスラム国だからイスラム国国って意味になっちゃうよ
IS国がイスラム国だとして重力波となんの関係があるの

黒色火薬は中国人の発明だよ

あとなんで急に誤字が増えたの?キーボードの調子が悪いんだったら完全に壊れる前に電池を変えるか買い換えるかした方がいいよ

>>917
検索したけど>>917が何を言いたいのか分からん
0919名無しさん@お腹いっぱい。
垢版 |
2017/03/21(火) 23:34:57.02ID:d0MfFhIb
テレポーテーションなんてできるの?
できたとしてもそこにいるのは違う自分何じゃとおもってしまう
0920名無しさん@お腹いっぱい。
垢版 |
2017/03/22(水) 00:48:12.26ID:oiVhBDb2
できないよ。人を電子までバラして別の場所に出現させても再構築できないからね。
もし完成したらその時は人類は不老不死も手に入れてるだろう
0921名無しさん@お腹いっぱい。
垢版 |
2017/03/22(水) 00:54:03.29ID:qspIC3j2
あなたも時間差でテレポーテーションしている。

人間の血液は3ヵ月で全量が入れ替わる。新陳代謝だ。
10年前のあなたの写真を見てみよう。
その写真に写るあなたの肉体はもうこの世に存在しない。
肉体を構成していた物質がほとんど入れ替わっているから。

あなたに残っているのは記憶という情報だけだ。
0923名無しさん@お腹いっぱい。
垢版 |
2017/03/22(水) 01:07:28.98ID:qspIC3j2
>>916
思考実験だからこそ矛盾が生じる。

矛盾を生じさせない為にブラックホールは観測不可能となる。
観測できちゃったらブラックホールが存在しないことになってしまう。
0926名無しさん@お腹いっぱい。
垢版 |
2017/03/22(水) 08:04:02.99ID:I6gNLyLj
>>907
正しい理論や法則は一つしかないという幼稚な思考の人から見れば
異なる理論を唱える人が否定してるように見えるだけ

正しい理論が一つしかないという考えなら
例えば、相対性理論と量子論は必ずどちらかが間違ってるということになる
物質の表現方法だって、りんご、原子、粒子、エネルギーと色々あって一つじゃない
君の考え方は、この中で正しいのは一つだと言ってるのと同じ
正しさで考える人間は物理学に向いてない
0927名無しさん@お腹いっぱい。
垢版 |
2017/03/22(水) 08:12:51.33ID:U24T38x6
なんかの宇宙の番組でやってたけど
天の川銀河は大きな銀河団に属していてその銀河団も渦巻状になってて
さらにその銀河団ももっと大きな銀河団に属してるって言ってた。
さらに大きな銀河団もあるんでしょうか?
というかこれ本当に事実なんでしょうか?
0928名無しさん@お腹いっぱい。
垢版 |
2017/03/22(水) 08:20:28.16ID:ppRtmivV
>>925
逆だよ
エネルギーが完全に0だよ
そこまで達するのに有限回の操作では不可能
0929名無しさん@お腹いっぱい。
垢版 |
2017/03/22(水) 08:33:13.78ID:S30LxlDb
テレポートは出来るようになるだろ。
人間のテレポートは意識のテレポートだけど意識は解明できない。
でも原理は分からなくても納得できる理屈が納得して使うようになる。
例えば人間の体は母親から出て来るけど初めから意識があったわけじゃないよな。
赤ん坊に意識が宿るのはテレポートの一種だろ。
受精して細胞分裂して肉体が出来るという納得できる理屈があるから誰も不思議に思わない。
これは単なる意識の不連続の一例だけどな。
0930名無しさん@お腹いっぱい。
垢版 |
2017/03/22(水) 08:56:05.98ID:S30LxlDb
今は受精とか細胞分裂とかいう知識があるけどそれがない昔ならどうだった。
それでもやっぱり不思議に思わないだろ。
昔の人も自分の体があって自分に意識があることに何も違和感を持たなかった。
これは喩えるなら内臓に触覚がないのと似た話で、内臓に触覚は必要がないから動物には備わってない。
自分の意識がどこかからやってきたかもしれないという違和感は持つ必要がないんだよね。
つまり今胃が動いてるとか腸がぜん動してるなとか、感覚がないから意識にも上らないのと同じで意識がどこから来たという疑問も意識にはのぼらない。
そんな時に受精して細胞分裂して自分が出来たと知ると、じゃあ意識はどこから来たと疑問提起することもできる。
内臓に感覚触覚がないのは如何様にも変化するDNAの性質で説明できても不連続なはずの意識に違和感を持たない理由を説明できるか。
これはもう動物として生きる為の解が意識に組み込まれてると考えるしかないと思う。
宇宙の仕組みと同じ、天体が存在するための解が物理法則だ。
テレポート可能な解も必ずあるはず。
0931名無しさん@お腹いっぱい。
垢版 |
2017/03/22(水) 09:01:35.08ID:ngTg0pHz
>>926
相対性理論と量子力学はスケールが違うけどブラックホールの有無はそういうのは関係ないと思うんだけど?
リンゴと表現しても分子の集合体と表現しても矛盾は生じないけどブラックホールの存在の有無だと矛盾が生じるんじゃない?
0933名無しさん@お腹いっぱい。
垢版 |
2017/03/22(水) 09:21:20.29ID:S30LxlDb
根拠というのは一次元の一方向の時間経過でしか必要にならない。
それ以上の次元の時間は因果律を保証しないから。
宇宙は一次元の一方向にしか経過しない時間軸で宇宙は誕生するか?
しないなら宇宙誕生に根拠はないはずだよ。
物理だ根拠だ言ってる間は時間軸を超えた思考はできないね。
だから苦し紛れに別宇宙を出して偶然というしかなくなる。
けどそれももう物理じゃなくなってる。
0936名無しさん@お腹いっぱい。
垢版 |
2017/03/22(水) 10:11:30.86ID:S30LxlDb
別の宇宙が無数にあるというのも妄想
だからどのみち妄想以外でこの宇宙の物理定数が丁度いい値になってる説明はできない
0938名無しさん@お腹いっぱい。
垢版 |
2017/03/22(水) 10:16:22.20ID:encXGGr0
>>929
おまえ自体妄想の塊の件wwww
0941名無しさん@お腹いっぱい。
垢版 |
2017/03/22(水) 10:43:13.32ID:S30LxlDb
宇宙の質問?
本みたら理論の式で解が書いてあるだろw
それ見て分からないバカに教えて何がうれしんだ?
解が書かれてないことこそが難しいんだよ
0943名無しさん@お腹いっぱい。
垢版 |
2017/03/22(水) 10:48:45.93ID:Qaf4v1/N
ここは「宇宙の質問が書き込まれたら誰かが即答するスレ」だから
議論スレじゃないから、お前の目的に合わせたスレ立てろよw

お前さ、なんで自分でスレ立てて自分の妄想垂れ流さないんだ?
なんでこのスレに寄生するんだ?
それやると誰もかまってくれないからだろ?
要するにお前の妄想なんて、その程度の価値しかないってことだよw
0944名無しさん@お腹いっぱい。
垢版 |
2017/03/22(水) 11:44:41.07ID:encXGGr0
>>939
マイクロチューブルだぞ

まず全身麻酔の原理から始めないといけない。
通常寝てる時でも人は意識があり体内時計が動いている、
だから目が覚めた時に大分時間が経っているのが大まかにわかる。
しかし全身麻酔を施した場合、人の意識は完全に消える、
意識がなくなったと感じた次の瞬間に目が覚めて、にも関わらず十数時間経っていて、体内時計が完全に止まっていた状態になる。

その全身麻酔の原理は未だに解明されてないが、
効用としては、細胞内のマイクロチューブルの働きを止める作用を持っている。
さらにマイクロチューブルは構造的にチューブリンと呼ばれる極小のタンパク質から作られる管状になっていて、閉じられた空間を形成する
そしてそのマイクロチューブルは電子一つによって管が開閉される。

そのマイクロチューブルと意識には密接な関係があることはわかっているが、
電子一つによって操作され、管内でなにかしらの計算処理をすることで意識が作り出される。
さらに閉じた管内ではフェムト秒単位の重ね合わせが発生している。

以上により、意識はマイクロチューブル内の電子の重ね合わせ、またはエンタングルによって作り出される可能性が高い。


というのがホーキンスと共にブラックホールの特異点を証明したペンローズの量子脳理論。
まだ完全論ではないが、マイクロチューブルと意識の関連性は確定している
0946名無しさん@お腹いっぱい。
垢版 |
2017/03/22(水) 12:46:02.80ID:encXGGr0
素で間違えたwwwwwwww
失礼wwwwwwww

0948名無しさん@お腹いっぱい。
垢版 |
2017/03/22(水) 14:00:38.71ID:h6blO11Y
正確な時計は周囲の時計を狂わせる
http://news.mynavi.jp/news/2017/03/22/171/
http://news.mynavi.jp/news/2017/03/22/171/images/001.jpg
量子力学と相対性理論によって解明

図のように、一般相対性理論では、空間のどのポイントでも
他から影響を受けずに正確に時刻を測れる理想的な時計を考えることができる。
しかし、量子力学も考慮に入れた場合、隣り合う時計同士は互いに独立ではなく、
干渉しあって時間が不正確になる
0949名無しさん@お腹いっぱい。
垢版 |
2017/03/22(水) 14:16:29.30ID:I6gNLyLj
>>931 >ブラックホールの存在の有無だと矛盾が生じるんじゃない?

私はブラックホールの話や、有り無しの話はしてないけど、一応答えますね

無いという概念は人の頭の中にしかないです、なので有と有の関係性を見出すのは物理学ですが
無いことを考えるのはものすごく無駄です、とても幼稚です、無いとはどういう概念なのか理解できてないだけです

次に矛盾とはどういう概念なのか
それは、違うという概念でしかないです
こちらの考え方から見ればあちらは違って見える、ただそれだけです

哲学を学べば正しさや矛盾が幻でしかないことが理解できます
理論の存在理由は正しさや矛盾ではなく、何が最も納得できる合理的な考え方か、それで決まります
納得するための考え方が一つである必要はない
0950名無しさん@お腹いっぱい。
垢版 |
2017/03/22(水) 14:25:17.32ID:I6gNLyLj
なので、仮に理論Aと理論Bが矛盾して見えるような場合、
互いの自分の理論の中では矛盾が無いはずなので問題は無いですが
その二つを同時に使いたい場合は、不便である
二つとも必要なら二つが矛盾しなくなる様な新しい理論を考えればいいだけです(統一理論)
どちらが間違ってるのかという発想はとても幼稚なのです
0951名無しさん@お腹いっぱい。
垢版 |
2017/03/22(水) 14:37:26.83ID:Qaf4v1/N
理論の有用性は客観的な説明能力で決まるのだ
なるたけシンプルな方が理論としては優れているといえる
そして、ボクの考えた宇宙論のほぼすべてが観測事実を客観的に説明する能力に欠けているw
0952名無しさん@お腹いっぱい。
垢版 |
2017/03/22(水) 14:41:17.84ID:I6gNLyLj
>観測事実を客観的に説明する能力に欠けているw

それは、相対論からみた量子論、あるいは量子論からみた相対論も同じですよ

矛盾や違いを指摘することは新しい理論を生み出すのにとても重要ですが、
ただ違うことを言い続けるのはただのアホです
0953名無しさん@お腹いっぱい。
垢版 |
2017/03/22(水) 14:46:04.48ID:I6gNLyLj
>ボクの考えた宇宙論

どの理論も認められるまではそれと同じですよね

◯◯理論では◯◯はどうなりますか?
こういう質問なら答えは限定されてきますが、
そういう形式の質問でないなら、見る人が何を求めてるかによって
答えは一つではなくなってきます(ボク理論)
0954名無しさん@お腹いっぱい。
垢版 |
2017/03/22(水) 14:46:41.39ID:S30LxlDb
あの、宇宙は今の1次元1方向の時間軸の延長で発生したと思いますか?
そうじゃないなら因果律は保証されないので客観的に説明とか全くありません。
0955名無しさん@お腹いっぱい。
垢版 |
2017/03/22(水) 14:47:14.37ID:YQyU9GYF
天体にも星座の見分け方にも疎い全くの初心者なのですが
ある星空が写った写真一枚の星の位置から
星座を見つけたり写真を写したおおよその場所を特定する為のスキルを教えて欲しいのですが
お願いします。
そう言ったスキルが乗ってる本やHPでもありがたいです。
0956名無しさん@お腹いっぱい。
垢版 |
2017/03/22(水) 14:53:57.78ID:Qaf4v1/N
>>953
矛盾の話なんかしてないが
相対性理論はマクロの領域に関しては非常に有意な説明能力を持っている
量子力学はミクロ領域でこれまた非常に優位な説明能力を持っている
ちゃんと説明能力は満たしてるので、これはこれでいいのだ
ニュートン力学も光速近くになると破綻して上手く説明できなくなるが、日常生活の領域ではちゃんと必要な説明能力を満たしてるので問題ないわけだ
重要なのは特定の観測事実に対して再現性と客観性をもった説明能力があるということ
ただ、ミクロ領域は量子力学を使ってマクロは相対性理論を使おうというのはやはり不都合があるのでミクロもマクロもすぱっと説明出来る統一された理論が欲しいわけだが

ボクの考えた宇宙論のほとんどは観測事実を説明出来てないだろ?
だからアカンのだよ
説明できているように見えるなら、ひょっとしたら使えるかもしれんので検証作業が行われて認められるに至るだけの話だ
0957名無しさん@お腹いっぱい。
垢版 |
2017/03/22(水) 14:55:44.31ID:ngTg0pHz
>>949
裸の特異点が存在し得るかどうかを考えることも無駄ってこと?
無いという概念が理解できてないってのがどういう事かよく分からないから分かりやすく解説してくれてるサイトがあったら教えて欲しい

理論と理論の間じゃなく観測結果と理論の間に違いがあったら観測結果に合うように理論を修正すべきじゃないの?
0958名無しさん@お腹いっぱい。
垢版 |
2017/03/22(水) 14:58:36.37ID:Qaf4v1/N
>>955
経験則によるところが大きいんで、自分でいっぱい星空みなさいというツマラン答えになっちゃうが
まあ、http://www.astroarts.com/alacarte/index-j.shtml
とかが参考になるんじゃないかな

とりあえず天体写真から場所を特定するには日時がわかってないとさすがに無理
日時が分かれば、惑星や目立つ星の位置から地域をおおよそではあるが特定出来るから
天の川がやっぱりいちばん目立つんで、これが写ってたらかなりラクよ
0959名無しさん@お腹いっぱい。
垢版 |
2017/03/22(水) 15:00:25.57ID:I6gNLyLj
>観測事実を説明
事実が自分の解釈次第で一つではないことを理解してないようですね

>検証作業が行われて認められるに至るだけの話だ
検証されて正しさが証明されるような言いかたですけど、それは全然違います
自分の概念、考え方を切り替えることでそれが認めらるようになる、それが理論です
なぜそうするか?そうせざる負えない現象が観測されるからです(←これを事実とは言わない、人の解釈次第なので)
0960名無しさん@お腹いっぱい。
垢版 |
2017/03/22(水) 15:02:33.11ID:I6gNLyLj
>957 >存在し得るかどうかを考えること
それは有を考えてるので物理学ですよ
私が言ってるのは無を考えるのが物理学ではないということ
0962名無しさん@お腹いっぱい。
垢版 |
2017/03/22(水) 15:05:13.40ID:YQyU9GYF
>>958
やっぱり経験がモノを言うんですね。
星座だけ判別出来れば十分助かります。
ありがとうございます、頑張ってみます。
0963名無しさん@お腹いっぱい。
垢版 |
2017/03/22(水) 15:07:08.58ID:I6gNLyLj
>>961
物理学の元は哲学ですよ?
見たままの世界を検証すればいいだけの物理学は何十年も前のはるか昔の話です
見たままでは矛盾しか発見されないので、哲学的に、自分の考え方を変化せざる負えないレベルに到達してるのが、
現代の物理学です
0964名無しさん@お腹いっぱい。
垢版 |
2017/03/22(水) 15:07:27.04ID:S30LxlDb
物理は空間があることが前提になってる
その空間の中の物質の運動を理論化したのが物理
だからアインシュタインが言ってるのはオーウソだよ
物理は空間にタッチできない
ただし相対論は完璧なまでに運動を説明するから誰も反論できない
↑これは数式
でも解釈は子どもが考えても分かる、ウソ

空間が膨張して天体が遠ざかるなら天体は空間に固定されているか天体と空間の間に摩擦が無ければならない
だが固定されているなら自由に運動できないし摩擦があるなら慣性の法則は成立しない
おーうそ
0965名無しさん@お腹いっぱい。
垢版 |
2017/03/22(水) 15:11:37.55ID:Qaf4v1/N
>>962
とりあえず、金星・木星・火星が季節ごとにどの方向に見えるかを覚えるといいよ
とくに金星と木星は目立つから

次に天の川の季節ごとの見え方を覚えるといい
天の川を基点にして、季節ごとの一等星以上の星の位置を覚えるとそこから有名な星座の位置がわかるようになる
個人的には北斗七星とカシオペア座をすぐに見つけられるようにしておくとかなり役立つ
これは北極星が山に隠れて見えない時とかに方角を割り出すのに役立つから

これだけ覚えておくと、星座はかなり見つけやすくなる
ただ、星座って想像以上にかなりでかいってのは体感で覚えておいたほうがいい
大きい星座は視界に収まりきらないので、見えてるけど認識出来ないというのがザラ
0968名無しさん@お腹いっぱい。
垢版 |
2017/03/22(水) 15:17:13.63ID:Qaf4v1/N
>>962
あと、蛇足だけどアンドロメダ銀河は肉眼で見つけれるようになっておくとなぜか星空の把握能力が高まるw
アンドロメダ銀河は周辺視野を上手く使えないと見えないから、自然とそういう目の使い方を覚えられるよ
0970名無しさん@お腹いっぱい。
垢版 |
2017/03/22(水) 15:26:03.63ID:I6gNLyLj
>>969
>>951 >そして、ボクの考えた宇宙論のほぼすべてが観測事実を客観的に説明する能力に欠けているw

あなたのこの煽りは、質問スレに何の関係があるの?
私から見れば関係の無い話を最初に始めたのはそちらです、そして私は哲学的な疑問には答えてます
0971名無しさん@お腹いっぱい。
垢版 |
2017/03/22(水) 15:30:31.37ID:Qaf4v1/N
>>970
あるよ

このスレは>>955のような質問に対して即答するスレであって自説を開陳するスレではないので、ボクが考えた◯◯をひたすら言い続けるってのはスレ違いってことだよ

ということだよ
あと、哲学的な疑問に答えるスレでもないからな
そういうのは哲学板って聞くから、ここで答えなくてもいいよ

まあ、俺もスレチな話題にずれたのは認めるがちゃんと>>955のような質問が出たら即レスしてるぞ?
0973名無しさん@お腹いっぱい。
垢版 |
2017/03/22(水) 15:32:18.75ID:I6gNLyLj
理論毎に答えが異なるのは分かりますよね?
とある理論の難しい話を一般向けに修正した話し方が、幼稚な僕理論に見えてる可能性は十分にありますよ?
つまり、あなたから見て幼稚で間違ってるように見えるだけかもしれないので、そんな煽りはただの不適切
0974名無しさん@お腹いっぱい。
垢版 |
2017/03/22(水) 15:37:15.71ID:S30LxlDb
上のほうで「全部の素粒子の運動が静止したら時間は経過してないことになる」とかいうレスがあったけど
その場合空間はどうなる?
膨張とまんのか?あ?
0976名無しさん@お腹いっぱい。
垢版 |
2017/03/22(水) 17:26:55.82ID:S30LxlDb
真空の揺らぎか?
真空w圧力単位持ちだして何が言いたいのか
でその真空とやらは相対論の言ってる空間と同じもの?
同じものなら統合できてると言ってるのだし、違うなら違う場があるってことだろ。
オマエは相対論で扱う場はないと言ってる?
答えて見ろオラ
0977名無しさん@お腹いっぱい。
垢版 |
2017/03/22(水) 17:59:30.37ID:NspC7dfV
>>923
>矛盾を生じさせない為にブラックホールは観測不可能となる。

>>916の思考実験は、事象の地平面よりも外側の話だ。
事象の地平面よりも外側なら観測できるではないか?
0979名無しさん@お腹いっぱい。
垢版 |
2017/03/22(水) 20:20:02.11ID:qsyxMF5d
>>977
彼の言ってることはまったく逆で、思考の中の実験なのに矛盾が起きちゃうってことは、
その段階で確実に何か間違えてるんだと思うが
0982名無しさん@お腹いっぱい。
垢版 |
2017/03/22(水) 20:59:34.27ID:dtl3dIDK
俺はだめだった
術後麻酔切れて目が覚めてから
次の日の朝まで12時間くらい、頭ちょっと動かすだけでも全身に激痛が走る
寝られれば気が楽だったんだけど
寝ても30分くらいで目が覚めてほぼ一晩中同じ姿勢で起きてる状態だった
まさに地獄
0983名無しさん@お腹いっぱい。
垢版 |
2017/03/22(水) 21:27:59.08ID:qsyxMF5d
そっか切ってんだもんね
切ってんだから麻酔醒めたら痛いよね
タイムリープごっことかに浸ってる場合じゃないよね
0984名無しさん@お腹いっぱい。
垢版 |
2017/03/22(水) 22:03:03.62ID:S30LxlDb
盲腸は医者の食い扶持
過払い金請求は弁護士の食い扶持
みんな用意されたものなんです
これも一つの解
天体が存在するために盲腸が必要なわけじゃないが
文明が存続するためには必要なんですな
もちろん温暖化もせず太陽も永久に光り続けるなら別だか
0986名無しさん@お腹いっぱい。
垢版 |
2017/03/22(水) 22:33:50.06ID:x4BEopwQ
テレビでやってたが虫垂炎で周辺を切除手術した人は
その後2〜3年の死亡率が増えたらしいねー
(虫垂に有った有用細菌や免疫材料が無くなった為)
0987名無しさん@お腹いっぱい。
垢版 |
2017/03/22(水) 22:44:27.12ID:jwYgTlwk
先月の夜 ベランダで星みてたんよ
オリオン座もうあんなとこか冬も終わりかって
マンションの最上階に住んでんだけど 、上10か15mくらいの上空で蒼白い炎、、、バーナーの様 なものが現れて数秒後に消えたんよ
一瞬でないからハッキリ見えたんだけど 確かに機体のエンジン、、の形
ステルス不調でチョット見えましたみたいな

いまだになんやねんておもうわ
0989名無しさん@お腹いっぱい。
垢版 |
2017/03/23(木) 02:15:45.66ID:cy48n7Do
それ戦闘機がブラックホールに吸い込まれる瞬間を目撃したんだよ
地球上でもバミューダ海域などにブラックホールはあるけど気圧の関係で偶然日本にブラックホールが来たんだろうな
0991素人君
垢版 |
2017/03/23(木) 06:20:40.67ID:TlnZ90TF
>>968
アンドロメダ銀河て視認できるんですか?
0992名無しさん@お腹いっぱい。
垢版 |
2017/03/23(木) 08:10:47.63ID:aufcIotH
慣れれば簡単に見える
ただし、写真のような渦巻きには見えない。ぼんやりした雲のように見える
4等星ぐらいの明るさだから街中では無理かな
0993名無しさん@お腹いっぱい。
垢版 |
2017/03/23(木) 09:58:00.19ID:Rz2F9YTn
相対論で扱う場
電磁気論で扱う場
量子論で扱う場
弦理論で扱う場
天体が等方的に遠ざかる=斥力場

空間というのはこれら全部をくっ付けたものだよね?
これら全部を数式で統合できないなら、はっきり言ってシンプルさに欠ける
ごちゃごちゃしすぎてる
オッカムのカミソリ的に言えば宇宙を作ってるのは物理定数でなくて意識が造ってるとするほうがシンプルであって真なのだ
空間が有るというのは幻想、空間は物理量を持たない、そして意識が物理現象と考えるのも幻想、意識は物理量を持たない
ただしそれじゃあつまらないというのは分かる
だから、意識が宇宙を作ってると考えたらシンプル過ぎて何も疑問はないくらいのシンプルな理論が作れるならそれでもいいんだ
言い換えよう、万有引力の法則が天動説を基にしたごちゃごちゃとした式だったなら、誰もそれが真とは信じない、シンプルに表せる地動説が真なのだ
0994名無しさん@お腹いっぱい。
垢版 |
2017/03/23(木) 11:49:11.29ID:XZ+vof+V
大雑把すぎて恐縮なのだが人口爆発の影響で移住政策が取られたら、選択の余地がない貧民は地球に残るのか、それとも他の星へ行くのかな。選別の基準なども教えていただきたい。
0995名無しさん@お腹いっぱい。
垢版 |
2017/03/23(木) 12:09:50.77ID:Rz2F9YTn
貧民は地球に残るかどうかの選択をするほど短期間に移住できるわけじゃない。
テクノロジーの進歩より世代交代の方が早いから残る残らないで問題になったり残った方はみな死んでしまうとかはない。
最終的には南極が肥沃な農地になる可能性もあるが、そこまで環境が変化するまえに疫病洪水海面上昇によって直接的に多くの人が死んでくし
それでも熱帯国以外では経済成長はするから、その結果出生率は下がる、この先人口爆発するリスクのある地域は南米とアフリカくらいだろうか。
中国すら一人っ子政策を撤廃してしまったが、これは条件付きかもしれないが、この事実は中国は民主化して日本韓国のようになるフラグ。
つまり宇宙開発は万が一の時の担保として、気候変化の抑制と同時に両面で対策が進んでいく。
しかしこれはどっちにしろ宇宙に進出するためのテクノロジーをもたらす。
特に温暖化はエコ技術の進歩をもたらす。
つまり石油があることによって経済成長が可能、その上気候変動を巻き起こすからエコ技術の開発が進むという、人類存続の一つの解なのだ。
1000名無しさん@お腹いっぱい。
垢版 |
2017/03/23(木) 12:35:41.35ID:wB/DoOou
1000
10011001
垢版 |
Over 1000Thread
このスレッドは1000を超えました。
もう書けないので、新しいスレッドを立ててくださいです。。。
life time: 24日 21時間 44分 32秒
10021002
垢版 |
Over 1000Thread
2ちゃんねるの運営はプレミアム会員の皆さまに支えられています。
運営にご協力お願いいたします。


───────────────────
《プレミアム会員の主な特典》
★ 2ちゃんねる専用ブラウザからの広告除去
★ 2ちゃんねるの過去ログを取得
★ 書き込み規制の緩和
───────────────────

会員登録には個人情報は一切必要ありません。
月300円から匿名でご購入いただけます。

▼ プレミアム会員登録はこちら ▼
http://premium.2ch.net/

▼ 浪人ログインはこちら ▼
https://login.2ch.net/login.php
レス数が1000を超えています。これ以上書き込みはできません。

ニューススポーツなんでも実況